0% found this document useful (0 votes)
22 views354 pages

Corporate Reporting Pq&a Redo

The document outlines a professional level examination for corporate reporting, focusing on the financial statements and consolidation of Bagat Plc and its subsidiaries. It includes detailed financial data, acquisition details, and specific questions related to accounting practices and tax implications. The examination requires candidates to prepare consolidated financial statements and analyze various accounting scenarios based on provided information.

Uploaded by

ookunlola
Copyright
© © All Rights Reserved
We take content rights seriously. If you suspect this is your content, claim it here.
Available Formats
Download as PDF, TXT or read online on Scribd
0% found this document useful (0 votes)
22 views354 pages

Corporate Reporting Pq&a Redo

The document outlines a professional level examination for corporate reporting, focusing on the financial statements and consolidation of Bagat Plc and its subsidiaries. It includes detailed financial data, acquisition details, and specific questions related to accounting practices and tax implications. The examination requires candidates to prepare consolidated financial statements and analyze various accounting scenarios based on provided information.

Uploaded by

ookunlola
Copyright
© © All Rights Reserved
We take content rights seriously. If you suspect this is your content, claim it here.
Available Formats
Download as PDF, TXT or read online on Scribd
You are on page 1/ 354

s

THE INSTITUTE OF CHARTERED ACCOUNTANTS OF NIGERIA

PROFESSIONAL LEVEL EXAMINATION - NOVEMBER

2014 CORPORATE REPORTING

TimeAllowed: 3 hours
SECTION A: COMPULSORYQUESTION (30MARKS)

QUESTION 1

1. Bagat Plc has two subsidiaries (Megat and Mingat) and one associate (Cagat). Since the adoption of
IFRS by Government Bagat has been preparingits consolidated financial statements in accordance
with the principles of International Financial Reporting Standards (IFRS). The draft Statements of
Financial Position of Bagat and its two subsidiaries as at 31 May 2013 are as follows:

BAGAT MEGAT M1NGAT


N'M N'M N'M
Assets:
Non-current assets:
Plant 5,300 4,600 3,220
Investments - Megat 6,000
Mingat 2,560
Associate Cagat 400
Available for Sale Investment 1,020 120 100
15.280 4,720 3.320

Current assets:
Inventory 2,700 1,100 1,460
Trade receivables 1,820 900 640
Cash and cash equivalents 2.040 2.000 160
6.560 4.000 2.260
Total assets 21.840 8.720 5.580

Equity and liabilities:


Share capital 10,400 4,400 2,000
Retained earnings 4,800 3,000 1,600
Other components of equity 240 80 140
Total equity 15,440 7,480 3,740
Non-current liabilities:
Long-term loans 2,400 300 100
Deferred tax 500 180 60
Total Non-current liabilities 2.900 480 160
Current liabilities:
Trade payables 2,300 600 1,200
Current tax payable 1,200 160 480
3,500 760 1,680
Total equity and liabilities 21.840 8.720 5.580

The following information is relevant to the preparation of the group financial


statements:

i. On 1 June, 2012, Bagat acquired 80% of the equity interest of Megat Plc. On the
date of acquisition, the retained earnings of Megat was N2.72billion and other
components of equity were N80million. There had been no new issue of capital by
Megat since the date of acquisition. The purchase consideration comprised cash of
N6billion whereas the fair value of the identifiable net assets of Megat on this
date was N8billion. The excess of the fair value of the net assets is due to an
increase in the value of non-depreciable land. An independent valuer has stated
that the fair value of the non-controlling interests in Megat was N1.72billion on 1
June,
2012. It is the policy of Bagat to measure non-controlling interests on the basis
of their proportionate share in the identifiable net assets of the acquired
subsidiary and not at fair value (full goodwill method).

ii. Also on 1 June, 2012, Bagat acquired 70% of the ordinary shares of Mingat. The
consideration for the acquisition of these shares was N2.56billion. Under the
purchase agreement of 1 June, 2012, Bagat is required to pay the former
shareholders of Mingat 30% of the profits of Mingat on 31 May, 2014 for each of
the financial years to 31 May, 2013 and 31 May, 2014. The fair value of this
arrangement was estimated at N120million at 1 June, 2012 and this value has
not changed. This amount has not been included in the financial statements.

The fair value of the identifiable net assets at 1 June, 2012 of Mingat was
N3.52billion and the retained earnings and other components of equity were
N1.1billion and N140million respectively. There had been no new
issue of share capital by Mingat since the date of acquisition and the excess of the
fair value of the net assets is due to an increase in the value of property, plant
and equipment (PPE). The fair value of the noncontrolling interests in Mingat
was N1.06billion on this date. PPE is depreciated on a straight-line basis over
seven years.

iii. Finally, Bagat acquired a 25% interest in Cagat Plc on 1 June, 2012 for N400million
achieving significant influence over that company in its financial and operating policy
decisions. Cagat Plc retained earnings for the year to 31 May, 2013 was N200 million.

iv. Included in trade receivables of Bagat at 31 May, 2013 is a receivable from Megat
of N30million. Unknown to Bagat, Megat has paid this amount through a bank
transfer by the close of work on 31 May, 2013 but it had not yet been reflected in
the bank statement of Bagat.
Megat has already passed accounting entries to reflect this transaction.

v. Goodwill arising on the purchase of Mingat was tested for impairment on 31 May,
2013 and this provided evidence of impairment to the tune of N36million. No
accounting entries have been passed to reflect the impairment.

Required:

Prepare a consolidated statement of financial position as at 31 May, 2013 for the


Bagat Group. (30 Marks)

SECTION B: ATTEMPTTWO OUT OF THREE QUESTIONS (40 MARKS)

QUESTION 2

(a) Labalaba Plc operations involve selling cars to the public through a chain of retail
car showrooms. It buys most of its new vehicles directly from the manufacturer
on the following terms:
(i) Pay the manufacturer for the cars on the date they are sold to customers or six months
after they are delivered to its showroom whichever is earlier.

(ii) The price paid will be 80% of the retail price as set by the manufacturer at the date that
the goods are delivered.
(iii). Pay the manufacturer 1.5% per month (of the cost to Labalaba) as a 'display
charge' until the goods are paid for.

(iv). May return the cars to the manufacturer at any time up to the date the cars
are due to be paid for and incur the freight cost of any such returns. Labalaba Plc
has never taken advantage of this right of return.

(v). The manufacturer can recall the cars or request them to be transferred to
another dealer at any time up to the time they are paid for by Labalaba.

Required;

Advise the management of Labalaba Plc as to which party bears the risks and
rewards in the above arrangement and show whether there is a sale and how the
transactions should be treated by each party. (7 Marks)
Mr. Ojoowuro, the director of a grocery store, has noticed that the tax charge for
his company is N15million on profits before tax of N105million. This is an effective
rate of 14.3%. Another company, 1rin Plc has an income tax charge of N30million
on profit before tax of N90million. This is an effective rate of tax of 33.3%, yet both
companies state that the rate of income tax applicable to them is 25%. Mr.
Ojoowuro has also noticed that in the statements of cash flows each company has
paid the same amount of tax of N24million.
Required:

Advise Mr. Ojoowuro on the possible reasons why the income tax charge in the
financial statements as a percentage of the profit before tax may not be the same
as the applicable income tax rate and why the tax paid in the statement of cash
flows may not be the same as the tax charge in the statement of profit or loss and
other comprehensive income. (7 Marks)
Maidogo Limited sells N1XAQ, a product manufactured by it, from several retail
outlets. 1n previous years, the company has undertaken responsibility for fitting
the product in customers' premises. Customers pay for the product at the time they
are ordered. The average length of time it takes from ordering to its fitting is 14
days. 1n previous years, Maidogo Limited had not recognised a sale in its books
until the product had been successfully fitted because the rectification costs of any
fitting error would be expensive.
With effect from 1 April, 2013, Maidogo Limited changed its method of trading by
sub-contracting the fitting to approved contractors. Under this policy, the sub-
contractors are paid by Maidogo Limited and they (the subcontractors) are liable
for any errors made in the fitting. Consequently, Maidogo Limited is proposing to
recognise sales when customers order and
N'000
Sales made in retail outlets for the year to 31 March, 2014 69,000

Sales value of NIXAQ fitted in the 14 days to 14 April, 2013 3,600


Sales value of NIXAQ fitted in the 14 days to 14 April, 2014 4,800

Note: The sales value of NIXAQ in the 14 days to 14 April, 2013 are not included in the annual
sales figure of N69million, but those for the 14 April, 2014 are included.

Required:

Discuss whether or not the above represents a change of accounting policy, and calculate the
amount that you would include in the revenue for NIXAQ in the year to 31 March, 2014.
(6 Marks) (Total
20 Marks)

QUESTION 3 Prochain Plc


The Directors of Prochain Plc have pursued an aggressive policy of expansion in the last two
years. They have developed several new products and market share has increased.

The financial statements for the year ended 31 December, 2013 which will be presented to the
Board of Directors at its next meeting is being finalised.

The financial statements at the year-end are presented below:

Statement of profit or loss and other comprehensive income for the year ended 31 December

2013 2012
(N'm) (N'm)
Revenue 34,200 28,900
Cost of sales (24.000) (20.250)
Gross profit 10,200 8,650
Distribution costs & administration expenses (5,120) (3,300)
Finance costs (520) (450)
Profit before tax 4,560 4,900
Income tax (1,300) (1,400)
Profit for the year 3,260 3,500

Other comprehensive income


Total comprehensive income for the year
The results of the company as well as certain key ratios that will form part of the covenants in
respect of the loan facilities will be discussed at the Board of Directors meeting.

Statement of Financial Position as at 31 December

2013 2012
Note N'000 N'000 N'000 N'000
Non-current assets
Property, plant & equipment 25,930 17,880
Available for sale investment 6.200 32,130 5.400 23,280
Current assets
Inventories 4,500 3,600
Trade receivables 4,300 5,200
Cash and cash equivalents - 8.800 120 8,920
Total assets 40.930 32.200
Equity and liabilities
Equity
Share capital (N0.50k) 10,000 10,000
Revaluation reserve 1 4,200 1,100
Other reserves 2 1,800 1,000
Retained earnings 7,460 23,460 4,200 16,300
Non-current liabilities:
Term loan 6,000 6,000
6% bonus bonds (2015) 3 5.400 11,400 5.200 11,200
Current liabilities:
Trade and other payables 5,800 4,700
Short-term borrowings 270 6,070 - 4,700
Total equity & liabilities 40.930 32.200

Notes:

1 The movement on the revaluation reserve relates to property, plant and


equipment revalued in the year.

2 The movement on other reserves relates to the gains on the


investments available for sale.

3.
The bonds are repayable on 1 July, 2015.
The key ratios for the loan covenants include:

Target
i. Gearing 50%
ii. Interest cover 9.5 times
iii. Current ratio 1.5:1
iv. Quick ratio 1.1:1

Required:

(a) Based on the results of Prochain Plc for the year ended 31 December, 2013,
calculate the key ratios for the loan. (8 Marks)

(b) Prepare a report commenting on the financial performance for the year in
relation to the key ratios for the loan. (12 Marks)
(Total 20 Marks)
QUEST1ON 4

(a) The following is the statement of financial position of Lagos Plc as at 31 December, 2013
with its immediate two comparative years.

2013 2012 2011


Non-current assets N'000 N'000 N'000
Property, plant & equipment 30,600 26,010 22,628
Goodwill on business acquisition 4,300 4,300 -
Investment property 15,400 11,704 10,537
50.300 42.014 33.165
Current assets
Inventory 16,900 12,675 11,281
Trade Receivables 21,680 18,862 18,673
Other Receivables 413 560 616
Cash & cash equivalents 5,437 3,821 2,980
44.430 35.918 33.550
Total Assets 94.730 77.932 66.715
Equity and Liabilities
Ordinary share capital 20,612 20,612 20,612
Share premium 600 600 600
Retained earnings 30,163 17,164 12,482
51.375 38.376 33.694
Current liabilities
Trade payables 24,000 21,120 19,008
Other payables 6,816 7,210 6,273
Bank overdraft 11,223 9,299 6,044
Current tax payable 1,316 1,927 1,696
43,355 39,556 33,021
pay for the goods rather than when they have been fitted. Details of the relevant sales
figures are;
The management of Lagos Plc is not sure of the impact of IAS12 (Income Taxes) on its
retained earnings as at 31 December, 2013 as well as what the new deferred tax
balance will be on migrating to IFRS.

The following information was also available as at the year end.

N'000
Tax written down value of PPE 40,300
Tax written down value of goodwill on business acquisition 4,300

Tax base of trade receivables 29,800


Tax base of trade payables 13,000

Assume that current tax has been correctly computed in line with the applicable tax
laws at 30%.

Required:

Using relevant computations, advise the management of Lagos Plc on the impact of
deferred tax calculated on retained earnings in accordance with IAS 12. (12 Marks)

(b) On 1 June, 2013, Bam Plc acquired Mango Limited for N3,150million. The fair value of
the identifiable net assets of Mango Limited at this date was N2,550million and
retained earnings and other components of equity were N825million and N105million
respectively. Mango Limited share capital was N1,500million.

The excess of the fair value of the net assets is due to an increase in the value of
property, plant and equipment.

Required:

Evaluate the impact of full deferred tax on the excess of the fair value of the net assets
attributable to increase in the value of property, plant and equipment of Bam Plc.
(8 Marks)
(Total 20 Marks)

SECTION C: ATTEMPT TWO OUT OF THREE QUESTIONS (30 MARKS)

QUESTION 5
Critics of traditional corporate financial reporting under Generally Acceptable Accounting
Practice (GAAP) argue that financial statements alone are not considered sufficient without a
narrative that provides a context within which to interpret the financial position, financial
performance and cash flows of an entity.
A financial expert within the board of Abcon Kombe Plc, aware of the above criticism has
proposed that Abcon Kombe Plc should include in its financial statements, management
commentary to satisfy the numerous analysts that use its annual reports.
Required:

(a) Advise the Board on FIVE elements of information which IFRS Practice
Statement expects to be included in management commentaries in order to meet its
objectives. (5 Marks)
(b) Relate the FIVE elements ofinformation above to the needs of the various
primary users. (7 Marks)

(c) Justify why management commentaries should be made compulsory in


Nigeria's financial reporting environment. (3 Marks)
(Total 15 Marks)

QUESTION 6

(a) An entity is normally viewed as a going concern. It is assumed that the entity has neither
the intention nor the desire of liquidation or of curtailing materially the scale of its
operations.
However, if the going concern is threatened, the financial statements would be prepared
on a different basis.

Required:

State the factors that indicate an organisation may no longer be a going concern under
the following categories.

i. Financial

ii. Operations

iii. Legal or regulatory (6 Marks)

(b) Luck & Co. has been making losses over the last few years. Its statement of financial
position at 31 December, 2013 showed the following:

Luck & Co. Statement of financial position as at 31 December, 2013

N N
Non-current assets:
Property, plant and equipment 80,000
Current assets:
1nventory 20,000
Receivables 40.000 60.000
140,000
Equity and Liabilities:
Ordinary capital 100,000
Retained earnings (140,000)
Secured loan stock 100,000
Payables 80.000
140,000
On liquidation, the assets would realise the following:

N
Property, plant and equipment 30,000
1nventory 12,000
Receivables 36,000
78.000

1f the company continues to trade for the next four years, profit after charging N20,000
per annum as depreciation on the property, plant and equipment would be as follows:

N
2014 4,000
2015 20,000
2016 26,000
2017 28.000
78.000

Assume that there would be no surplus cash to settle the payables and loan- stock
holders until after four years when inventory and receivables could be realised at their
book values.

Required:

Evaluate the financials and advise the management of Luck and Co on the options
available to them and redraft the statement of financial position of Luck and Co after
the exercise. (9 Marks)
(Total 15 Marks)
QUESTION 7

The Chief Financial Officer (CFO) of Niger Breweries Plc, a company listed on the Nigerian
Stock Exchange recently attended IFRS certification training of the Corporate Finance Faculty
of the Institute of Chartered Accountants of Nigeria (ICAN). He realised that due to the
complexity of International Financial Reporting Standards (IFRS), judgments used at the time
of transition to IFRS may result in prior year adjustments and changes in estimates disclosed
in financial statements.

He also understood from the introductory session on IAS 8 (Accounting Policies, Changes in
Accounting Estimates and Errors), that the selection of accounting policy and estimation
techniques is intended to aid comparability and consistency in financial statements. Among
other things, IFRS also places particular emphasis on the need to take into account qualitative
characteristics and the use of professional judgment when preparing financial statements. The
IFRS may appear prescriptive and the achievement of all the objectives for a set of financial
statements will rely on the skills of the preparers. When selecting or changing accounting
policies, changing estimation techniques, and correcting errors, entities should follow the
requirements of IAS 8.

However, the application of IAS 8 is usually often dependent upon the application of
materiality analysis to identify issues and guide reporting. Entities also consider the
acceptability of the use of hindsight in their reporting.

With the above knowledge, the CFO of Niger Breweries is still not clear about the
circumstances under which Niger Breweries can change its accounting policies conscious that
IAS 8 emphasises consistency in the application of accounting policies once chosen.

Required:

a. Advise the CFO on the circumstances where an entity may change its accounting
policies, setting out how a change in accounting policy is applied and the difficulties
faced by entities when a change in accounting policy is made.
(8 Marks)

b. Discuss why the current treatment of prior period's errors could lead to earnings
management by companies, together with any further arguments against the current
treatment.

(7 Marks)
(Total 15 Marks)
SOLUT1ON 1
BAGAT PLC GROUP

CONSOL1DATED STATEMENT OF F1NANC1AL POS1T1ON AS


MAY 2013
AT 31 ASSETS Notes N'M N'M
Non-current assets
1 14,160
Property, plant and equipment Goodwill
2 180
1nvestment in Associate Available for sale
3 450
financial assets
4 1.240
Current Assets 16.030
5 5,260
1nventories
6 3,330
Trade Receivables 12,820
7 4,230
Cash and cash equivalent 28.850

Equity and liabilities


Equity attributable to owners of the parent 10.400
Share capital 5,760
8 240
Retained earnings
Other component of equity 16.400
2.850
Non-controlling interest 9
19,250

Non-current liabilities 10 2,800


11
Long-term liabilities Deferred tax 740
3.540

Current liabilities
Trade and other liabilities 12 4,220
Current tax payable 13
1,840
6.060 9,600
28.850
Notes

N'M
1 Property, plant and equipment
Plant- Bagat 5,300
-Megat 4,600
-Mingat 3,220
Fair Value -Non-depreciable Land (wk1) 800
Fair Value Adjustment (wk2) 280
14.200
Depreciation Adjustment (wk2) (40)
14.160

2 Goodwill
On acquisition of Mingat 216
Goodwill Impairment (wk5) (36)
180

3 Investment in Associate:
Associate -Cagat 400
Post-acquisition Retained Earning (Wk3) 50
450

4 Available for Sale Financial Assets


- Bagat 1,020
- Megat 120
- Mingat 100
1.240

5 Inventories
- Bagat 2,700
- Megat 1,100
- Mingat 1,460
5.260

6 Trade Receivables
- Bagat 1,820
- Megat 900
- Mingat 640
Receivable from Megat (Intergroup) (30)
3.330
7 Cash and Cash equivalent
- Bagat 2,040
- Megat 2,000
- Mingat 160
Intergroup receivable from Megat 30
4.230

Consolidated retained earnings Bagat Megat Mingat


As at 31.05.2013 4,800 3,000 1,600
At Acquisition (2,720) (1,100)
Gain on bargain purchase (wk1) 400.00
Share of post acquisition retained
Prof Ass (wk3) 50.00
Additional dep. PPE Fair value (ek2) 0.00 0 (40)
Impairment of goodwill (wk5) (36.00)
Post acquisition retained profit 5,214.00 _28Q_ 460
Bagat share of post prof in megat 224.00
Bagat share of post prof in minget 322.00
5.760.00
Non-controlling interest Meya Minaga Total

At acquisition (wk 1&2) 1,600 1,056


Post 56 138
1,656 1,194 2.850
Alternatively NCI can be determined
as follows:
At consolidation date
20% x 7,480 1,496
30% x 3,740 1,122
Fair value Adj 800x 20% & 240 x 30%
(wk 1&2) 160 72
NCI 1,656 1,194 2,850
N'M

Long-term liabilities
- Bagat 2,400
- Megat 500
- Mingat 100
2,800
Deferred tax

- Bagat 500
- Megat 180
- Mingat 60
740
12 Trade and other liabilities
- Bagat 2,300
- Megat 600
- Mingat 1,200
- Contigent consideration 120
4.220

13 Current tax payable


- Bagat 1,200
- Megat 160
- Mingat 480
1.840

Workings
Bagat NCI

1 Purchase of Megat
Purchase consideration 6,000
Fair value of assets acquired
Share capital 4,400
Retained earnings 2,720
Other components of equity 80
FV Non-depreciable land (balancing figure) 800
Total fair value of net assets 8.000
Bagat/NCI at 80%/20% 6,400 1.600
Gain on bargain purchase (400)
The land is non depreciable, so no
depreciation adjustment.
Gain on bargain purchase is taken to earnings
going by IFRS 3

2 Purchase of Mingat Bagat NCI


Purchase consideration 2,560
Contingent consideration 120
Total consideration FV 2,680
Fair value of assets acquired
Share capital 2,000
Retained earnings 1,100
Other components of equity 140
3,240
FV PPE (balancing figure) 280
Total fair value of net assets 3.520
Bagat/NCI at 70%/ 30% 2,464 1.056
216
For the fair value adjustment, accounting entry will be
PPE
Fair Value Adjustment PPE
The fair value adjustment PPE is pre and forms part of NA at acq.

In addition there will be depreciation adjustment 280/7 years i.e DR CR


40
P or L Mingat 40
PPE 40
The contingent consideration is taken to current liabilities

3 Investment In Associate Cagat


We are interested in post acquisition
retained profit i.e 25% x 200 i.e N50 million
investment in Associate Cagat 50
Consolidated retained earnings 50

4 Receivable from Megat N30 million


This amount still appears as trade
receivables in the books of Bagat
Adjusting entry will be
Cash and cash equivalent 30
Trade receivables 30

5 Goodwill impairment
Consolidated retained earnings 36
Goodwill on consolidation 36

EXAMINER'S REPORT

The question tests students' understanding of preparation of Consolidated Statement of


Financial Position in accordance with international Financial Reporting Standards (iFRS).
Major areas of coverage are the determination of fair value of assets acquired and goodwill on
acquisition, consolidation of retained earnings and computation of non-controlling interests.

Almost all the candidates attempted the question and performance was very poor.

The commonest pitfall was the inability of the candidates to determine goodwill based on the
fair value of assets acquired rather than proportionate share of equity and its other
components.
SOLUTION 2

(a) LABALABA PLC

The issue involved here is an example of a sales or return transaction. From


LABALABA'S point of view, the main issue is to determine the point at which purchase
has taken place and when recognition of revenue should occur. Usually, in the case of
this type of agreement, there is conflicting evidence as to which party bears the risk
and rewards relating to the cars.

By virtue of the arrangements in place as regards the substance of the transaction, the
manufacturer retains the legal right of ownership until the goods are paid for by
LABALABA Plc.

Consistent with this, the manufacturer also has the right to have the goods returned or
passed on to another dealer. The fact that LABALABA Plc. can equally return the cars
to the manufacturer before the expiration of six months after they are delivered to its
show room suggests that the manufacturer is also exposed to the risk of obsolescence or
fall in prices or values.

Judging from the above, it appears that the cars have not been sold and should
therefore remain in the inventory of the manufacturer and not recognized in the
accounting records of LABALABA Plc until they are either paid for or after the
expiration of the six months from delivery whichever is earlier.

However, looking at some other factors in the transaction, contrary view may equally be
expressed. These factors are:

■ The price for the goods is fixed as at the transfer date but not at the date that
they are deemed 'sold'. This therefore implies that LABALABA Plc is already
protected against possible increase in price.

The 1.5% paid to the manufacturer appears to be a finance charge even though
it is referred to as 'display charge'. A finance charge of this nature indicates that
LABALABA Plc. has already incurred a liability on the purchase of such goods
from the manufacturer. This liability is the account payable in respect of the
cost of the cars.
LABALABA Plc has right of return, but has not exercised it because of the cost
of return.

■ There is an explicit freight cost but this may not be the only cost.

■ The fact that LABALABA Plc has a right to return the goods within a
specified time frame but has not exercised that right, suggests possibly
that it may not be a reasonable option that it can explore.

■ it could well be that LABALABA may suffer inadequate future supplies


from the manufacturer if it returns the goods.

Arising from the foregoing, the sales will not be recognized in the books of the
manufacture; hence it will be included in its inventory.

(b) MR. OJOOWURO

Effective tax rate can be obtained by expressing the amount of tax charge as a
proportion of profit before tax for the same accounting year, in view of this,
effective tax rate may not be the same for any two entities when compared
together. The reasons for this difference may be embedded in the different
issues that come into play in determining the tax charge for the year, in
arriving at tax charge in an income statement in a particular reporting year, the
following issues must be borne in mind:

■ Current tax liability


This represents the amount of income tax payable in respect of the
taxable profit for the current year which may differ considerably from the
accounting profit. This may not be the same for two entities such as
Ojoowuro and irin even though they posted the same profit for that year
because of differences in the two entities allowable and non-allowable
expenditure which will invariably affect their taxable profit differently.
■ Under/over provision
This is the difference between the actual provisions made in respect of
chargeable tax in previous years when compared with the actual year tax
liability as determined by the relevant authority. The adjustment for
under/over provision for tax in previous years in the current year income
statement will affect the current year tax charge to profit or loss. While
under-provision increases the current year tax charge, over-provision on
the other hand will reduce it.
In the case of Ojoowuro, it is possible that it had over-provision in the previous
year hence its income tax charge for the current year has a reduction effect. On
the other hand, Irin might have under provided for tax last year hence the need
to charge more in the current year that has no bearing with its level of profit for
the current year.

■ Another reason for differences in the amount of income tax charge for
different companies is the effect of deferred tax. Deferred tax is the estimated
future tax consequences of transactions and events recognized in the financial
statements of the current and previous periods. When there is deferred tax
adjustment occasioned by either taxable temporary or deductable temporary
differences in the current year, the tax charge to income statement in the
current year will be affected. While increase in deferred tax liability in the
current year will increase income tax chargeable, a decrease will give rise to
decrease in income tax charge.

In the case of increase in deferred tax asset, it will have a reduction effect on
income tax charge and if otherwise, an increase.

■ Other more complex items such as multiple tax reliefs may also be included in
issues that affect tax charge.

The combined impact of the above factors will have effect on the effective tax rate of
an entity.

However, the effect of taxation, in the statement of cash flow is a different issue
entirely. The reason for this is that statement of cash flow only captures cash
movement. For instance, an entity may have a tax liability of N10million and only
able to pay N5million during the year. It is only the amount paid in respect of such
taxes that will be captured in the statement of cash flows while the outstanding
balance will be reflected in the statement of financial position. It is also possible for
an entity to have settled substantially part of its tax liability for the current year
through withholding tax payments.

Withholding tax as a tax deducted at source will be available as tax credit against
tax liability in the current year thereby reducing the outstanding tax payment that
will affect the statement of cash flows. Other differences may
be due to items such as deferred tax movements that do not affect cash
flows.

(c) MAIDOGO LIMITED

The issue here is one of when revenue should be recognized in the income
statements. It involves identification of:

■ When the transfer of significant risks takes place;


■ Whether the entity retains any continuing managerial involvement to the degree
usually associated with ownership;
■ If the amount of revenue can be measured reliably;
■ Whether it is possible that the economic benefits associated with the transaction
will flow to the entity.
Judging from the general guidance in IAS 18- Revenue, the idea of recognising
revenue after the NIXAQ is fitted was quite appropriate. At that time, the order
can no longer be cancelled, all monies could have been received and the goods
delivered. However, given a change in method of trading by sub-contracting
the fitting to approved contractors who are liable for any error made in the
fitting, the directors want to change the timing of revenue recognition.

Given the change in the circumstances of these transactions as a result of the


involvement of the sub-contractors to bear the risk of fitting NIXAQ it may be
appropriate for the entity to change the timing of recognition of revenue
bearing in mind the conditions precedent for revenue recognition stated above
in line with IAS 18.

Notwithstanding, if this change is justified, it cannot be construed to represent


a change in accounting policy; hence it will not require retrospective
application. It will be treated as an ordinary change in accounting estimate
which will require a prospective application.

Arising from the foregoing the amount to be included in sales


revenue for NIXAQ in the year to 31 March, 2014 without the
change in policy will be as indicated below: N'000

Sales in retail outlet for the year 69,000


Sales value of NIXAQ fitted in the 14 days to 14 April 2013 3,600
72,600
Less sales value of NIXAQ fitted in the 14 days to 14th April 2014 4,800
Sale 67,800
i. e (sales of N4.8m from retail premises not fitted will not be recognized).

If there is a change in accounting policy, the new accounting policy should be applied as
if it had always been in place and the income recognized in the year to 31 March 2014
would be N69 million.

Conclusively, changes in accounting procedures resulting from circumstances that


differ from previous circumstances is not a change of accounting policy. Thus the
amount to be recognized in income for the year to 31 March 2014 would be:

N'000
Sales in retail outlet 69,000
Sales value of NIXAQ fitted in the 14 days to 14th April 2013 3.600
Total sales revenue for the year to 31 March 2014 72.600

Given this change in the recognition of revenue, there may be need for the management of
MAIDOGO LTD to begin to make provision for warranty obligation which will be based on
past experience.

EXAMINER'S REPORT

This three-part question tests candidates understanding of principles and practices involved in
the transfer of risks and rewards in a sale or return transaction. Income Tax treatment in
financial statements, and revenue recognition in Income Statement.

Most of the candidates attempted the question and performance was fair.

Candidates commonest pitfall was the use of income recognition principles in place of practices
for the transfer of risks and rewards in sale or return transactions.

Candidates are advised to understand the requirements of questions being attempted to


assure improved performance.

SOLUTION 3

(a) Computation of Key Ratios for the Loan Covenants

Year 2013 Year 2012 Target


(i) Gearing: Fixed Int. Capital x 100 11,400 x 11,200 x
Fixed Int.Capital+Equity 100 100
11,400 + 11,200 +
23,460 = 32.7% 16,300 40.73% 50%

i i i Profit before Int &Tax 4,560 + 520 4,900 + 450


UU Interest Cover : — --------- ------------
520 = 450
9.8 11.9
times times
The comments are based on the financial statements of Prochain Plc and the ratios calculated are based on the annual report as
presented in (a) above
Current Ratio : Current Assets 8,800 8,920
Current Liabilities 6,070 4,700
1.45:1 1.90:1
1.5:1

(iv) Quick Ratio: Current Assets less Inventory 8,800 -4,500 8,920 - 3,600
Current Liabilities
6,070 = 0.71 :1 4,700
1.13:1
1.1:1

Other ratios Necessary for the performance Review are:

Profitability Ratios

(v) ROCE — Pro f^ bef°re interest _ PBIT


Capital Employed Equity+longterm debt

2013 2012
_ 4, 560+520 4,900 +
450
_ 23, 460+ 11,400 16,300 + 11,200
= 14.6% = 19.5%

Gross
(vi) GROSS PROFiT % =
Profit
Turnover
2013 2012
_ 10,200 8,650
_ 34,200 28,900
= 29.8% = 29.9%

Pro fit Before interest & Tax


(vii) PROFIT MARGIN =
Turnover

2013 2012
_ 4, 560+520 _ 4,900 + 450
34,200 28,900
= 14.9% = 18.5%

(c) Report on Prochain Plc.


The comments are based on the financial statements of Prochain Plc and the ratios
calculated are based on the annual report as presented in (a) above.
The Quick ratio also decreased from 1.13 in 2012 to 0.71 in 2013.

Financial Performance

(i) Aggressive expansion policy pursued in the last two years helped in increasing
revenue by 18% while the company was able to maintain the gross profit margin
at 29%.

However, the profit margin dropped by 3.6%, while distribution and


administrative expenses as well as finance cost in the period increased
substantially by 55%.

(ii) Return on Capital Employed has fallen from 19.5% to 14.6%. This is because
profit is decreasing while capital employed is increasing due to revaluation of
assets.

Financial Position: (based on the key ratios for Loan Covenants)

(iii) Gearing
The Company's gearing has improved, decreasing to 32.7% from 40.73% which
is within threshold set by the bank (i.e Target of 50%)
This shows that there is a reduction in the level of financial risk bearing faced
by Prochain Plc.

Bonds payable are due for repayment in less than 12 months' time-July 2015.
The Company needs to secure some longer term funding to trade and expand as
well as repay the bonds; hence the application for the bank Loan.

(iv) Interest Cover

The interest cover dropped from 11.9 times in year 2012 to 9.8 times in year
2013. Although, the Company met the target of 9.5times , the bank will be
concerned about the falling margin and the effect on the Interest cover.

The interest is 6% on the bond, a funding with a significantly higher associated


cost is likely to decrease the interest cover further. The interest cover is already
close to the acceptable threshold set by the bank.

(v) Current and Quick Ratio

The Current ratio dropped below the target from 1.9 in 2012 to 1.45 in 2012. The Quick
ratio also decreased from 1.13 in 2012 to m0.71 in 2013
Changes in the two ratios moved against the target of 1.5:1 and 1.1:1 for current ratio and
quick ratio in year 2013 respectively.

This working capital problem may be an issue to be given further consideration by the Board.

Cash held decreased from a positive balance of N120,000,000 to an overdraft of N270,000.

The company should consider the option of selling investments which performed well in the
year to inject more working capital.

EXAMINER'S REPORT

The question examines the computation, interpretation and application of ratios.

Majority of the candidates attempted the question. Candidates' performance on ratio


computation was fair but most of them were unable to interprete correctly the computed ratios.

Candidates should undertake in-depth study and practice questions covering all areas of the
syllabus.

SOLUTION 4
Relevant Asset/Liability Carrying Tax Base Temporary Tax Rate Asset/
Liability
(a) LAGOS PLCAmount Difference
N'000 N'000 N'000 % N'000
Property, Plant & Equipment 30,600 40,300 9,700 30% 2,910
Good will 4,300 4,300 - - -
Trade Receivables 21,680 29,800 8,120 30% 2,436
Trade Payables 24,000 13,000 11,000 30% 3.300
Total Deferred Tax Assets at
31 December, 2013 N8.646
Computation of Deferred Tax Asset/Liability

Journal Entries:

DR Deferred Tax (in SOFP)


CR Deferred Tax credit (in SCi)
With N8,646,000

Advice to Management of Lagos Plc.


■ Given the above background information, the temporary differences in
Lagos Plc deferred tax computation result in deductible temporary
differences thereby resulting into a total deferred tax asset of
N8.646million at the end of 31 December 2013.

■ The required treatment therefore is to debit (DR) deferred tax assets


and credit (CR) retained earnings provided it is probable that there
shall be available future taxable profit to utilize the deferred tax asset.

■ The effects of this deferred tax asset is that it will increase the Non-
current assets of Lagos Plc in the Statement of Financial Position at 31
December 2013 by N8.646 million and increase the retained earnings
by the same amount in the Statement of changes in equity.

■ The deferred tax balance is a deferred tax asset which will be classified
as a Non-current asset.

(b) BAM PLC ACQUIRED MENGO LIMITED.

Calculation of Deferred Tax on Fair Value Adjustment

N'M N'M
2,550
Fair value of net assets Less:
Book value of net assets: 1,500
Ordinary shares Capital
825
Retained Earnings Other
105
equity
(2,430)

Fair value Adjustment (subject to deferred tax) 120

Deferred Tax Liability (30% x 120) 36

Goodwill on initial recognition = purchase consideration - fair value of Net Asset acquired =
N(3,150 - 2,550)m = N600m.

Final Goodwill recognized = N(600-36)m = N564m.

The amounts that arise in deferred tax on business acquisition is adjusted to the amount of
goodwill.
EVALUATION COMMENT:

■ Based on the above calculation, since the excess of the fair value of identifiable net
assets over the book value of same assets on acquisition is mainly attributable to
differences in property, plant, and equipment which will later be subjected to
depreciation, such difference will give rise to deferred tax liability which should be
incorporated into the financial statements as at the date of acquisition.

■ Furthermore, the amount that arise in deferred tax on business acquisition is adjusted
to the amount of goodwill.

EXAMINER'S REPORT

The question examines the provisions and requirements of IAS 12 (Income Taxes) on Deferred
Taxes and their impact on Retained Earnings.

About 50% of the candidates attempted the question and performance was below average.

Candidates displayed a very poor understanding of the relevant provisions of the standard on
Deferred Assets/Liabilities.

There is need for candidates to be more diligent in their study of all the standards.

SOLUTION 5

(a) Five elements of information which IASB practice statement expects to be included in
management commentaries are:

(i) The nature of the business.

(ii) Management objectives and its strategies for meeting those objectives.

(iii) The entity's most significant resources, risks and relationships.

(iv) The results of operations and prospects.

(iv) The critical performance measures and indicators that management was
to evaluate the entity's performance against stated objectives.
(b) (i) Nature of business

The knowledge of the business in which a company/ entity engages in and the
external environment a business operates will enable the users to have further
information about an entity and how it works.

(ii) Objectives & strategies

It provides readers of commentaries in financial statements opportunity to


assess the strategies adopted by the entity and the likelihood that those
strategies will be successful in meeting management's stated objectives.

(iii) Resources, risks & relationships

This is a basis for determining the resources available to the entity as well as
obligations to transfer resources to others. The ability of the entity to generate
long term sustainable net inflow of resources and the risks to which those
resources generating activities are exposed, both in the near and in the long-
term will be of benefit to the users.

(iv) Results and prospect

Users of financial statements would also need to understand whether an entity


has delivered in line with expectations and, implicitly, how well management
has understood the entity's market, executed its strategy and managed the
entity's resources, risks and relationships.

(v) Performance & measures

Users should be able to focus on critical performance measures and


indicators that management uses in assessing and managing the entity's
performance against stated objectives & strategies.

(c) Reasons for Management commentaries

(i) Promoting an entity and attraction of investors, lenders, customers and


other Stakeholders.

(ii) Enables management to communicate its plans and outlook to the users.

(iii) Enhanced understanding of financial statements for better decision


making.
(iv) it makes the report more explanatory, transparent and acceptable.

(v) Without mandatory management commentaries, directors may take a


minimalist approach to disclosure which will make financial reports inadequate
and less informative.

EXAMINER'S REPORT

This question tests candidates' knowledge of the provisions, requirements and rationale of
iFRS Practice Statement relating to Management Commentaries in Financial Statements.

Most of the candidates attempted the question but performance was very poor.

Candidates generally displayed poor knowledge of the provisions of the iFRS Practice
Statements.

Emphasis should be placed on all aspects of iFRS provisions and requirements for improved
performance in this paper.

SOLUTiON 6

(a) The Financial factors that indicate an organization may no longer be a going

(i) Financial concern are:

■ Default on loan or similar agreements


■ Arrears as to payment of dividend
■ Denial of usual trade credits from suppliers
■ Restructuring of debt
■ Recurring operating losses
■ Negative workings capital
■ Negative cash flow from operating activities
■ Adverse key financial ratios
■ Non-compliance with statutory capital requirements
■ Need to dispose off substantial assets.

(ii) Operations

■ Labour difficulties i.e strike, high labour turnover etc.


■ Emergence of a highly successful competitor
■ Loss of key customers
■ Decrease in market share
■ Withdrawal of Operating licence where applicable
■ Loss of principal suppliers

(iii) Legal or regulatory factors

■ Changes in law or regulations that adversely affect the entity


■ Pending legal or regulatory proceedings against the entity
■ Uninsured or underinsured insurable risks
■ Natural disaster such as drought, catastrophe, earthquake or flood.
■ Withdrawal of operating licence

(b) Options available to the management of Luck & Co:

Option 1

If liquidation takes place now, the amount that will be available to the loan stock
holders is N78,000, leaving them with a deficiency of N22,000. In this case, there would
be nothing for the creditors and shareholders.

Option 2

If trading continues for the next four years and the estimated results are achieved, the
available cash would be as follows:

N
Profit 78.000
Depreciation 80.0
Inventory 20,000
Receivables 40,000
218.000
With this option, loan stockholders and creditors would be paid in full N100,000 and
N80,000 respectively, leaving N38,000 available for the shareholders.

However, the loan stockholders probably have the right to appoint a receiver and would
insist on some compensation for not enforcing their right.

In addition, the creditors might also expect compensation for having to wait for four
years before receiving payment.
Option 3

The loan stockholders and creditors may be persuaded to waive the amounts owed to
them in exchange for ordinary shares to enable them have full participation in the
future profit of the company. Terms of such an exchange might be as follows:

(i) 75,000 units of N1 ordinary shares to the loan stockholders (3 of N1 ordinary


shares for every N4 of loan stock)
(ii) 40,000 units of N1 ordinary shares to the creditors (1 of N1 ordinary shares for
every N2 due)
(iii) 25,000 units of N1 ordinary shareholders (1 of N1 ordinary shares for every 4 of
the existing N1 ordinary share.

LUCK & CO

Redrafted Statement of Financial position after the exercise.

N
Assets:
Property, Plant and Equipment 80,000
inventory 20,000
Receivables 40,000
140.000
Equity 140.000

EXAMINER'S REPORT

This question examines candidates understanding of the financial, operational and legal/
regulatory indicators of going concern challenges in an entity and available options in tackling
them.

Most of the candidates attempted the question but performance was poor.

Candidates were unable to differentiate between financial, operational and legal /regulatory
indicators.

Candidates need to read standard texts on all aspects of the syllabus.


SOLUTION 7

International Accounting Standard 8 (IAS8) Accounting Policies, Changes in Accounting Estimates and Errors
prescribes the criteria for selecting and changing accounting policies, accounting for changes in
estimates and reflecting corrections of prior period errors. Changes in accounting policies and
corrections of errors are generally accounted for retrospectively, unless this is impracticable;
whereas changes in accounting estimates are generally accounted for prospectively.

(a) Circumstances for Change of accounting Policy

An entity can change an accounting policy only if:

• It is required by a standard e.g.an IFRS or


• The change results in the financial statements providing reliable and more
relevant information i.e. voluntary change.

Application or treatment of Change of accounting Policy:

If the change is required by a Standard, an entity shall account for the change in accordance
with the specific transitional provisions, (i.e. the standard may specify retrospective
application or prospective application), if any.

Where there are no specific transitional provisions in the Standard requiring the change in
accounting policy or an entity changes an accounting policy voluntarily, it should apply the
change retrospectively.

Where a change in accounting policy is applied retrospectively, an entity should adjust the
opening balance of each affected component of equity for the earliest prior period presented
and the other comparative amounts for each prior period presented as if the new accounting
policy had always been applied.

Difficulties in treatment of change of accounting policy:

Majority of the difficulties are related to the impracticability of applying the change
retrospectively. This makes it difficult to achieve comparability of prior periods with the
current period. For instance:

(i) Data might not have been collected in the prior periods to allow for retrospective
application.
(ii) Restating comparative information for prior periods often requires complex and
detailed estimation.

(iii) When making estimates for prior periods, the basis of estimation should reflect the circumstances which existed
at the time and it becomes increasingly difficult to define those circumstances with the passage of time.

(iv) Estimates and circumstances might be influenced by knowledge of events and circumstances which have arisen
since the prior period.

(v) The effects of the retrospective application or restatement may not be determinable.

(vi) The retrospective application/restatement requires assumption about what management's intent would have
been in that period. IAS 8 does not permit the use of hindsight when applying a new accounting policy, either in making
assumptions about what management's intentions would have been in a prior period or in estimating amounts to be
recognised, measured or disclosed in a prior period.
The standard permits exemption from this requirement when it is impracticable to determine either the period-specific
effects or cumulative effect of the change. When it is impracticable to determine the effect of a change in accounting
policy on comparative information, the entity is required to apply the new accounting policy to the carrying amounts of
the assets and liabilities as at the beginning of the earliest period for which retrospective application is practicable. This
could actually be the current period but the entity should attempt to apply the policy from the earliest date possible.

Prior period errors and earnings management

The term "error" refers to an unintentional misstatement in financial reports, including the omission of an amount or a
disclosure. Material prior period errors are corrected retrospectively in the first financial statements issued after their
discovery. Correction is made by restating the comparative amounts for the prior period(s) presented in which the error
occurred. If the error occurred before the earlier comparative prior period presented, the opening balances of assets,
liabilities and equity for the earliest prior period should be restated to reflect correction of the error(s). IAS 8 Accounting
Policies, Changes in Accounting Estimates and Errors requires prior period errors to be amended retrospectively by
restating the comparatives as if the error had never occurred.

Hence, the impact of any prior period errors is shown through retained earnings rather than being included in the
current period's profit or loss. Managers could use this treatment for prior period errors as a method for manipulating
current period earnings. Restatements due to errors and irregularities can be
considered to indicate poor earnings quality and to threaten investor confidence, particularly if they occur
frequently. Thus, it might appear that the factors associated with earnings corrections could be linked to
earnings management.

Earnings Management, specifically Income Smoothing, is a form of creative accounting which involves:

• The use of inappropriate provisions (this reduces profit in good years and increases them in poor years)
• Not providing for liabilities, either at all or not in full, as they arise or
• Restructuring costs not being charged to income

Arguments against the approach in IAS 8 are:

- That the standard allows inappropriate use of hindsight;

- That the treatment renders errors less prominent to users; and


- That it allows amounts to be debited or credited to retained profits without ever being included in a
current period profit or loss.

Managers have considerable discretion regarding the degree of attention drawn to such changes. The
information content and prominence to users of disclosures regarding prior period errors are issues of
significance, with potential economic and earnings quality implications. Expenses could be moved backward
into a prior period, with the result that managers are given a possible alternative strategy with which to
manage earnings. It is possible to misclassify liabilities, for example, as non-current rather than current, or
even simply miscalculate reported earnings per share. Under IAS 8, the prior period error can then be
amended the following year, with no lingering effects on the statement of financial position as a result of the
manipulation.

EXAMINER'S REPORT

The question tests the provisions and requirements of IAS 8 (Accounting policies, changes in Accounting Estimates
and Errors).

About 50% of the candidates attempted the question but they displayed inadequate understanding of its
requirements.

Most of the candidates that attempted the question displayed poor understanding of the provisions on treatment of
prior period errors.
Candidates are advised to thoroughly study and practice with standard examination questions on all aspects of
iFRS.
THE INSTITUTE OF CHARTERED ACCOUNTANTS OF NIGERIA

PROFESSIONAL LEVEL EXAMINATIONS - MAY 2015

CORPORATE REPORTING Time Allowed: 3 hours ATTEMPT FIVE QUESTIONS IN ALL

SECTION A: COMPULSORY QUESTION (30 MARKS)

QUESTION 1
1. Barewa Plc has two subsidiary companies and one associate. Since the adoption of
International Financial Reporting Standards (IFRS) by companies listed on the Nigeria
Stock Exchange, Barewa has been preparing its consolidated financial statements in
accordance with the provisions of International Financial Reporting Standards (IFRSs).
The draft Statements of Financial Position of Barewa and its two subsidiaries as at 31
May, 2013 are as follows:

Barewa Megida Mindara


N’m N’m N’m
Assets:
Non-current assets:
Plant 2,650 2,300 1,610
Investments - Megida 3,000
Mindara 1,280
Associate Calamari 200
Available for sale 510 60 50
7,640 2,360 1,660

Current assets:

Inventory 1,350 550 730


Trade receivables 910 450 320
Cash and cash equivalent 1,020 1,000 80
3,280 2.000 1,130
Total assets 10.920 4,360 2,790

Equity and liabilities:

Share capital 5,200 2,200 1,000


Retained earnings 2,400 1,500 800
Other components of equity 120 40 70
Total equity 7,720 3,740 1,870

Non-current liabilities:

Long-term loans 1,200 150 50


Deferred tax 250 90 30
Total non-current liabilities 1,450 240 80

Current liabilities:

Trade Payables 1,150 300 600


Current tax payables 600 80 240
1,750 380 840
Total equity and liabilities 10.920 4,360 2,790

The following information is relevant to the preparation of the group financial


statements:
(i) On 1 June 2012, Barewa acquired 80% of the equity interest of Megida plc. On the
date of acquisition, the retained earnings of Megida was N1.36billion and other
components of equity was N40million. There had been no new issue of share
capital by Megida since the date of acquisition. The consideration comprised cash
of N3billion whereas the fair value of the identifiable net assets of Megida on this
date was N4billion. The excess of the fair value of the net assets is due to an
increase in the value of non-depreciable land. An independent valuer has stated
that the fair value of the non-controlling interest in Megida was N860million on 1
June, 2012. It is the policy of Barewa to measure non-controlling interest on the
basis of their proportionate share (interest) in the identifiable net assets of the
acquired subsidiary and not at fair value (full goodwill method).

(ii) Also on 1 June 2012, Barewa acquired 70% of the ordinary shares of Mindara. The
consideration for the acquisition of these shares was N1.28billion. Under the
purchase agreement of 1 June, 2012, Barewa is required to pay the former
shareholders 30% of the profits of Mindara on 31 May, 2014 for each of the
financial years to 31 May, 2013 and 31 May, 2014. The fair value of this
arrangement was estimated at N120 million at 1 June, 2012. This value had not
changed and has not been included in the financial statements.

The fair value of the identifiable net assets at 1 June, 2012 of Mindara was N1.76
billion, and the retained earnings and other components of equity were N550
million and N70 million respectively. There had been no new issue of share
capital by Mindara since the date of acquisition and the excess of the fair value of
the net assets is due to an increase in the value of property, plant and equipment
(PPE). The fair value of the non-controlling interest in Mindara was N530 million
on this date. PPE is depreciated on a straight-line basis over seven years.

(iii) Barewa acquired a 10% interest in Calamari, a public limited company on 1 June,
2011 for N80million. The investment was accounted for as an available-for-sale
investment and at 31 May 2012, its value was N90 million. On 1 June, 2012,
Barewa acquired an additional 15% interest in Calamari for N110million and
achieved a significant influence. Calamari made profits after dividends of
N60million and N100million for the years to 31 May, 2012 and 31 May, 2013.

(iv) Finally, on 1 June, 2012, Barewa purchased an equity instrument of 100 million
pesos which was its fair value. The instrument was classified as available-for-
sale. The relevant exchange rate is as follows:

Nto pesos Fair value of instrument- pesos


(million)
31 May 2012 5.1 100
31 May 2013 5.0 90

Barewa has not recorded any change in the value of the instrument since 31 May,
2012. The reduction in fair value as at 31 May, 2013 is deemed to be as a result of
impairment.

(v) The directors have included a loan to a director of Barewa in cash and cash
equivalents of N10million. The loan has no specific repayment date on it but is
repayable on demand. The directors feel that there is no problem with this
accounting entry as there is a choice of accounting policy within International
Financial Reporting Standards (IFRS) and that showing the loan as cash is their
choice of accounting policy as there is no IFRS which states that this policy
cannot be utilised.

(vi) There is no impairment of goodwill arising on the acquisitions.

Required:

Prepare a consolidated statement of financial position as at 31 May, 2013 for the Barewa
Group. (30 Marks)
SECTION B - ATTEMPT ANY TWO OUT OF THREE QUESTIONS IN THIS SECTION (40
Marks)

QUESTION 2
The following transactions relate to Alilerimba Limited:
(i) Alilerimba Limited issued 400,000 convertible bonds on July 1, 2011. The bonds had a 3-
year tenure and were issued with a total fair value of N4million which is also the par
value. Interest is paid annually in arrears at a rate of 16% per annum and bonds, without
the conversion option, attracted an interest of 19% per annum on July 1, 2011. The
company incurred 10% issue costs. If the investor did not convert to shares they would
have been redeemed at par.
At maturity, all the bonds were converted into 1 million ordinary shares of N4 of
Alilerimba, no bonds could be converted before that date.
The directors are uncertain on how the bonds should have been accounted for up to the
date of the conversion on June 30, 2014 and have been told that the impact of the issue
costs is to increase the effective interest rate to 24.00%.
(ii) Alilerimba Limited purchases telephone handsets from a manufacturer for N120,000
each and sells the handsets directly to customers at N90,000 if they purchase credit card
in advance on a prepaid phone. The costs of selling the handsets are estimated at N600
per set. The customers using a prepaid phone pay N12,600 for each credit card at the
purchase date. Credit card expires after 6 months from the date of sale. Cards are
activated when sold, the average unused credit card is N1,800 per card after 6 months.
Alilerimba Limited, also sells handsets to dealers for N50,000 and invoices the dealers for
those handsets. The dealer can return the handset up to a service contract being signed
by a customer. When the customer signs a service contract, the customer receives the
handset free of charge.
Alilerimba Limited allows the dealer a commission of N168,000 on the connection of a
customer and the transaction with the dealer is settled net by a payment of N78,000 by
Alilerimba Limited being the cost of the handset to the dealer (N90,000) deducted from
the commission (N168,000).
The handset cannot be sold separately by the dealer and the service contract lasts for a
12- month period. Dealers do not sell prepaid phones, and Alilerimba Limited receives
monthly revenue from the service contract.
The Chief Operating Officer, a non-accountant has asked for an explanation of the
accounting principles and practices which should be used to account for the above events
on the purchase of handsets and the recognition of revenue from customers and dealers.

(iii) Alilerimba Limited issued 8% preference shares which entitles the holders to receive
cash, with a redemption feature.

Required:

Advise the directors of Alilerimba Limited on:

a. Accounting for the convertible bonds. (12 Marks)


b. T
he principles and practices which should be used for the purchase of handsets and the
recognition of revenue from customers and dealers. (6 Marks)
c. The provisions of IAS 32 on the presentation in financial statements of financial
instruments which entitle the holder to receive cash with a redemption feature.
(2 Marks)
(Total 20 Marks)
QUESTION 3
Real Expansion Plc is a large group that seeks to grow by acquisition. The directors have
identified two potential entities and obtained copies of their financial statements. The
accountant of the company computed key ratios to evaluate the performance of these
companies relating to:
0 Profitability and returns;
0 Efficiency in the use of assets;
0 Corporate leverage;
and 0 Investor based
decision
The computation generated hot arguments among the directors and they decided to engage a
Consultant to give expert advice on which of the companies to acquire.
Extracts from these financial statements are given below:
COMPANY A (N’000) COMPANY B (N’000)
2014 2013 2012 2014 2013 2012
Revenue 77,888,548 64,088,879 59,864,385 14,096,123 12,932,549 12,726,227
Results from operating 9,130,834 5,526,734 4,802,379 1,795,956 1,868,652 2,600,357
activities
Profit before taxation 7,616,444 3,262,719 2,558,644 2,043,293 1,857,089 2,300,357
Profit for the year 6,434,601 2,856,504 1,678,471 1,467,344 1,328,580 1,774,660
Total comprehensive 6,160,014 2,928,875 1,678,471 1,467,344 1,328,580 1,443,990
income for the year
Share capital 320,295 320,295 320,295 249,986 249,986 249,986
Share premium 299,140 299,140 299,140 312,847 312,847 312,847
Retained earnings 16,709,260 11,958,545 9,688,160 10,764,382 9,463,028 8,334,437
Equity 17,328,695 12,577,980 10,307,595 13,753,157 12,455,803 11,327,212
Current liabilities 29,867,824 27,862,495 29,670,126 9,693,313 4,760,565 3,650,513
Non-current liabilities 8,666,690 10,929,695 8,507,941 3,990,011 5,240,199 3,043,865

Total non-current 38,238,065 35,873,744 33,480,167 9,693,742 9,891,975 9,113,908


assets
Total Current assets 17,625,144 15,496,426 15,055,492 17,472,739 12,564,592 8,907,682
Share price at year-end 90 49 42 14.43 10.50 9.50
(N)

Required:
a. As the Consultant to the company, carry out a financial analysis on the financial
statements and advise the company appropriately. (15 Marks)
b. State the major limitations of ratio analysis for performance evaluation.
(5 Marks)
(Total 20 Marks)
QUESTION 4 LIKELY EFFECT LIMITED

Likely Effect Limited has shown a sincere intention to be IFRS compliant. Among a number of
events and transactions, there is the need to change the accounting policies of the company in
trying to comply with a few other standards. As the Consultant of the company, your attention
was drawn to the fact that prior to 2013, the company had capitalised training costs.

According to IAS 38, training cost is regarded as an internally generated intangible asset and
cannot be capitalised. Therefore, there is the need for a change of accounting policy which must
be applied retrospectively.

The training costs capitalised in 2012 was N6m while the total for periods before 2012 was
N12m.

Training costs incurred in 2013 is N4.5m. Retained earnings were N600m and N649m at the
beginning and end of 2012 respectively. The corporate income tax rate is 30% for the relevant
periods. Additional information available is given below:

2013 2012
(N‘M) (N‘M)
Income tax expense 24 21
Profit after tax 56 49
Share capital 50 50
Required:
a. Advise the directors on the implication of the change in accounting standard relating to
treatment of intangible assets and tax effect on the company.
(5 Marks)
b. Prepare statements of profit or loss and other comprehensive income and changes in
equity showing a retrospective application of the change in policy.
(7 Marks)
c. Analyse the effects of the change in accounting policy on periods before 2013.
(8 Marks)
(Total 20 Marks)
SECTION C - ATTEMPT ANY TWO OUT OF THREE QUESTIONS IN THIS SECTION (30
Marks)

QUESTION 5

International Financial Reporting Standards (IFRS) for small and medium size entities
(SMEs) was mandatorily adopted in Nigeria as at January 1, 2014. Entities that do not meet
the IFRS for SME criteria shall report using Small and Medium size Entities Guidelines on
Accounting (SMEGA).

Jossy Limited has total costs excluding land of two-hundred million naira. Being a family
business, the labour force totalled 150 workers with an annual turnover of N18 million. The
management of this company sought your advice to have better understanding of some of the
recognition and measurement principles of SMEs.

Required:
a. Justify the need for IFRS for SMEs financial statements. (6 Marks)
b. Assess the circumstances of Jossy Limited and advise on the principal recognition and
measurement principles that will reduce the company’s reporting burden.
(9 Marks)
(Total 15 Marks)

QUESTION 6

NICE & DICE is a large charity located in Abuja and set up to provide support and assistance
to disadvantaged people in major cities. Most of the charity’s income comes from members of
the public through direct cash collections and regular monthly payments from donors. The
other source of funding comes from government bodies who give grants to support specific
projects that are recognised as being beneficial to the public good.
received and the manner in which it has been spent. The trustees are concerned that this type of analysis does not really
reflect the performance of the charity. They would like to report performance in terms of the work done rather than in
terms of cash inflows and outflows. They want donors to appreciate how efficient the charity is.
The statement of financial position of the charity is a typical one for a large organisation. Nice & Dice owns numerous
properties in Abuja, some of which have been owned for many years. These are shown at historical cost less depreciation.
The trustees does not wish to revalue the properties because this will create the impression that the charity is wealthy
and that it does not require further financial support.

You are required to:

a. Prepare a report to the trustees of Nice & Dice advising them on the reasons why specialised entities are
required to publish detailed information about their activities.
(5 Marks)
b. Analyse the problems of quantifying and reporting the efficiency of not-for-profit
organisations such as Nice & Dice. (5 Marks)
c. Discuss the decision of the trustees to value its properties at cost less deprecation rather
than at fair value. (5 Marks)
(Total 15 Marks)
QUESTION 7 ONDO TELECOMS LIMITED

Ondo Telecoms Limited is one of the biggest telecoms companies in Abuja. One month after the year-end, the Chief
Finance Officer (CFO), while reviewing the company’s activities came up with the following issues for the year ended 30
September, 2014.
a. The Board of Directors is not impressed with the performance of the Home Broadband
operating segment which posted a loss of N1.7billion in 2014 financial year following another loss of N0.8billion
in the 2013 financial year.
b. The carrying amount of the assets in the segment is N4.3billion as at 30 September, 2014
and N4.5billion as at 30 September, 2013. Professional valuers were engaged and they came up with fair value of
N4.2billion as at 30 September, 2013.
c. The Board of Directors made the final decision in June 2014 to sell off the assets in this
segment and concentrate on other business lines. Since the beginning of September, four serious bidders have
been negotiating with Ondo. The board anticipates the sale to be
concluded by end of May 2015 with the transaction cost of N0.3million.

d. On 1 November 2013, Ondo Telecoms Limited acquired a block of flats with an


estimated useful life of 50 years at a total cost of N225million. The blocks of flats are to
be rented out to its employees and engineers at market prices. The decision to acquire
the block of flats was made by the board due to the need to have the engineers close to
the head office to attend to technical issues immediately they arise.
e. Professional valuers were engaged to value the flats as at 30 September, 2014 and a fair
value of N232million was determined.
f. International Telecom Limited which acquired Edo Communications Limited during the
year has just published its results. Edo Communications Limited was a direct
competitor to Ondo Telecom Limited and does similar business. The CFO noted that
International Telecom Ltd. shows an asset of N110million arising from Edo
Communication Limited customer lists’. This made the CFO realise how valuable the
customer details are and has engaged a professional valuer who valued them at
N98million.
g. Over the years, Ondo Telecoms Limited’s main business has been provision of mobile
and fixed landlines services as well as broadband services. In July 2013 Ondo Telecoms
Limited bid for the award of a subscription television licence from government.
h. Ondo Telecoms Limited won the bid and paid N560million for a five-year licence
beginning 1 October, 2013. The licence is transferred and at the time of winning the
bid, the fair value of the licence was estimated at N580million. Due to the slow uptake
of the television business, the licence was revalued at N420million as at 30 September,
2014 by a professional valuer.
Required:
Advise with suitable computations, how the above transactions should be accounted for in the
financial statements of Ondo Telecoms Limited under IFRS for the year ended 30 September,
2014. (Total 15Marks)
SOLUTION 1
BAREWA PLC AND ITS SUBSIDIARIES
CONSOLIDATED STATEMENT OF FINANCIAL POSITION AS AT 31 MAY 2013
WORKINGS NM NM
Assets
Non-current assets
Property, plant and equipment (2,650 + 2,300 +1,610 +400 + 140 -20)
6 7,080
Goodwill 2 168
Investment in Associate 3 225
4 560
Available for sale financial assets (510 + 60+ 50 -60)
8,033

Current Assets
2,630
Inventories (1,350 + 550 + 730) 1,680
Trade receivables (910 + 450 + 320) 10
Loans receivable 2,090 6,410
Cash and cash equivalent (1,020 + 1,000 + 80 - 10)
14.443
Equity and liabilities
Equity attributable to owners of the parent Share 5,200
capital Retained earnings 2,848
4
3 110
Other component of equity (120 - 10) Non- 8,158
1,425
controlling interest 9,583

Non-current liabilities
Long term liabilities (1,200 + 150 + 50) 1,400
Deferred tax (250 + 90 + 30) 370
1,770
Current liabilities
Trade and other liabilities (1,150 + 300 + 600 + 2,170
120) Current tax payable (600 + 80 + 240) 920
3,090 4,860
14.44
3

Workings

(1) Purchase of Megida


Barewa NCI
NM NM NM
3,000
Consideration
Fair value of assets acquired:
Share capital 2,200
Retained earnings 1,360
Other components of equity 40
FV Non-depreciable land (balancing figure) 400
Total fair value of net assets 4.000
Barewa (80% of 4,000)/NCI (20% of 4,000) 3,200 800
-200
Gain on bargain purchase
Gain on bargain purchase is taken to retained earnings going
by IFRS 3

(2) Purchase of Mindara


Barewa NCI
NM NM NM
Consideration Contingent 1,280
consideration Total 120
consideration FV Fair 1,400
value of assets acquired:
Share capital Retained 1,000
earnings Other 550
components of equity 70
1,620
FV PPE (balancing figure) 140
Total fair value of net assets (given) 1,760
Barewa (70% of 1,760)/NCI (30% of 1,760) 1,232 528
Goodwill 168

(3)
Investment in Associate - Cambari NM
st
1 Acquisition 10% 90
nd
2 Acquisition 15% 110
Total acquisition 25% now valued at 200
Add post acquisition retained
profit 25% x 100 25 Dr inv in Ass. Cr P or
To Statement of Financial L 225
Position
There is a movement from N80million to N90million (i.e.
N10million gain):
Dr Inv in Associate Cr Other Component of equity 10
10
On acquisition of further shares the N10 million is realised:

Dr Other component of equity Cr Profit or loss 10


10

(4) Retained earnings


Barewa Megida Mindara NCI
As at 31/05/2013 2,400 1,500 800
At acquisition (1,360) (550)
Gain on bargain purchase 200
Realised gain of inv in associate-other component
of
equity 10
Share of post acquisition retained profit from
Associate 25
Additional dep PPE fair value (20)
Impairment AFSFA (60)
Post acquisition retained profit 2,575 140 230
Barewa Share of post acquisition Prof in Megida
(80% x 140) 112 2
Barewa share of post acquisition Prof in Mindara 8
(70% x 230) 161
6
2.848 9
9
(5) Non-controlling interest 7
Megida Mindara
At acquisition 800 528
Post acquisition 28 69
NCI 828 597
1.425
Check: at consolidation date
20% x 3,740 748
30% x 1,870 561
Fair value Adjustment not yet accounted 80 36
NCI 828 597

Fair value adjustment

NM
Megida - land 400

Mindara - PPE 140

Less Under depreciation 140/7 -20


120

EXAMINER’S REPORT
The question tests candidates understanding of IFRS provisions and principles on
consolidation of group accounts with associate. Candidates are expected to prepare
consolidated statement of Financial Position together with relevant notes and schedules.

Almost all candidates attempted the question but performance was very poor. The commonest
pitfalls were candidates’ inability to compute non-controlling interest, group reserve and
goodwill on consolidation.

Candidates are enjoined to appreciate that while the emphasis in Financial Reporting is on
compliance with regulations enunciated in IFRS and NGAAP, Corporate Reporting emphasises
their competence to evaluate acceptability of alternatives from both IFRS compliance and
reporting perspectives as well as their effects on results, position and risks disclosed by
entities.

QUESTION 2

(i) ALILERIMBA LIMITED

CONVERTIBLE BOND
Some compound instruments have both a liability and an equity component from the
issuer’s perspective.

In this case, IAS 32 (Financial Instruments Presentation) requires that the component
parts be accounted for and presented separately according to their substance based on
the definitions of liabilities and equity.
The split is made at issuance and not revised for subsequent changes in market
interest rates, share prices, or other events that change the likelihood that the
conversion option will be exercised.

A convertible bond contains two components. One is a financial liability, namely the
issuer’s contractual obligation to pay cash in the form of capital, and the other is an
equity instrument, which is the holder’s option to convert into shares.

When the initial carrying amount of a compound financial instrument is required to be


allocated to its equity and liability components, the equity component is assigned the
residual amount after deducting from the fair value of the instrument as a whole, the
amount separately determined for the liability component.

In the case of the bond, the liability element will be determined by discounting the
future stream of cash flow which will be the interest to be paid and the financial capital
balance assuming no conversion. The discount rate used will be 19% which is the
market rate for similar bonds without the conversion right.

The difference between cash received and the liability component is the value of the
option.

Present value of Interest at end of: NM


Year 1 June 2012 (N4m x 16%) - 1.19 537,815
2 June 2013 (N4m x 16%) - 1.192 451,968
3 June 2014 (N4m + (N4m x 16%) - 1.193) 2,753,376
Total liability component 3,743,159
Total equity component 256,841
Proceed of issue 4,000,000

The issue cost will have to be allocated between the liability and equity components in
proportion to the above proceeds.

Liability Equity Total


N N N
Proceeds 3,743,159 256,841 4,000,000
Issue cost 374,315 25,685 400,000
3,368,844 231,156 3,600,00
The credit to equity of N231,156 would not be re-measured. The liability component of
N3,368,844 would be measured at amortised cost using the effective interest rate of
24%, as this spreads the issue costs over the tenure of the bond. The interest payments
will reduce the liability at the year-end.

The initial entries would have been:

Financial Instrument N Interest N


Dr Cash 4,000,000 Cr Cash 400,000
Cr Liability 3,743,159 Dr Liability 374,315
Cr Equity 256,841 Dr Equity 25,685

The liabilitycomponent balance on 30 June 2014 becomes N4m as a result of the


effective interest rate of 24% being applied and cash flows at 16% based on nominal
value.

B/FWD Effective Cash flow C/FWD


Interest
at 24% 16%
N N N N
3,368,844 808,522 640,000 3,537,366
3,537,366 848,967 640,000 3,746,333
3,746,333 899,120 640,000 3,995,453

On conversion of the bond on 30 June 2014. ALILERIMBA Limited would issue 1


million ordinary shares of N4 each and the Original Equity Component together with
the balance on the liability will become the consideration.

N
Share capital 4,000,000
Share premium 231,156
Equity and liability component 4,231,156
(N4,000,000 + N256,841 - N25,685)

(ii) IAS 18 - ‘REVENUE’


IAS 18 requires the recognition of revenue by reference to the stage of completion of
the transaction at the reporting date.
Revenue associated with the provision of services should be recognised as the services are rendered.
ALILERIMBA Limited should record the receipt of N12,600 per credit card as deferred revenue at the
point of sale.
Revenue of N10,800 (i.e. N12,600 - N1,800) should be recognised over the six months period from the date
of sale.
The unused credit card of N1,800 would be recognised when the credit card expires as that is the point at
which the obligation of ALILERIMBA Limited ceases.
Revenue is earned from the provision of services and not from the physical sale of credit cards.
IAS 18 does not deal in detail with agency arrangements, but says the gross inflows of economic benefits
include amounts collected on behalf of the principal and which do not result in increases in equity for the
entity.
The amounts collected on behalf of the principal are not revenue.
Revenue is the amount of ‘commission’.
Additionally, where there are two or more transactions, they should be taken together if the commercial
effect cannot be understood without reference to the series of transactions as a whole.
Consequent upon the above, the company should not recognise revenue when the handset is sold to the
dealer as the dealer is acting as an agent for the sale of the handset and the service contract.
The company has retained the risk of the loss in value of the handset as they can be returned by the dealer
and the price set for the handset is under the control of the company.
The handset sale and the provision of the service would have to be assessed as to their separability.
However, the handset cannot be sold separately and is commercially linked to the provision of the service.
The company would therefore recognise the net payment of N78,000 as a customer acquisition cost which
may qualify as an intangible asset under IAS 38, and the revenue from the service contract will be
recognised as the service is rendered.
The intangible asset would be amortised over the 12 months contract.
The cost of the handset from the manufacturer will be charged as cost of goods sold
(N120,000)
(iii) IAS 32 - (FINANCIAL INSTRUMENTS: PRESENTATION)
IAS 32 - (Instruments: Presentation) Outlines the two options:
• Recognise the financial liability as debt
• Recognise the financial liability as equity
In this case, IAS 32 states that when a financial instrument entitles the holder to
receive cash or contains a redemption feature, such instruments are classified as debts.

This is because the substance, not the legal form, is reflected in the financial
statements.
A contractual obligation exists to deliver cash and this obligation gives rise to a
liability (debt).

When a financial instrument does not entitle the holder to receive cash or does not
contain a redemption feature, you recognise these types of instruments in equity.

EXAMINER’S REPORT
The question tests the requirements, principles and practices of accounting for convertible
bonds, revenue recognition and presentation of financial instruments based on IFRS.
About 20% of the candidates attempted the question and performance was poor. Candidates’
major challenge was their lack of in-depth knowledge of the provisions and applications of IAS
18-Revenue recognition and IAS 32- Financial Instruments (Presentation).

Candidates are advised to thoroughly study and practice questions on relevant provisions of
the International Financial Reporting Standards (IFRS) to enhance their performance in this
paper in future.

SOLUTION 3

(a) Expected ratios and their formulae are:


1. Profitability and returns:
i) Profit margin = [100
1
11 1 —1

1111111
PBIT is Profit before interest and tax
ii) Returns on Capital Employed (ROCE)
------ ---------- 1 [100
111

111111 lSl 1 111 1 11


Capital employed = Total assets less current liabilities

Efficiency in the use of assets


1111111
Asset Turnover ratio =
111111 ILL 1 111 1 11

Corporate
leverage Debt
Equity ratio 1111 111111
11111^11111

Alternatively
11 11 111 1 111 11 1 11 111
1111 1 1111 11 1111111

Investors Based Ratios:

(i) EPS = Earnings Per


1 1 .miffl11 1 1 1 1 1 im 1 1 1 1
Share PAT is Profit after
Taxation
(ii) PE = Price Earnings =1 11111111111

(iii) NAPS = Net Asset Per Share = 111111111 111L1111 L&L1 11L111111
1 1 .11^1111111 ^11111

(iv) EY = Earnings Yield =


1 11 1 11111 11 1

Ratio Computation
2014 Company 2012 2014 Company B 2012
A 2013 2013
Total liabilities (N) 38,534,514 38,792,190 38,178,067 13,683,324 10,000,764 6,694,378
Total assets (N) 55,863,209 51,370,170 48,485,662 17,473,168 17,696,002 14,371,077
Capital employed(N) 25.995.385 23,507.675 18,815,536 17,473,168 17,696,002 14,371,077
No. of ord. share 640,590 640.590 640.590 499.972 499,972 499,972
Profit margin 0.10 0.05 0.04 0.15 0.14 0.18
ROCE 0.29 0.14 0.14 0.12 0.11 0.16
Asset turnover 3 times 2.7 times 3.18 times 0.8 times 0.73 times 0.89 times
Debt/Equity 0.69 0.76 0.79 0.50 0.45 0.37
NAPS 27.05 19.64 16.09 27.51 24.91 22.66
EPS 10.05 4.46 2.62 2.94 2.66 3.55
PE 8.96 10.99 16.03 4.92 3.95 2.68
Comments and advice on the Ratios.

Based on the financial statements of the two companies and the computed ratios, the
following
conclusions are made:
(i) Company A is bigger than Company B.
(ii) Although Company B seems to be more profitable having a higher profit
margin, the
returns on capital employed for Company A is higher.
(iii) Company A is more efficient in the use of assets than Company B.
(iv) The net asset per share for the two companies are very close, but the earnings per share and
price earnings ratios for Company A is higher.
(v) Real Expansion Plc. should acquire company A based on the performance evaluation above.

(b) LIMITATIONS OF RATIO FOR FINANCIAL ANALYSIS

Traditional ratio analysis is becoming outdated because of a number of draw- backs


identified with ratio analysis which include:
(i) Absence of valid universal definition of standard ratios
(ii) Dual meanings and implications of some ratios. This fact calls for the need for
further enquiries or investigations before coming to conclusions when using ratios for
financial analysis. Difference in meaning of certain terms e.g. profit (gross, net, before
taxation or after taxation etc) assets (tangible or intangible), should preference share be
part of debt?
(iii) Incomparable financial statements in the face of different accounting policies, degrees of
diversification and other policies.
(iv) Impact of seasonality on accounting figures may render ratios based on those figures
unreliable for comparative analysis.
(v) Management can easily manipulate accounting figures. Financial ratios are not very useful
when viewed in isolation.
(vi) Accounting figures and valuations are affected by changes in price level resulting
from inflation, hence ratios based on amalgam of currency values are not very
reliable. Also changes in technology and environment affects accounting figures.
(vii) Information in annual report is in summary form and may be inadequate or unsuitable when
detail analysis is needed.
(viii) Accounting information presented in annual report is historic, suffer short-term changes
hence of little value for future decisions or predicting the future.
(i) Basis of information is often out of date, therefore timeliness of information leads to problem of
interpretation.

(ii) Analysis of accounting information identifies symptoms and not causes. EXAMINER’S REPORT
The question tests candidates competence in identifying, computing and interpreting relevant financial ratios for
evaluation, profitability and efficiency in asset utilization, corporate leverage and investment decisions.

Almost all the candidates attempted the question but displayed poor understanding of the requirements of the
question.

Most of the candidates could not identify the relevant ratios while others did not know appropriate formulae.

Candidates are advised to sustain knowledge acquired at the Financial Accounting and Financial Reporting stages
of the examination.
SOLUTION 4

LIKELY EFFECT LIMITED

a. Users of financial statements need to be able to compare financial statements of an


entity over time. This will be in agreement with consistency convention and in order to
identify trends in its financial performance and financial position. Changes in
accounting standards will therefore have significant impact on the choice of accounting
policy adopted by entities and consequently cause major alterations in operating results
in the financial statements.

Changes in accounting policies are rare but are allowed or permitted if and only if:
i. required by statute;
ii. occasioned by a change in standard e.g. a new IFRS replacing an existing one;
iii. the change would result in more reliable and relevant financial information (IAS
8)

Whenever there is a change in accounting policy, the change and its effects on the
financial statements must be disclosed.

Since Likely Effect Limited has shown intention to be IFRS compliant, it is required
that the entity must give appropriate effects to the treatment of its intangible assets.
Therefore, its training costs which were previously capitalized as intangible assets
would be derecognized in compliance with the requirements of IAS 38, because such
cost will not qualify as intangible asset within the context of the standard.

The appropriate treatment is to carry out the derecognition in a retrospective manner.


By retrospective application, it means that the entity should adjust the opening balance
for each item of equity affected by the change for the earliest prior period presented
and the other comparative amounts for each prior period presented as if the new
accounting policy or standard had always been applied.

Arising from the foregoing, the directors of LIKELY EFFECT LIMITED will be
required to adjust the net of tax effect of training cost capitalized for each of 2012 and
2013 with respect to IFRS Statement of Profit or Loss. However, the cumulative effect
of previously capitalized amount of the same cost prior to 2012 will be adjusted from
the retained earnings of the earliest comparative period net of tax (January 1, 2012).

b. (i) LIKELY EFFECT LIMITED


Statement of Comprehensive Income
for the Year ended 31 December, 2013
2013 2012
N’000 N’000
Profit before Income Tax 80,000 64,000
Less: Income Tax 24,000 19,200
Profit for the year 56,000 44,800
Other items of Comprehensive Income - -
Total Comprehensive Income 56,000 44,800

(ii) Statement of Changes in Equity


for the year ended 31 December, 2013

Share Retained Total


Capital Earnings
N’000 N’000 N’000

Balances previously reported as at 2011 50,000 600,000 650,000


Reduction in Retained Earnings resulting from (8,400) (8,400)
change in
Policy 50,000 591,600 641,600
Balances as at 31 December, 2011 (Restated) 44,800 44,800
Comprehensive Income for the Year ended 31
December, 50,000 636,400 686,400
2012 (Restated) 56,000 56,000
Balances as at 31 December, 2012
Comprehensive Income for the Year ended 31 50,000 692,400 742,400
December,
2013
Balance as at 31 December, 2013
Workings:

(i) Profit after income tax + Income tax = Profit before Income tax
=N56,000,000 + N24,000,000
= N80.000.000
N’000
(ii) Profit after income tax in 2012 = 49,000
Income tax = 21,000
Profit before income tax = 70,000
Less training cost capitalized in the period = 6,000
64.000

(iii) Income tax payable should be reduced by the tax element on N6,000,000
overstatement of profit.
Income tax = N21,000,000 - (N6,000,000 x 30%)
=N21,000,000 - 1,800,000
= N19-200-000

(iv) Reduction = Retained Earnings as a result of change in


capitalization of training costs = N12,000,000 x (1 -
30%)
= N 8,400,000
Effects of change in Accounting Policy on previous periods:
c.

Apart from restatement of comparative figures for 2012 on the Statement of


Comprehensive Income, the following items will be affected as analysed
below:
Effect on 2012 figures: N‘000

i. Decrease in profit before tax (the amount of training cost) (6,000)

ii. Decrease in income tax expenses (30% of N6,000) 1,800

iii. Decrease in profit for the period (N6,000 - N1,800) (4,200)

iv. Decrease in comprehensive income (4,200)

v. Decrease in intangible assets (6,000)

vi. Decrease in deferred tax liabilities 1,800

vii. Decrease in retained earnings at the end of the December 2012 (12,600) (649,000
- 636,400)

Effects on Period before 2012 N‘000


(8,400)
Decrease in profit for the period
(Total training cost before 2012 x (1 -
30%) (N12,000,000 x 1 - 30%) = Net effect

ii. Decrease in Comprehensive income (8,400)

iii. Decrease in intangible assets (12,000)

ii. Decrease in Deferred tax liabilities (30% of 12,000) 3,600

Judging from the above, the adoption or compliance with IFRS by LIKELY EFFECT LIMITED
in Year 2013 will affect the entity’s prior periods information as presented under the old
NGAAP or accounting policies.

Some of the likely effects may include the following:

Previous Retained Earnings:


The cumulative effect of the change would have been adjusted on the retained earnings
available as at the beginning of earliest comparative period. As shown in the initial
calculation, the cumulative effect of the change in treatment of training cost on the opening
retained earnings as at 1 January, 2012 net of tax gives rise to a reduction in opening retained
earnings by absolute figure of N8.4m. This reduction has inadvertently reduced the total
equity thereby affecting all relevant ratios relating to equity in the entity. Ratio such as debt
to equity, return on equity, return on capital employed, gearing, etc. previously computed will
be affected by this change.

Comparative period profit for the year:


The effect of the change in the accounting policy will equally have directional effect on the
adjusted profit for the year in respect of 2012 financial year-end. The amount of training
expenses of N6m (N4.2m net of tax) was adjusted to arrive at profit for the year thereby
reducing the initial profit for the year of N49m to N44,8m after the said adjustment for the
effect of the change in accounting policy. This will have an analytical effect of reduction in net
profit margin, return on asset, etc. calculated.

EXAMINER’S REPORT
The question examines the implications of and applicable provisions for changes in accounting standard
relating to intangible assets based on IAS 8-Accounting Policies, Changes in Accounting Estimates and
Errors and IAS 38- Intangible Assets.

Most of the Candidates attempted the question but performance was generally poor. Candidates’ major
challenge was their lack of requisite knowledge of IFRS provisions and their application in the
preparation of Statements of Profit or Loss and Other Comprehensive Income and Changes in Equity.

Candidates are advised to study diligently the provisions and application of various International
Financial Reporting Standards.
SOLUTION 5

a. JOSSY LIMITED
International Financial Reporting Standards (IFRS) for SMEs aim to simplify financial
reporting for SMEs by omitting irrelevant topics, reducing guidance and disclosure
choice. It is designed to facilitate financial reporting by Small and Medium Size
Entities in a number of ways.

The management of Jossy Limited should note the underlisted needs for IFRS for SMEs
financial statements:

i. It provides significantly less guidance than full IFRS;


ii. It is structured according to topics which should make it practical to implement.
iii. There is need for considerable reduction in disclosure requirements;
iv. Guidance not relevant to private entities is excluded;
v. The standard has been written in a clear language that can easily be understood;
vi. Topics not relevant to SMEs are omitted.
vii. Where full IFRS allow accounting policy choices, the IFRS for SMEs allows only
the easier option;
viii. Many of the principles for recognizing and measuring assets, liabilities, income
and expenses in full IFRS are simplified.

b. Principal recognition and measurement principles that will reduceJossy Limited’s


reporting burden as a SME are:

i. IAS 38 - Intangible assets requires internally generated assets to be capitalized


if certain criteria (proven future economic benefits) are met. In reality, it is an
onerous exercise to test these criteria for each type of internally generated asset
and leads to inconsistency with some items being expensed and some
capitalized.

The IFRS for SMEs removes these capitalization criteria and requires all
internally generated research and development expenditureto be expensed
through profit or loss.
ii. IFRS 3 - Business Combinations requires goodwill to be tested annually for
impairment. In reality, it is difficult to ascertain the recoverable amount for
goodwill, instead the assets of the business need to be combined into cash
generating units or even a group of cash generating units in order to determine
any impairment loss. The impairment then needs to be allocated to goodwill and
the other individual assets. This is a complex exercise.
The IFRS for SMEs requires goodwill to be amortised instead. This is a much simpler approach
and the IFRS for SMEs specifies that if an entity is unable to make a reliable estimate of the
useful life, it is presumed to be ten years, simplifying things even further.

iii. IAS 20 - Accounting for Government Grants and disclosure of Government Assistance requires
grants to be recognized only when it is reasonably certain that the entity will comply with the
conditions attached to the grant and the grants will be received. Grants relating to income are
recognized in profit or loss. Grants relating to assets are either netted off the cost of the asset
(reducing depreciation by the amount of the grant over the asset’s useful life) or presented as
deferred income (and released to profit or loss as income over the useful life of the asset).

The IFRS for SMEs simplifies this and specifies that where there are no specified future
performance conditions, the grants should be recognized as income when it is receivable.
Otherwise, it should be recognized as income when the performance conditions are met. This is
more consistent with IASB Framework’s definition of income than the IAS 20 approach.

iv. IAS 23 - Borrowing Costs requires borrowing costs to be capitalized for qualifying assets for the
period of construction. This involves a complex calculation particularly where funds are borrowed
generally as a weighted average rate on loans outstanding has to be calculated in order to
determine the amount of interest to be capitalized.

The IFRS for SMEs requires borrowing costs to be expensed, removing the need for such a complex
calculation.

v. IAS 36 - Impairment of Assets requires annual impairment tests for indefinite life intangibles,
intangibles not yet available for use and goodwill. This is a complex, time-consuming and
expensive test. The IFRS for SMEs only requires impairment test where there are indicators of
impairment.

The full IFRS requires impairment losses to be charged firstly to other comprehensive income for
revalued assets then to profit or loss. The IFRS for SMEs requires all impairment losses to be
recognized in profit or loss, given that tangible and intangible assets cannot be revalued under the
IFRS for SMEs.
May 21, 2015

i. IAS 16 - Property, Plants and Equipment allows assets to be held under the
cost or revaluation model. The IFRS for SMEs does not permit the revaluation
model to be used.

ii. IAS 38 - Intangible Assets allows either the cost model or revaluation model
(where there is an active market). The IFRS for SMEs does not permit the
revaluation model to be used.

Both of these eliminate the use of other comprehensive income, simplifying


financial reporting and the need for costly revaluations.

iii. IAS 40 - Investment Property allows either the cost model or fair value model
(through profit or loss) are permitted. The IFRS for SMEs requires the fair
value model (through profit or loss) to be used as long as fair value can be
measured without undue cost or effort. This promotes consistency in the
treatment of investment properties between SMEs financial statements.

EXAMINER’S REPORT

The question examines the justification for and recognition and measurement
principles of IFRS for SMEs.

It was attempted by most of the candidates but their performance was very poor.

Most of the candidates enumerated the potential benefits of IFRS vs NGAAP instead of
specific need for IFRS for SMEs.

Candidates should learn to correctly identify the requirements of questions before


attempting them.

SOLUTION 6

NICE & DICE


The Board of Trustees
Nice & Dice
Abuja

Dear Sirs,

REPORT ON THE OPERATIONS OF NICE & DICE

Specialised entities, such as Not-for-Profit Organizations (NFPOs) are not set-up to make

profit.
They are exempted from tax. Example of NFPOs include charities, professional organisations,
trade unions, religious, arts, community, research and campaigning bodies.

a. The following are reasons why specialised entities are required to disclose detailed
information about their activities:
i. Specialized entities are subjected to a great deal of legislation and regulations in order
to protect donors from being misled or deceived about the manner in which their gifts are being
applied.

ii. Charities must be adequately controlled to prevent fraudulent or dishonest ones from
raising funds on false pretence.

iii. Donors deserve to be protected having been persuaded to make gifts to pursue good
works.

iv. Charities that raise funds from government must be accountable. Government grants
can be substantial and government always has a responsibility to ensure that its funds are put
to good use.

v. Members of the public must be protected even if their donations are not large.
vi. Charities are exempted from Income tax and must disclose this fact in financial
statements.

vii. All other privileges granted them must be disclosed.

viii. Anomalies in terms of trading activities must be prevented through disclosure.

ix. Reporting and accountability can attract donations

b. Some of the problems of quantifying and reporting the efficiency of Not-for-Profit


Organizations such as Nice & Dice are listed below:
i. The effects of the work done by most charities cannot be quantified in monetary terms.

ii. There is no equivalent of profit which balances income against expenses in order to determine
whether there is a net gain or loss.

iii. Describing the application of funds does not indicate assurance that funds were spent on the
aims of the charity.

iv. Measures of output can be difficult to determine and they can be open to misunderstanding or
even distortion.

v. Efficiency can be difficult to measure because charities often have differing levels of non-
financial inputs.

vi. Employment of professionals, administrators and financial staff may be difficult for some
charities.

c. In evaluating decisions of the trustees to value its properties at cost less depreciation
rather than fair value, we have to consider the underlisted points:
i. Non-current assets must be stated at their fair value in the statement of financial position.

ii. All portfolios of assets must be shown in the statement of financial position at their fair value
and not book values.

iii. Choosing not to attach a fair value to these properties compromises any attempt to measure
the efficiency of an entity.
iv. Fair value shows the efficient use of resources rather than book value.

v. Showing properties at fair value might however create some misleading impression that the
charity does not really need on-going financial support.
vi. The provision of up-to-date information about fair value will also impose costs for the
collection and audit of this information.

vii. Fair values are of relatively little value unless there is a possibility that the assets will be
sold.
viii. Commercial organisations realistically use fair value but charity
organisations are less likely since the properties will be of no use in
themselves without continuing financial support to meet running costs.

EXAMINER’S REPORT

The question tests candidates knowledge of the nature, operations and corporate reporting
responsibilities of Not-for-Profit Organisations based on IFRS provisions.

About 50% of the candidates attempted the question but their performance was poor. They
displayed a poor understanding of the requirements of the question.

Candidates are advised to ensure full coverage of all aspects of the syllabus in their
preparation for the examination.

SOLUTION 7

ONDO TELECOMS LIMITED

Financial Statements (Under IFRS) for the year ended 30 September 2014

i. Non-current assets held for sale (or disposal):


The provision of IFRS 5 should be applied in handling this transaction. According to
this standard, the conditions to be met before an asset could be classified as being held
for sale include:

a. Availability for immediate sale in its present condition subject only to terms that are
usual and customary for sale of such assets. This is evidenced bymanagement
seeking buyers as shown by the bids;

b. The sale must be highly probable; and


c. The sale is to be completed within a year from the date of classification.
Having negotiated with four serious bidders and expecting the sale to be concluded
by May 2015, the above-stated conditions have been met.

S Measurement
o Such asset should be measured at the lower of carrying amount and fair
value less cost to sell.

S Computation
o Carrying amount at 30 September, 2013 - N4.5 million. o Fair value less
cost to sell (N4.2 billion - NO.3million) o The lower of the two (as above)
which is N4.1997 billion will serve as the carrying amount as at 30
September, 2014 o The difference between N4.5 billion and N4.1997 billion
will be expensed during the year ended 30 September, 2014

S Discontinued Operations
o A discontinued operation is a component of an entity that either has been
disposed or classified as held for sale.
o Since it is a whole segment classified as held for sale, it should also be
shown as discontinued operations with the following disclosures:
o Loss for year 2014 is N1.7 billion should be shown separately in the
Statement of Comprehensive Income.
o Prior year amount should also be represented to show the loss of N0.8
billion.

ii. The following computations are relevant:

- The block of flats should be accounted as Property, Plant and Equipment (PPE)
under IAS 16.

The property does not qualify as investment property because it is occupied by


staff even when they are paying market rate (IAS 40). Therefore, the following
recognition and disclosure principles are detailed below:
o The block of flats could be carried:
■ At cost less depreciation; and
■ At revaluation model/basis.

Valuation at cost
-Cost N225m
Less: Depreciation
lw T225 / 11 / x
(N /50 x /12 ) 4.125m
Carrying amount N220.875m

Revaluation model:
Revalued amount N232m

On revaluation, the following gain will appear under comprehensive income:

Revaluation amount N232m


Carrying amount of initial cost
after N220.875m
11 months depreciation
Gain on Revaluation N11.125m

iii. Valuation of Customer Balances (Lists):

Ondo’s Telecon Limited’s customer list of balances is an internally generated intangible


asset that does not qualify for recognition under IAS 38 because the internal costs of
producing these items cannot be distinguished separately from the costs of delivering
and operating the business as a whole.

These items would have qualified for recognition if they were purchased separately as
business acquisition as it happened in the case of International Telecom Limited which
acquired Edo Communications Limited during the period under consideration.

It must be noted therefore that the CFO of Ondo Telecoms Limited would expect to
continue to derive benefits from the customers, it cannot recognize the value in its
financial statements because the entity does not have control over the loyalty of those
customers hence, it remains an internally generated intangible asset until it is
packaged and sold singly or separately to a buyer.

iv. At the end of the first year of its acquisition, due to slow uptake of the television
business, the licence market value has reduced to N420million at 30 September 2014. If
this fair value is synonymous to its recoverable amount as at this date, then it can be
concluded that the asset has suffered an impairment and the impairment should be
duly accounted for (IAS 36). In calculating the impairment, the carrying amount will be
compared with the recoverable amount and the excess of carrying amount over the
recoverable amount will be charged to income statement.

The computation is as follows:


N’m
Cost of Acquisition 560
Amortization at 30/09/2014
(560 / 5 x 1) (112)
Carrying amount 448
Recoverable amount (420)
Impairment loss 28

Subsequently, the carrying amount of the licence will change to N420m and amortised
over the remaining useful life.

EXAMINER’S REPORT

The question tests candidates knowledge of IFRS provisions on classification of NonCurrent


Assets as held for sale, investment property based on IAS 16, 40 and 38.

Very few candidates attempted the question and performance was very poor.

Candidates commonest pitfalls were their lack of knowledge of the bases for classification of
non-current assets held for sale, requirement of investment property, and calculations of
impairment or revaluation gain.

Candidates should prepare and ensure good understanding of salient IFRS issues that may
create reporting alternatives in the preparation of financial statements.
PROFESSIONAL LEVEL - NOVEMBER 2015

CORPORATE REPORTING
Time Allowed: 3 hours
ANSWER FIVE QUESTIONS IN ALL

SECTION A: COMPULSORY QUESTION (30

MARKS)

QUESTION 1

The following financial statements relate to Abia, a public limited company.

Abia Group: Statements of comprehensive income for the year ended April 30, 2015.

Abia Banye Choba


N'm Wm Wm
Revenue 1,620 470 284
Cost of sales (1,372) (274) ( 168)
Gross profit 248 196 116
Other income 62 34 24
Distribution costs (60) ( 4 2) ( 5 2)
Administrative expenses (110) ( 58) ( 2 4)
Finance costs (16) ( 12) ( 16)
Profit before tax 124 118 48
Income tax expense (42) (46) ( 20)
Profit for the year 82 72 28
Other comprehensive income for the year, net of tax: 40 18 12

Available-for-sale financial assets (AFS)


Gains (net) on PPE revaluation 24 12 -
Actuarial losses on defined benefit plan (28) _ - -
Other comprehensive income for the year, net of tax 36 30 12
Total comprehensive income for the year 118 102 40

The following information is relevant to the preparation of the group statement of comprehensive
income:
(i) On May 1, 2013, Abia acquired 70% of the equity interests of Banye, a public limited
company. The purchase consideration comprised cash of N300 million and the fair value of
the identifiable net assets was N320 million at that date. The fair value of the non -controlling
interest in Banye was N108 million on May 1, 2013. Abia wishes to use the 'full goodwill'
method for all acquisitions. The share capital and retained earnings of Banye were N110
million and N170 million respectively and other components of equity were N20 milli on at the
date of acquisition. The excess of the fair value of the identifiable net assets at acquisition is
due to an increase in the value of plant, which is
depreciated on the straight-line method and has a five year remaining life at the date of
acquisition. Goodwill has been impairment tested annually and as at April 30, 2014 had
reduced in value by 15% and at April 30, 2015 had lost a further 5% of its original value. The
goodwill impairment should be allocated between group and NCI on the basis of equity
shareholding.
(ii) Banye acquired 80% of the equity interests of Choba, a public limited company, on May 1,
2013. The purchase consideration was cash of N272 million. Choba's identifiable net assets
were fair valued at N230 million and the NCI of Choba attributable to Abia had a fair value of
N52 million at that date. Goodwill had been impairment tested and no impairment had
occurred. Choba's profits are deemed to accrue evenly over the year.
(iii) Abia has sold inventory to both Banye and Choba in October 2014. The sales price of the
inventory was N20 million and N10 million respectively. Abia sells goods at a gross profit
margin of 20% to group companies and third parties. At the year-end, half of the inventory
sold to Banye remained unsold but the entire inventory sold to Choba had been sold to third
parties.
(iv) Abia sold N10 million worth of goods to a customer who recently made an announcement that
it is restructuring its debts with its suppliers including Abia. It is probable that Abia will not
recover the amounts outstanding. The goods were sold after the announcement was made
although the order was placed prior to the announcement. Abia wishes to make an additional
allowance of N16 million against the total receivable balance at the year end, of which N10
million relates to this sale.
(v) Abia owned a piece of property, plant and equipment (PPE) which cost N24 million and was
purchased on May 1, 2013. It is being depreciated over 10 years on the straight -line basis with
zero residual value. On April 30, 2014, it was revalued to N26 million and on April 30, 2015,
the PPE was revalued to N16 million. The whole of the revaluation loss had been posted to the
statement of comprehensive income and depreciation has been charged for the year. It is
Abia's company policy to make all necessary transfers for excess depreciation following
revaluation.
Required:
a. Compute the goodwill on acquisition of the subsidiaries, showing amount
attributable to the group and the NCI respectively (8 marks)
b. Prepare a consolidated statement of comprehensive income for the year ended
April 30, 2015 for the Abia Group and show all workings. (22 marks)
(Total 30 Marks)
SECTION B: ANSWER TWO OUT OF THREE QUESTIONS IN THIS SECTION (40 MARKS)

QUESTION 2

Megida Plc is in the hospitality industry. The specified ratios and the average figures for the
hospitality industry as at December 31, 2014 are stated below:

Return on capital employed 22.1%


Net assets turnover 1.8 times
Gross profit margin 30%%
Net profit before tax 12.5°%
Current ratio 1.6:1
Quick ratio 0.9:1
Inventory holding period 46 days
Receivables collection period 45 days
Trade payable period 55 days
Debt to equity 40%%
Dividend yield 6%
Dividend cover 3 times
Megida’s statement of profit or loss for the year ended
December 31, 2014 is as
follows:

N'000
Revenue 2,425
Cost of sales (1,870)
Gross profit 555
Other operating expenses (215)
Operating profit 340
Interest payable (34)
Loss on sale of equipment (120)
Profit Before Tax 186
Taxation (90)
Profit for the year 96
Profit for the year 96
Dividends (90)
Retained profit for the year 6
Retained profit brought forward 179
Retained profit c/f 185
Statement of financial position as at December 31, 2014 N'000 N'000

Non-current assets (Note 1) 540


Current Assets:
Inventory 275
Receivables 320
595
Less: Current Liabilities:
Bank overdraft 35
Trade payables 350
Proposed dividend 30
Taxation 85
500
Net Current Assets 95
635
8% Loan Stock (300)
335
Capital and reserves
Ordinary shares (N0.5 each) 150
Profit and loss accounts 185
335
Notes to the accounts
1. Non-current assets
N'000
Cost 3,600
Accumulated depreciation (3,060)
Carrying amount 540
2. The Stock Exchange quotation of Megida shares throughout the year averaged N6 per
50k share.

Required

a. State THREE uses and THREE limitations of ratio analysis. (6 marks)

b. Comment briefly on the company's performance when compared with


the industry average in the areas of liquidity and gearing. (14 marks)
(Total 20 Marks)
QUESTION 3

Mimiko is a major property developer, which buys land for the construction of
housing.
a. Mimiko took out a foreign currency loan of 5 million Diran at a fixed interest
rate of 8% on May 1, 2014. The interest is paid at the end of each year. The
loan will be repaid after two years on April 30, 2016. The interest rate is the
current market rate for similar two-year fixed interest loans.
Mimiko has a financial statement for year ended April 30, 2015 and the
average currency exchange rate for the year is not materially different from
the actual rate.
Exchange rates
N1 = Diran
May 1, 2014
5
April 30, 2015
6
5.6
Average exchange rate for year ended April 30, 2015

Required:
Advice Mimiko on how to account for the loan and interest in the financial statements for the
year ended April 30, 2015. (6 Marks)

b. One aspect of Mimiko's business is to provide low-cost homes in Abuja through the
establishment of a separate entity, known as a housing association due to the high cost of rents
in that city. Mimiko purchases land and transfers ownership to the housing association before
construction starts. Mimiko sells rights to occupy the housing units to members of the public
but the housing association is the legal owner of the building. The housing association enters
into loan agreements with the bank to cover the costs of building the homes. However,
Mimiko negotiates and acts as guarantor for the loan, and bears the risk of increases in the
loan's interest rate above a specified rate.

Currently, the housing rights are normally all sold out on the completion of a project. Mimiko
enters into discussions with a housing contractor regarding the construction of the housing
units but the agreement is between the housing association and the contractor. Mimiko is
responsible for any construction costs in excess of the amount stated in the contract and is
responsible for paying the maintenance costs for any units not sold. Mimiko sets up the board
of the housing association, which comprises one person representing Mimiko and two
independent board members.

Mimiko recognises income for the entire project when the land is transferred to the housing
association. The income recognised is the difference between the total sales price for the
finished housing units and the total estimated
costs for construction of the units. Mimiko argues that the transfer of land represents a sale of
goods which fulfils the revenue recognition criteria in IAS 18 Revenue. (8 marks)

c. Mimiko leased its head office during the current accounting period and the agreement
terminates in six years' time. There is a clause in the operating lease relating to the internal
condition of the property at the termination of the lease. The clause states that the internal
condition of the property should be identical to that at the outset of the lease. Mimiko has
improved the building by adding another floor to part of the building during the current
accounting period. There is also a clause which enables the landlord to recharge Mimiko for
costs relating to the general disrepair of the building at the end of the lease. In addition, the
landlord can recharge any costs of repairing the roof immediately. The landlord intends to
replace part of the roof of the building during the current period. (6 marks)

Required:
Discuss how items b & c above should be dealt with in the financial statements of Mimiko.
(Total 20 Marks)

QUESTION 4
a. The Abuja Municipal Development Authority has mapped out a development strategy for the
development of the Municipal. A certain area was marked "Model Area A" which allows
companies to use the area for two years and dismantle all constructions made on the allo cated
space.

Because of the strategic location of this Model Area, Mafowosere Plc, a company operating in
the office furniture and interior decoration industry, feels that this can help to promote its
merchandise. It therefore pays for all the costs of the "model area" including design, and
construction costs. The company plans to use the area for the allowed period and dismantle all
constructions and installations thereafter.

The costs of dismantling the 'model area' are normally 15% of the original constr uction cost
and the elements of the area are worthless when dismantled. The current accounting practice
adopted by Mofowosere Plc is to charge the full cost of the "model area" against the statement
of profit or loss and other comprehensive income in the year when the area is dismantled. The
accumulated cost of the "model area" shown in the statement of financial position at
December 31, 2014 is N38 million. The company has estimated that the average age of the
"model area" is 10 months at December 31, 2014.

Assume a discount rate of 12%, if necessary. (7 Marks)


b. Mafowosere Plc recently undertook a sales campaign whereby customers can obtain free 3
after sales maintenance service, by presenting a coupon, which has been included in an
advertisement in a national newspaper, on the purchase of the company's furniture. The offer
is valid for a limited time period from September 1, 2014 to December 31, 2014.

The management are unsure as to how to treat this offer in the financial statements for the
year ended December 31, 2014. (5 Marks)

c. The Abuja Municipal Development Authority, in order to encourage the flow of direct foreign
investment into the "model area", decided to subsidise the cost of production of any foreign
company that invests in the "model area". Mafowosere Plc, through its foreign wholly owned
subsidiary claimed the sum of N100 million subsidy.

During an audit exercise, this anomaly was discovered and a fine of N10 million was imposed
on Mafowosere Plc. The company is to repay the N100 million subsidy plus N12 million
interest. The total repayment has been regarded as an intangible asset which has to be
amortised. (3 Marks)

d. Mafowosere Plc gives a standard six months warranty to all its customers. The company has
extended the warranty to one year for certain major customers and has insured against the cost
of the six months extended period of the warranty. The warranty has been extended at nil cost
to the customer.

The claims made under the extended warranty are made in the first instance against
Mafowosere Plc and then Mafowosere Plc in turn makes a counter claim agains t the insurance
company. Past experience has shown that 85% of their furniture will not be subject to
warranty claims in the first six months, 10% will have minor defects and 5% will require
major repair. Mafowosere estimates that in the second six months of the warranty, 15% of the
furniture sold will have minor defects and 10% will require major repair.

In the year to December 31, 2014 the following information is relevant:

Standard Extended Selling Price


Warranty Warranty Per Unit of
(Units) (Units) Warranty (N)

Sales 3000 7000 2000

Major Minor Defect


Repair
N N
Cost of repair (average) 600 150
Assume that sales of furniture are on December 31, 2014 and any warranty claims are made on
December 31 in the year of the claim. Assume a risk adjusted discount rate of 5%. (5 Marks)
Required:
Discuss how items (a & d) above should be dealt with in the financial statements of
Mafowosere Plc for the year ended December 31, 2014 under International Financial
Reporting Standards.
(Total 20 Marks)

SECTION C: ANSWER ANY TWO OUT OF THREE QUESTIONS IN THIS SECTION


(30 MARKS)

QUESTION 5

Global Electricity Plc specialises in the generation and supply of electricity and gas. The directors
have reviewed the Financial Statements and feel that there is very little information about their
corporate environmental governance and human capital management.
The company discloses the following social and environmental information in the financial
statements.

(i) Corporate Environmental Governance


- the highest valuation dosage to a member of the public
- total acid gas emissions and global warming potential
- contribution to clean air through emission savings
(ii) Human Capital Management
- investments in the training and development of staff;
- full commitment to equal opportunities
- the number of employees injured at work in the year
The company wishes to enhance its disclosures based on the current best practices
in these areas.

Required:

a. Advise the Directors of Global Electricity on FIVE areas each of corporate environmental
governance and human capital management to measure and report the company's performance
apart from their disclosures.
(10 Marks)
b. Identify and explain five benefits that companies derive from disclosure of
social and environmental information in their annual reports.
(5 Marks)
(Total 15 Marks)
QUESTION 6

a. At December 31, Year 1, Haruna owned 90% of the shares in Satima. At this date the carrying
amount of the net assets of Satima in the consolidated financial statements of Haruna Group
was N800 million. None of the assets of Satima are re-valued.
On January 1, Year 2, Haruna sold 80% of the equity of Satima for N960 million in cash.
The remaining shares in Satima held by Haruna are estimated to have a fair value of N100
million.

Required:
Explain how the disposal of the shares in Satima should be accounted for in the consolidated
financial statements of Haruna Group. (7 marks)

b. H Plc acquired 90% of the equity shares of S Limited for N120 million. Goodwill on
consolidation was N18 million. There had been no impairment of goodwill since th e date of
acquisition. H Plc sold a 50% holding (leaving it with a 40% holding) for N100 million. This
transaction resulted in H plc losing control of S Limited. The fair value of the residual
investment (i.e. the remaining 40%) was estimated to be N70 million. The carrying value of
the net assets of S Limited at December 31, was N124 million.
Required:

Calculate the gain or loss on disposal. (8 Marks)


(Total 15 Marks)

QUESTION 7

a. The principal aim when developing accounting standards for small and medium-sized
enterprises (SMEs) is to provide a framework that generates relevant, reliable and useful
information which should provide a high quality and understandable set of accounting
standards suitable for SMEs. There is no universally agreed definition of SMEs and it is
difficult for a single definition to capture all the dimensions of a small or medium -sized
business. The main argument for separate SME accounting standards is the undue cost burden
of reporting, which is proportionately heavier for smaller firms.

Required:
Discuss the main differences and modifications to IFRS which the 1ASB made to reduce the
burden of reporting for SMEs, giving specific examples where possible. (8 Marks)
b. Wamako Sokoto has met the definition of an SME in its jurisdiction and wishes
to comply with the IFRS for 'Small and Medium-Sized Entities'. The entity wishes to seek
advice on how it will deal with the following accounting issues in its financial statements for
the year ended November 30, 2014. The entity currently prepares its financial statements in
accordance with full IFRS.
(i) Wamako Sokoto purchased 90% of Muktar Gongora, a SME, on December 1, 2013. The
purchase consideration was N11.4 million and the value of Muktar Gongora's
identifiable assets was N12 million. The value of the non-controlling interest at
December 1, 2013 was estimated at N1.4 million. Wamako Sokoto wishes to use the full
goodwill method, if allowed. The estimated life of goodwill cannot be estimated with
any accuracy. Wamako Sokoto wishes to know how to account for goodwill under the
IFRS for SMEs.
(ii) Wamako Sokoto has incurred N2 million of research expenditure to develop a new
product in the year to November 30, 2014. Additionally, it incurred N1 million of
development expenditure to bring another product to a stage where it is ready to be
marketed and sold.

Required:
Discuss how the above transaction should be dealt with in the financial statements of
Wamako Sokoto with reference to the IFRS for Small and Medium-Sized Entities
(7Marks)
(Total 15 Marks)
SOLUTIONS
SOLUTION 1

(a) Abia Plc

i. Purchase of Banye

Group NCI
Total
N'M N'M N'M N'M
Fair Value of consideration paid by Abia/NCl 300.0 108.0
Fair value of Net asset acquired
Share capital 110.0
Retained earnings 170.0
Other components of equity 20.0
Fair value adjustment plant (balancing figure) 20.0
Fair value of net asset acquired 320.0
Share thereon: Abia 70% and NCI 30% 224.0 96.0
Goodwill 760 120 88.0

ii Purchase of Choba
Group NCl Total
70% 30%
Value of cash consideration 190.4 81.6
Fair value NCl to Abia - ___ - 52.0 52.0
190.4 133.6
Fair value of asset: Choba 230
Share thereon: Abia 56% and NCl 44% (128.8) (101.2)
Goodwill 61.6 32.4 94.0

b. Aba Group
Statement of Comprehensive Income for the year ended 30 April, 2015
N'M
Revenue 2,334.0
Cost of sales (1,786.0)
Gross profit 548.0
Other income 120.0
668.0
Distribution cost (154.0)
Admin expenses ( 203.6)
Impairment of receivable ( 6.0)
Operating profit 304.4
Finance cost (44.0)
Profit before tax 260.4
Tax (108.0)
Profit for the year 152.4

Other comprehensive income


Available for sale financial asset 78.0
Revaluation PPE 39.2
Actuarial loss defined benefit (28.0)
Other comprehensive income for the year 89.2
Total comprehensive income for the year 241.6
Profit for the year attributable to:
Owners of the parent Non-controlling interest (Wk 121.
8a) 0
31.4
Total comprehensive income attributable to: Owners 152.
of the parent Non-controlling interest (Wk 8b) 4
192.4
49.2
241.6
Workings

Consolidation Schedule for statement o Df comprehensive income


Abia Banye Choba CPL
N'm N'm N'm N'm
Revenue 1,620.0 470.0 284.0
Intragroup sales (wk 3) (30.0)
Revenue bankrupt customer (wk 5) (10.0)
Net Revenue 1.580.0 470.0 284.0 2.334.0
Cost of sales (1,372.0) (274.0) (168.0)
Intragroup cost of sales 30.0
Unrealised prof (wk 4) (2.0)
Net cost of sales (1.344.0) (274.0) (168.0) (1.786.0)
Gross profit 236.0 196.0 116.0 548.0
Other income 62.0 34.0 24.0 120.0
Distribution cost (60.0) (42.0) (52.0) (154.0)
Admin expenses (110.0) (58.0) (24.0)
Depreciation (wk 2) (4.0)
Loss on revaluation of PPE (wk 6) (3.2)
Impairment of goodwill (see A part) (4.4)
Total admin expenses (203.6)
Impairment of trade receivable (6.0) (6.0)
Operating profit 118.8 121.6 64.0 304.4
Finance cost (16.0) (12.0) (16.0) (44.0)
Profit before tax 102.8 109.6 48.0 260.4
Tax (42.0) (46.0) (20.0) (108.0)
Profit for the year 60.8 63.6 28.0 152.4
Other comprehensive income Available for
sale financial asset Gain PPE revaluation 40.0 18.0 20.0 78.0
24.0 12.0
Revaluation adjustment (wk 6)
3.2 39.2
Losses defined benefit
Other comprehensive income for the (28.0) (28.0)
year
39.2 30.0 20.0 89.2
Total comprehensive income for the year

100.0 93.6 48.0 241.6

2. Group Structure

Abia

N'M N'M
Abia in Banye is 70% and so NCI is 30%

Abia in Choba is 70% of 80% ie 56%


NCI in Choba direct is 20%
44%
Indirect is 30% of 80% i.e. 24%
100%
Total

3 Depreciation on fair value adjustment of plant N20 m (see (a)


part of the solution) divided by 5 years (This will affect both
2014 and 2015 statement of profit or
loss) 4

4 Intra-group sales/cost of sales 30

5 Unrealised profit on intra-group sales (20% x 20 x 50%) = 2


6 Revenue (bankrupt customer) 10
Allowance for doubtful receivables 6 16
The N10million revenue should not be recog nized in the first instance as it
is not probable that the economic benefit attaching to the sales will flow
to the entity
7 Revaluation of PPE N'M

Cost 24.0
01.05.2013
Depreciation t0 (2.40)
30.04.2014
Carrying amount 21.60
30.04.2014
Revalued to 26.0
Revaluation Reserve OC1 4.40

01.05.2014 Bal b/f 26.00


30.04.2015 Dep 26/9 (No of years remaining
life of PPE) (2.89)
Carrying amount 30.04.2015 3.11
Revalued to (16.00)
Revaluation loss (7.11)

OCI (4.4 -0.49)** 3.91


Balance to profit or loss (3.20)
**Note: Adjustment for excess depreciation
following revaluation(i.e. depreciation
charged before and after revaluation) (2.89-
2.4)) i.e. 0.49

8(a) Profit or loss attributable to NCI:


NCI in profit for the year (30% x 63.6 + 44%
x 28) = 31.40
8(b) NCI in total comprehensive income
30% x 93.6 + 44% x 48) = 49.20
Marking Guide Marks Marks
(a) i. Calculation of goodwill on acquisition of Banye
Fair Value of considerations (Abia & NCI @V 2 Mark each) 1
Fair value of net assets (V 2 mark for each component) 2
Share of fair value of net assets (Abia & NCI @ V 2 Mark each) 1 4

ii. Calculation of goodwill on acquisition of Choba


Value of cash consideration - (Group & NCI @ %% mark each) 1
Fair value of NCI to Abia Fair value of net asset - Choba %
Share of fair value of net asset - (Abia & NCI @V 2 mark each) %
Group/NCI of respective percentages (V 2 Mark each) 1
1
4

(b) Statement of comprehensive income (18


entries @ V 3 mark each) 6

Workings (48 entries @ V 3 mark each) 16


Grand total 30

EXAMINER'S REPORT

The question tests candidates understanding of the computation of goodwill on acquisition of a


direct and indirect subsidiary and the preparation of consolidated statement of comprehensive
income.
Being a compulsory question, all the candidates attempted the question and most of them exhibited
an average understanding of the question some performed relatively well with about 30% of those
who attempted the question scoring 50% and above.
Some of the candidates were unable to compute non-controlling interest and/or identify relevant
components of statement of comprehensive income meant for consolidation. Dealing with non -
controlling interest in the indirect subsidiary as well as the group's policy on valuing non -controlling
interest at fair value was difficult to majority of the students. Adjustment for the revaluation of
Property, Plant and Equipment is another area that requires care especially for the depreciation
adjustment following revaluation. Only few students were able to deal with this adjustment. This has
been well explained in the solution.
Candidates are advised to deepen their understanding of group relationship, the relevant IFRSs
dealing with this as well as the adjustments and treatment of noncontrolling interest. In consolidated
statement of comprehensive income, students are advised to pay attention to calculation of profit for
the year attributable to NCI and the parent.
SOLUTION 2

Megida Plc

(a) i. Usefulness of Ratio Analysis

1. Performance Measurement: It provides indication of a firm's


performance and current financial position. It also measures
performance of managers.
2. Liquidation/Reorganisation Decision: It helps in identifying
whether or not to liquidate or reorganise a business in form of a
business combination, merger, acquisition or internal
reconstruction.
3. Investment/Divestment Decisions: It guides investment decisions as to whether
to further invest or divest where such investments is not performing
satisfactorily.
4. Trend Analysis: It aids in the construction of a pattern of a firm's performance
behavior and financial position.
5. Forecasting: It helps in predicting the firm's future performance.

6. Assets Utilization: It facilitates the assessment of the efficiency with which the
firm is utilizing assets for income generation.
7. Liquidity: It helps to determine the ability of the firm to meet its current
obligations as they fall due.
8. Comparative Analysis: It facilitates inter-firm and intra-firm comparison.
9. Market valuation: Some ratios are useful in determining the value of a
company (e.g. the use of EPS x PE to derive Market Value).

ii. Limitations of Ratio Analysis:

1. Comparison of ratios can be misleading unless they are calculated from financial
statements prepared under uniform accounting policies.
2. Due to impact of inflation, ratios calculated from financial statements prepared
on historic cost basis cannot give a true picture of yearly trends.
3. There are no generally accepted formulas for calculating certain ratios thereby
making it difficult for inter-firm comparison.
4. Only items that can be measured in monetary/quantitative terms are included in
financial statements. For instance, non-financial corporate strengths or
weaknesses are not incorporated.
5. For purposes of overall assessment of the financial strength or weakness of a
firm, the fact that there is no ideal ratio makes the task difficult. For example, a
current ratio of less than 2 could be dangerous for many firms, but quite
acceptable for others.
6. The factors influencing the performance of a firm in one year may change in
another, thus rendering horizontal analysis misleading.
7. The statements of financial position prepared at different points in time are static
in nature and therefore cannot give much information about the pre and post
statement of financial position events.
8. Ratios on their own are meaningless without a basis of making comparison.
9. 1ncidence of creative accounting can give misleading ratios.

10. Some items used in the financial statements are subjective since some are
provisions made based on management judgment. Thus, ratio computation based
on them would also be subjective.

(b) Megida Plc's performance compared with industry average:

i. Liquidity
The company shows real cause of concern because:
• Its current and quick ratios are worse than the industry average and are
far below expected level.
• Current liquidity problems appear to be due to high level of trade
creditors and huge bank overdrafts.
• High level of inventory contributes to the poor quick ratio.
• Receivables collection period is unreasonably long.
• It takes longer days for Megida Plc to settle its payables than industry
average, which might damage relationship with suppliers leading to
curtailment of further credit.

11. Gearing:
Gearing (as measured by debt/equity ratio) is higher than twice
the level of the sectoral/industry average.
Summary
The company's liquidity and gearing position is quite poor and give cause for
concern. If it is to replace its old assets in the near future, it will need to raise
finance. With the current high level of borrowing, this may be a serious problem for
the company. Whilst the gearing level may be at an uncomfortable level, it is
currently beneficial to shareholders. The company is making an overall return of
34.6% based on its return on capital employed, but only paying 8% interest before
tax, on its loan stock. The gearing level may become a serious issue if Megida Plc
becomes unable to meet the finance costs. The company already has an overdraft
and the ability to make further interest payments could be in doubt.

Workings:
Calculation of relevant Unit Industry Megida
ratios Average Plc’s
Return on capital employed (186 + 34)/635 x 100 % 22.1 34.6
(ROCE)
Current ratio (595/500) Ratio 1.6:1 1.19:1
Quick ratio (320/500) Ratio 0.9:1 0.64:1
Inventory holding period (275/1870 x 365) Days 46 54
Debt collection period (320/2425 x 365) Days 45 48
Credit payment period (350/1870 x 365) Days 55 68
Debt/equity ratio (300/335 x 100) % 40 90

Marking Guide Marks Marks


(a) (i) Uses of ratios (Any 3 @ 1 mark each) 3
(ii) Limitations of ratios (any 3 @ 1 mark each) 3
(b) Formula for six ratios at 1 mark each 6
- Computation of ratios (6 at %% mark each)
3
- Comparison with industry average (6 at % mark each) 3
- Implication/benefit of liquidity and gearingin relation
to ROCE 2
Grand total 20
EXAMINER’S REPORT

The question tests candidates’ understanding of the use of ratios to assess the financial state of a
company. As such it requires candidates to be able to identify the relevant ratios to use, knowing the
correct formula, picking the correct figures from the financial statements and computing correctly.
Finally they are expected to make relevant comments using the computed ratios comparing with the
industry average.
It was the most popular question of choice after the compulsory question an d most of those who
attempted the question did very well with about 40% scoring 50% and above. Poor performance by
some candidates can be attributed to their inability to limit themselves to the relevant ratios needed
to make informed comments as required by the questions as some went ahead to compute all ratios
given thereby wasting valuable time that would have been used to address other parts of the
question. There were instances where candidates did not get the formulas for the ratios correct, and
some got the formulas right but picked wrong figures from the financial statement and even where
they got both formulas and figures correct computed wrongly. This may be attributable to
examination pressure. Finally some students who got all the above correct were unable to make
informed comments on liquidity and gearing of the company.
Candidates should familiarise themselves with what is involved in answering questions on ratios as
outlined in this guide for future diets knowing that they should be able to:
• identify the relevant ratios to use,
• know the correct formulae,
• pick the correct figures from the financial statements,
• compute correctly, and
• make informed comments using the computed ratios compared with industry average.
SOLUTION 3

Mimiko

(a) The foreign currency loan will be accounted for as follows:

i) On the date the 5 million Diran loan was obtained, that is May 1, 2014, Mimiko will
have to convert it to naira at the exchange rate ruling on this date which is 5 Diran
to N1. This means 5000000/5 = N1, 000,000. It will be recognized as a liability
and the receipt into bank account.

Dr Bank 1,000,000
Cr Loan - Financial Liability 1,000,000
Being foreign currency loan obtained

ii) lnterest is calculated on due date as 8% x 5,000,000 i .e. 400,000 Diran which in
turn will be converted at the average exchange rate for the year, 5.6 Diran.
(400,000/5.6) which is N71, 429.00. It will be recognized as an expense in the
statement of profit or loss.

Dr Income statement Finance Cost 7 1 ,429


Cr Loan -Finance Cost payable 71,429
Being interest on loan recognized

iii) On payment of interest at year end the 400,000 Diran p aid will be converted at the
rate ruling on that day which is 6 Diran to the N (that is 400,000/6 = N66,667). As
interest had earlier been recognized at N71,429, there will be an exchange gain of
N4,762. Accounting entry in the financial statements will be:

Dr Finance Cost payable 71,429


Cr Bank 66,667
Cr Profit or loss - Exchange gain 4,762
Being payment of interest on foreign loan & exchange gain thereon

iv) At year end, the liability is translated at closing rate which is 6 Diran
to N1 i.e. 5,000000/6 = N833,333. This had already been recognized at
N1,000,000. Therefore there is an exchange gain of N166,667. Accounting entry
will be:

Dr Loan -Financial Liability 166,667


Cr Profit or loss - Exchange gain 166,667

Being gain on restatement of foreign currency loan


(b) The consideration should be whether or not the recognition of income for the entire project
by Mimiko on transfer of land to the Housing Association is in line with the provisions of
IAS 18 Revenue. According to IAS 18, revenue arising from the sale of goods should be
recognised when all of the following conditions have been satisfied (IAS 18.14):
(i) The seller has transferred to the buyer the significant risks and rewards of
ownership;
(ii) The seller retains neither continuing managerial involvement to the
degree usually associated with ownership nor effective control over the goods sold;
(iii) The amount of revenue can be measured reliably;

(iv) It is probable that the economic benefits associated with the transaction will flow to
the seller; and

(v) The costs incurred or to be incurred in respect of the transaction can


be measured reliably.

The key issue is whether Mimiko has transferred the risks and reward of the project to the
housing association and does not retain control usually associated with ownership. The
following help to determine the true position, Mimiko:
• provides a guarantee as regards the maintenance costs,
• is liable for certain increases in the interest rate over expectations,
• is responsible for financing variations in the procurement and construction contract which
the contractor would not cover.
• negotiates and guarantees loan and its repayment for the housing association.
• is exposed to risk as if he had built the housing units himself because he gives guarantees
in respect of the construction process.
• determines the membership of the board of the housing association and thus it is
questionable whether the board is independent of Mimiko or not.
• guarantees that the housing association would not be liable if budgeted construction costs
are exceeded, so Mimiko is exposed to financial risk in the construction process.
On the whole therefore Mimiko can be said to have retained the significant risks and had
effective control of the land transferred and also the entire construction process. Consequently,
the revenue recognition criteria under IAS 18 Revenue are not met on the transfer of the land.
Mimiko should account for the whole project as if he had built the housing units himself.
Accordingly, revenue should be recognised when the housing units are finished and deli vered to
the buyers in accordance with IAS 18. This will be only when the project is completed.

(c) The issues to be addressed are the treatment of:


• Cost to be incurred at the end of the lease to restore the internal condition of the property
to its identical state at the outset of the lease
• Cost incurred to add another floor to the building
• Cost chargeable by the landlord for general disrepair of the building at the
end of the lease
• Cost of immediate repair of the roof rechargeable by the landlord

The cost incurred by Mimiko to add additional floor to the building should be capitalised in
accordance with IAS 16 Property, Plant and Equipment and amortised over the six years of the lease. As
Mimiko has an obligation to restore the internal condition of the property to its identical state at the
outset of the lease, the additional floor creates an obligating event requiring a provision in line with
IAS 37. A provision should be made for the present value of the cost of removal of the floor in six
years' time. At the same time, Mimiko should recognise an asset for the cost of removal. The cost
should be recovered from the benefits generated by the new floor over the remainder of the lease.
The asset should be amortised over the six-year period. In effect, this is in substance a
decommissioning activity.
As regards the disrepair of the building, the estimated costs should be spread over the six years
of the agreement. IAS 37 Provisions, Contingent Liabilities and Contingent Assets would indicate that
Mimiko has a present obligation arising from the lease agreement because the landlord can
recharge the costs of any repair to Mimiko. The obligating event is the wear and tear to the
building which will arise gradually over the tenancy period and its repair can be enforced
through the legal agreement. The wear and tear will result in an outflow of economic benefits
and a reliable estimate of the yearly obligation arising from this will be made.

As regards the roof repair, it is clear from the lease terms that an obligation exists and therefore
a provision should be made for the whole of the rectification work when the need for the repair
is identified.

Marking Guide Marks Marks


(a) Foreign loan/interest conversion/translation and recognition 6
(b) Revenue recognition criteria (Any 4) 4

Key issues on transfer of risks and rewards attaching


to ownership (Any 4) 4 8
(c) Identification of lease terms that results in assets,
provisions, expenses 4
Comment on IAS 16 and IAS 37 2 6
Grand total 20
EXAMINER'S REPORT

The (a) part of the question tests candidates understanding of the treatment of the principal and
interest component of foreign currency loan in line with IAS 21 - Effect of Changes in Foreign
Exchange Rates, while the (b) part examines the practical application of principles around IAS 18 -
Revenue, IAS 16 - Property, Plant and Equipment and IAS 37 - Provisions, Contingent Liabilities
and Contingent Assets and finally the (c ) part tests candidates' understanding of treatment of
Operating Leases where the terms contain provisions that alter the nature of the lease.

Many candidates attempted the question and their performance was just average. The major
weakness observed on the part of candidates is lack of in-depth knowledge of the principles of IFRSs
and their practical application to scenario based transactions. For example many candidates could
not apply the principles of what constitutes sale under IAS 18 and conditions under which income
can be deemed to have been earned on transfer of land to the housing estate. Also many struggled
with understanding the difference between conversion and translation. The most difficult bit
appeared to be the treatment of modification and restoration of an item of operating lease where
again the candidates can apply knowledge of IAS 16 - Property, Plant and Equipment and IAS 37 -
Provisions, Contingent Liabilities and Contingent Assets to resolve the issues raised by the question.

The need for candidates to have a deeper working knowledge of the principles of IFRSs for
Corporate Reporting examinations cannot be overemphasized. Candidates are advised to familiarize
themselves with this aspect of the syllabus as it will continue to be a constant feature in future diets
given the adoption of IFRS in Nigeria.
SOLUTION 4

MAFOWOSERE

(i) The cost of the model area should be accounted for as Property, Plant and Equipment in
accordance with lAS 16 - Property, Plant and Equipment. Property, Plant and Equipment
are tangible assets that are held for use in the production or supply of goods and
services, for rental to others or for administrative purposes, and are expected to be
utilized in more than one accounting period.
The model area meets this definition since it will be in use for more than one accounting
year, and customers will be able to view the furniture items in the area. The costs of the
model area should be depreciated over their expected useful life to their expected
residual value.
MAFOWOSERE Plc, after initial recognition could use the cost model or revaluation
model for the measurement of the model area, lt would, however, be difficult to use the
revaluation model as it would not be possible to measure fair value reliably. Market
based information normally will have to be used.
MAFOWOSERE Plc has an obligation to dismantle the model area after two years. The
company should assess whether it has a present obligation as a result of past event. The
assessment should be carried out in accordance with lAS 37 - Provisions, Contingent
Liabilities and Contingent Assets. ln this instance, it would seem that a provision should
be set up and the amount added to the cost of the asset. The costs of dismantling to be
recognized are an initial estimate of the obligations which arises when Property, Plant
and Equipment is acquired and as a consequence of using the asset.
With the approximate age of the Property, Plant and Equipment of 10 months and the
15% of cost of dismantling, the cost shown in the statement of financial position of
December 31, 2014 should be N42.51m (N38m + 15% of N38m discounted for two
years at 12%). The accumulated depreciation should be N17.71 (N42.51m x 10/24). The
discount N0.45m (N4.51m x 12% x 10/24) should be unwound over the two year period as
a finance cost in the statement of profit or loss and other comprehensive income.
A provision for the dismantling costs will be set up for N4.51 m plus the unwound
discount of N450, 000 totaling N4.561m.

(ii) An obligation should not be recognized for the coupons and no provision created under
lAS 37 (Provision, contingent liabilities and contingent assets). A provision should only
be recognized where there is an obligating event. There has to be a present obligation
(legal or constructive), the probability of an outflow of resources and the ability to
make a reliable estimate of the amount of the obligation. These conditions do not seem
to have been met. Until the company's furniture is purchased, the free 3 after sales
maintenance service cannot be obtained, as this is the point at which the present
obligation arises, the probable outflow of resources occurs and an estimate of the
amount of the obligation can be made.

When the company's furniture is purchased, the service maintenance become s part of the
cost of the sale. The revenue recognized will be the amount received from the customer
(the sales price). The revenue will not be grossed up to include the value of the
maintenance service.

(iii) The payment of the fine constitutes a cost to the company and is not an intangible asset.
An intangible asset is a resource controlled by the company as a result of past events and
from which future economic benefits are expected to flow (IAS 38 - Intangible Assets).
This payment does not meet this definition. The fine should be charged against current
year's profits and disclosed as a separate line item under IAS 1 (Presentation of
Financial Statements, para 97/98).

(iv) The cause of the obligation is the initial sale of this furniture product with the warranty
given at that time. It would be appropriate for the company to make a provision for the
year made up of the 6 monthly warranty of N225,000 and N577,500 (appendix I)
respectively which represents the best estimate of the obligation.

Only if the insurance company has validated the counter claim will MAFOWOSERE Plc
be able to recognize the asset and the income. The company has to be virtually certain. If
it is, then MAFOWOSERE Plc may be able to recognize the asset. Generally, contingent
assets are never recognized, but disclosed where an inflow of economic benefit is
probable.

The company could discount the provision if it was considered that the time value of
money was material. The majority of provisions will reverse in the short -term (within 2
years) and therefore, the effects of discounting are likely to be immaterial. In this
scenario, using risk adjustment rate (IAS 37), the provision would reduce to N219, 510
in the first six months and N563, 410 (Appendix I) in the second six months. The
company will have to determine whether this is material or not.
APPENDIX I
DF DISCOUNTED
EXPECTED
EXPECTED VALUE
1st Six month warranty VALUE
(5%)
N'000
85% x Nil 10% x 10,000 x N150 x 6/12 75 N'000
5% x 10,000 x N600 x 6/12 150
225
(1.025) 219.51

2nd Six month extended warranty

15% of N7,000 x 150 157.5


10% of N7,000 x 600 420.0
577.5 (1.025) 563.41

Marking Guide Marks Marks

(i) Cost of model accounted under IAS 16 1 7


Specification of measurement model after initial
recognition 1
Assessment of obligation arising under IAS 37 1
Cost in SOFP calculation 1
Depreciation 1
Discount as a finance cost in SCI Provision for 1
dismantling 1
(ii) Conditions under which obligation is recognised 1
Relevance of the IAS 37 to the case 2
Treatment of cost of sale/revenue 2 5
(iii) Definition of Intangible asset and relevant IAS 1
Accounting treatment of fine and repayment 2 3

(iv) Calculation of 1st six months warranty 2


Calculation of 2nd six months warranty 2

Explanation 1 5
Grand total 20
EXAMINER’S REPORT

The question specifically requires the candidates to apply the principles of IAS 16 - Property, Plant
and Equipment, IAS 37- Provisions, Contingent Liabilities and Contingent Assets including
warranty and IAS 38 -Intangible Assets. Candidates are expected to demonstrate knowledge of the
standards covering issues of measurement, recognition, capitalisation of dismantling cost,
depreciation and unwinding of interest.

Most of the candidates did not attempt the question and those who did performed below average.
The candidates were unable to relate and apply the relevant IAS to the question.

Candidates are advised not only to study the IFRS, but to deepen their knowledge in the practical
application of its principles in different scenario based situations and cases.
SOLUTION 5

GLOBAL ELECTRICITY PLC

a. From: The Consultant,


Corporate Environmental Governance and Human Capital
Management

To: The Managing Director/Chief Executive Officer,


Global Electricity Plc

Dear Sir,

Advice on Corporate Environmental Governance and Human Capital


Management

Further to the meeting of last week between your company's representatives and our firm, we
hereby highlight additional areas you can consider on the above-stated subject matter.

Corporate Environmental Governance

The areas that further disclosures can be made, among others, are:

(i) Use of energy, emissions and waste disposal


(ii) Environmental remediation expenditure
(iii) Environmental disclosure assurance
(iv) Progress in addressing changes in legal requirements that are not yet
effective.
(v) Investment in local community initiatives
(vi) Accidents affecting the environment, such as, oil spillage, water
pollution and toxic waste
(vii) Transportation of products
(viii) Gas pipeline vandalisation

Human Capital Management

For human capital management, some areas where further disclosures can be made are:

(i) The size and composition of the workforce


(ii) Retention and motivation of employees
(iii) Remuneration and fair employment practices
(iv) Leadership and succession plan
(v) Accidents affecting the community
(vi) Policy for employing physically-challenged persons
(vii) Congruence of employee and organizational goals
(viii) Job rotation for experience acquisition
(ix) Mutual trust and confidence
If you need further clarification on any of these areas, please contact the undersigned.

Yours faithfully,

Managing Consultant

(b) Benefits that companies derive from disclosure of social and environmental information in
annual reports:

(i) Top Management/Board of Directors: top Management needs the disclosure to


respond to press criticisms, answer shareholders' questions and ensure that company
policies are followed.
Board of Directors, because of their growing legal liability, needs to know in some
details what social programmes the company undertakes.
(ii) Environmentalist and other stakeholders:
Management can disclose social and environmental information to satisfy the above
groups.
(iii) Social and environmental disclosures influence some investor's decisions
(iv) Disclosures also differentiate a company from its competitors.
(v) It can confer on the firm, some level of acceptability within the local environment.
(vi) It may indicate the ability to manage key resources, risks and relationship

Marking Guide Marks Marks


(a) Presentation 272
Relevant points on corporate environmental governance
(Any five points at 1V2 marks each) 714 10

(b) Relevant points on human capital management (any 5 @ 1 mark) 5

Grand total 15
EXAMINER'S REPORT

The question examines issues described as "Beyond Financial Reporting" in the syllabus
covering corporate environmental governance and human capital management and disclosure
of social and environmental information.

Most of the candidates attempted the question because they felt they could deal with the issues
raised by the questions. Unfortunately their performance was not too good.

The commonest pitfall was their inability to highlight the relevant issues on corporate
environmental governance and human capital management as well as disclosure of social and
environmental information. Another contributory factor was the failure to present their
recommendations in a report format.

Candidates are advised to properly cover all aspects of the syllabus in their preparation to
enhance better performance in future diets.
SOLUTION 6

HARUNA GROUP

(a) In accounting for this in the Consolidated Financial Statements of Haruna Group, the gain or
loss on disposal will be recognised in group income statement. This is calculated as follows

N'M
Fair value of consideration received (Sales proceeds) 960
Fair value of retained interest 100
1,060
Less: Carrying amount of Satima at the date of disposal (800 x 90%) (720)

Gain on disposal to be credited to profit and loss 340

Explanation:

If a part disposal results in loss of control the parent must recognise a profit or loss on
disposal in the consolidated statement of comprehensive income.

A part disposal which does not result in loss of control is a transaction between the owners of
the subsidiary.

ln this case, the parent company should recognise the profit on disposal in the consolidated
statement of comprehensive income.

(b) The gain on disposal recognised in the statement of comprehensive income


should be calculated as follows:
N'M
Consideration received for shares in Satima Limited 100.0
Fair value of residual investment 70.0
170.0
Net assets de-recognised 124.0
NCl (10% x N124 million) (12.4)
111.6
Goodwill derecognized 18.0
Net assets sold (129.6)
Gain on disposal, reported in profit or loss 40.4
Marking Guide Marks Marks
(a) i) Calculation of Gain or Loss on disposal
(5 entries @ 1 Mark each) 5

ii) Explanation on treatment of profit or loss on disposal (2 points


@ 1 mark each) 2 7

(b) Calculation of Gain or Loss on disposal


(8 ticks @ 1 mark each) 8
Grand total 15

EXAMINER’S REPORT

The question tests candidates’ knowledge of the computation and treatment of profit or loss on
disposal of interests in subsidiaries that results in loss of control.

This question was in fact the example on page 777 of the Study Text.

Almost all the candidates attempted the question and most of them displayed clear
understanding of the requirement with above average performance.
SOLUTION 7

IFRS FOR SMEs

(a) Differences
In deciding on the changes made to IFRS for SMEs, the needs of the users have been
taken into account, as well as the costs and other burdens imposed upon SMEs by IFRS.
Relaxation of some of the measurement and recognition criteria in full IFRS has been
made in order to achieve the reduction in these cost and burdens. Some disclosure
requirements in full IFRS are intended to meet the needs of listed entities, or to assist
users in making forecasts of the future. Users of financial statements of SMEs often do
not need such detailed information.

Small companies have different strategies with survival and stability rather than profit
maximization being their goals. The stewardship function is often absent in small
companies, thus there are a number of accounting practices and disclosures which may
not provide relevant information for the users of SME financial statements. As a result
the standard does not address the following topics:

(i) Earnings per share


(ii) Interim financial reporting
(iii) Segment reporting
(iv) Insurance (because entities that issue insurance contracts are not
eligible to use the standard); and
(v) Assets held for sale

Modifications
In addition there are certain accounting treatments, which are not allowable under the
standard. Examples of these disallowable treatments are the revaluation model for
property, plant and equipment and intangible assets.

Generally there are simpler and more cost effective methods of accounting available to
SMEs than those in full IFRS accounting practices, which have been disallowed.

Additionally, the IFRS for SMEs makes numerous simplifications to the recognition,
measurements and disclosure requirements in full IFRSs.

Examples of these simplifications are:


(i) Goodwill and other indefinite-life intangibles are amortized over their useful
lives, but if useful life cannot be reliably estimated, then the useful life is
presumed to be 10years.
(ii) A simplified calculation is allowed if measurement of defined benefit pension
plan obligation (under the projected unit credit method) involves undue cost or
effort.
(iii) The cost model is permitted for investments in associates and joint
ventures.

As a result of the above-stated differences and modifications, SMEs do not have to


comply with over 90% of the volume of accounting requirements applicable to listed
companies. If an entity opts to use the IFRS for SMEs, it must follow the standard i n its
entirety and it cannot cherry pick between the requirements of the IFRS for SMEs and
those of full IFRSs.

(b) i) Business Combination


IFRS 3 - Business Combinations allows an entity to adopt either the full or partial
goodwill in its consolidated financial statements. However, the IFRS for SMEs only
allows the partial goodwill method. As a result, IFRS for SMEs does not require
SME to determine the fair value of the non-controlling interests not purchased when
undertaking a business combination.

Beside, IFRS 3 - Business Combinations requires goodwill to be tested annually for


impairment, but IFRS for SMEs requires goodwill to be amortised annually within
the period of its useful life. This is a much simpler approach. The standard also
specifies that, when the useful life of the goodwill cannot be reliably estimated, it
should be presumed to be ten years.

Arising from the foregoing, the goodwill of Wamako Sokoto on acquisition of 90%
interest in Muktar Gongora will be as follows:

N'000
Consideration furnished 11,400
Fair value of identifiable net assets acquired
(90% of N12,000) 10,800
Goodwill on acquisition 600

The above-calculated goodwill of N600,000 will be amortised for a period of ten


years at N60,000 per annum.

(ii) Research and Development Expenditure


IFRS for SMEs requires that all internally generated research and development
expenditure shall be expensed through profit or loss in the year in which it
arises. Unlike full IFRS under IAS 38 -
Intangible Assets which requires that expenses in relation to internally generated
research and development should be expensed at research stage and capitalized
at development stage. The development stage requires a specified requirement
to be met before such capitalization can take place. Sometimes, these
requirements are difficult to determine. With the simplified approach adopted
under IFRS for SMEs, it has taken away the burden of determining the
development stage criteria.
As a result of the above, Wamako Sokoto total expenditure on research and
development of N2million and N1million respectively must be written off to
profit or loss for the year, resulting in writeoff of total sum of N3million.

Marking Guide Marks


(a) Differences between Full IFRS and IFRS for SMEs covering Earnings per
share, Interim financial reporting, Segment reporting,
Insurance, Assets held for sales 5

Modifications of Full IFRS to suit SMEs on Goodwill and other intangibles,

Defined Benefit plan and Cost model for investments 3

(b) i) No fair value for goodwill and Computation of goodwill 4

ii) Full IFRS - Research and Development and SMEs R & D 3


Grand total 15

EXAMINER'S REPORT

The question requests candidates to discuss and demonstrate the main differences and
modifications made to full IFRS to reduce the burden of reporting for SMEs.

Only some of the candidates attempted the question and their performance was poor. Majority
of the candidates could neither state the differences nor the modifications.

With the relevance and quantum of the SMEs in Nigeria's business environment, candidates
need to ensure proper understanding of the provisions of IFRS for SMEs.
THE INSTITUTE OF CHARTERED ACCOUNTANTS OF NIGERIA
PROFESSIONAL LEVEL CORPORATE REPORTING

YOU ARE REQUIRED TO ANSWER FIVE OUTOF SEVEN QUESTIONS IN THE PAPER

SECTION A COMPULSORY (30 Marks)

QUESTION 1

Given that accrual accounting tends to mask actual cash flow performance, stock analyst and rating
agencies are generally more interest in cash flow. The directors of Joy-land Plc, have called for the cash
flow statement of the group so as to have a view of earnings performance devoid of accruals. The
following draft group financial statements relate to Joy-land Plc.

Joy-land Plc Group: Statement of financial position as at November 30


2015 2014
N'm N'm

Assets
Non-current assets 1,308 1,016

Property, plant and equipment 32 24

Investment property 192 272

Goodwill 340 288

Intangible assets 216


376 360
Investment in associate
2,461 1,960
Available for-sale financial assets

Current assets
Inventories 420 512
Trade receivables 248 452
Cash and cash equivalents 928 572
1,596 1,536
Total assets 4,060 3,496

Equity and liabilities


Equity attributable to the owners of the parent:
Share capital 1,160 1,100
Retained earnings 1,404 1,296
Other components of equity 60 80
2,624 2,476
Non-controlling interest 220 144
2,844 2,620
Non-current liabilities
268 282
Long term borrowings
140 164
Deferred tax
100 88
Long term provisions - pension
508 536
Total non-current liabilities

Current liabilities
Trade payables 576 220
Current tax payable 132 120
Total current liabilities 708 340
Total liabilities 1,216 876
Total equity and liabilities 4,060 3,496

Joy-land Group: Statement of comprehensive income for the year ended November 30,
2015.
N'm
Revenue 1,728
Cost of sales (1,268)
Gross profit 460
Distribution costs (222)
Administrative expenses (144)
Other income 100
Operating profit 194

2
Finance cost paid (18)
Gains on property 36
Share of profit of associate 24
Profit before tax 236
Income tax expense 44
Profit for the year 192
Other comprehensive income
Items that will not be re-classified to profit or loss
Losses on property revaluation (28)
Actuarial losses on defined benefit plan (24)
Total items that will not be classifies to profit or loss (52)
Items that may be reclassified to profit or loss
Gain on available for sale financial assets (AFS) 8
Other comprehensive income for the year, net of tax (44)
Total comprehensive income for the year 148
Profit attributable to:
Owners of the parent 152
Non-controlling interest 40
192
Total comprehensive income attributable to
N’m
Owners of the parent 108
Non-controlling interest 40
148

Joy-land Group: Statement of changes in equity for the year ended November 30, 2015
Share Retained Total NCI
AFS Revaluation
Earnings
Financial Surplus
Assets PPE
N'm N'm N'm N'm N'm N'm
Balance 1 Dec 2014 1.100 1,296 16 64 2,476 144
Share capital issued 60 60
Dividends (20) (20) (52)

Right issue 8
Acquisition
Total comp. inc for 128 8 (28) 108 40
the year
Balance Nov 30, 2015 1,160 1,404 24 36 2,624 220

The following additional information relates to the financial statements of Joy-land


(i) On December 1 2013, Joy-land acquired 8% of the ordinary shares of Talk- peace. Joy-land had
treated this investment as available for sale in the financial statement to November 30, 2014.
On December 1, 2014. Joy- land acquired a further 52% of the ordinary shares of Talk-peace and
gained control of the company, the consideration for the acquisitions was as follows:

Holding Considerations
N'm
December 1, 2013 8% 16
December 1, 2014 52% 120
60% 136

At December 1, 2014 the fair value of the 8% holding in talk peace held by Joy-land at the time of the
business combination was N20 million and the fair value of the noncontrolling interest in Talk-peace was
N80million. no gain or loss on the 8% holding in Talk-peace had been reported in the financial statement
at December 1, 2014, the purchase consideration at December 1, 2014 comprised cash of N60 million
and share of N60million.

The fair value of identifiable net assets of Talk-peace at the date of acquisition comprised the following:
N'm

Property, plant and equipment 50


Intangible assets 52
Inventories 30
Trade receivables 20
Cash 28

(ii) Goodwill relating to all subsidiaries had been impairment tested in the year to November 30,
2015 and any impairment accounted for. The goodwill impairment related to those subsidiaries
which were 100% owned.
(iii) Joy-land purchase a research project from a third party including certain parents on December
1, 2014 for N32million and recongised it as an intangible asset, which is acceptable under IAS
38. During the year, Joy-land incurred further costs, which included N8million on completing the
research phase. N16million in developing the product for sale ad N4million for the initial
marketing costs. The initial marketing cost has correctly been accounted for. there were no
other additions to intangible assets in the period other than those on the acquisition of Talk-
peace.

(iv) On November 30, 2015, Talk-peace made a rights issue on a 1 for 4 basis. The issue was fully
subscribed and raised N20million in cash.

(v) Joy-land owns an investment property. During the year, part of the air- conditioning system of
the property, which had been a carrying value of
N2million, was replaced by a new system, which cost N4million. Joy-land uses the fair value
model for measuring investment property.

(vi) Joy-land sold off surplus land with a carrying amount of N40 million for cash of N60million and
in addition accepted plant valued N16million as part of the disposal agreement with the buyer.
Necessary accounting entry had already been passed for this, resulting in the gain on property
shown in the income statement above. Depreciation for the year to November 30, 2015 for
property, plant and equipment was N108million.

(vii) Although, Joy-land Plc has a contributory pension in line with Pension Act 2004, it also operates
a defined benefit scheme for few elected top executives and expatriates which figures for the
current year as follows:

N'm
Balance at the beginning, December 1, 2014 88
Charge to profit or loss for the year 16
Pension contributions paid during the year (28)
Actuarial loss to other comprehensive income 24
Balance at the end, November 30 2015 100
(viii)
The associate company did not pay any dividends in the year.

(viii) Deferred tax of N40illion arose on the gains on available for sale investments in the year.

Required
(a) As the CFO of the group, briefly explain to the legal and engineer directors what is meant by
earnings management giving TWO examples of how accruals could be employed in the earning
management. (3 marks)
(b) Determine the goodwill arising on the acquisition of the subsidiary on
December 1, 2014 and total goodwill impairments of the group as at November
30, 2015 statement of cash flow on the assumption that it is the policy of Joy- land Plc to value
Non-controlling interest at full fair value. (3 marks)

(c) Prepare a consolidated statement of cash flows for the Joy-land Group for the year ended
November 30, 2015 using the indirect method under IAS 7 'statement of Cash flow.
Note; Ignore deferred taxation other than where is mention in the question.
(Total 30 Marks)
SECTION B: ANSWER TWO OUT OF THREE QUESTIONS IN THIS SECTION (40 Marks)

QUESTION 2

Ehis Marvel, a public company, is a high street retailer that sells clothing and food. The managing
director is very disappointed with the current year's result. the company expanded its operations and
commissioned a famous designer to restyle its clothing products. This has led to increased sales in both
retail lines, yet overall profits are shown.
Extract from the Income Statement for the two years to March 31 2016 are shown. Income
statements Year to March 31, 2016 Year to March 31, 2015
N'000 N'000 N'000 N'000
Revenue -clothing 16,000 15,600
- food 7,000 23,000 4,000 19,600

Cost of sales- clothing 14,500 12,700


- food 4,750 (19,250) 3,000 (15,700)
Gross Profit 3,750 3,900

Ehis Marvel Plc - Statement of cash flow for the year to March 31, 2016
N'000 N'000
Note: figures in brackets are N'000

Cash flow from operating activities: 700


Net Profit before tax
Adjustment for:
Depreciation - non-current assets 3,800
Loss on disposal of fixtures 1,250
Interest expenses 300 5,350
Operating profit before working capital changes 6,050
Increase in inventory (2,900 - 1,500) (1,400)
Increase in trade receivables (100 - 50) (50)
Increase in trade payables (3,100 - 2,150) 950
Cash generated from operations 5,550
Interest paid (300)
Income tax paid (480)
Net cash from operating activities 4,770
Cash flow from investing activities:
Purchase of property, plant and equipment (10,550)
Disposal cost of fixtures (50) (10,550)
(5,780)
Cash flows from financing activities:
Issue of ordinary shares (2,000 + 1,000) 3,000
Long term loans (3,000 - 1,000) 2,000
Equity dividend paid (600) 4,440
Net decrease in cash and cash equivalents (1,380)
Cash and cash equivalents at beginning of period 450
(930)
Cash and cash equivalents at end of period

The following ratios have been calculated: 2016 2015


Returns on capital employed 9.3% 33.9%
Net assets turnover 2.1 times 3.3 times
Gross profit margin
- clothing 9.4% 18.6%
- food 32.1% 25%
Net profit (after tax) margin 2.0% 7.1%
Current ratio 0.71.1 0.71.1
Inventory holding period
- clothing 68 days 39 days
- food 15 days 17 days
Accounts payable period 59 days 50 days
Gearing 28% 17%
Interest cover 3,3% 25 times

The following information is relevant


(i) The floor areas (in squares metres) occupied were: March 31,
2016 2015
Clothing 48,000 35,000
Food 6,000 5,000
54,000 40,000
(ii) The share price of Ehis Marvel Plc averaged N6.00 during the year to March 31,
2015, but was only N3.00 at March 31, 2016.

Required:
Write a report analysing the financials of Ehis Marvel Plc, utilising the above ratios and the information
in the statement of cash flows for the two years ended March 31,
2016. Your report should refer to the relative performance of the clothing and food sales and be
supported by any further ratios you consider appropriate.
(Total 20 Marks)

QUESTION 3
Limelight, a public limited company, is a major player in commodity brokerage and supplies. The
following transactions relate to the year ended December 31, 2014. Profit before taxation for the year
was N487.5m. Taxable profit for the same period was N131.25m. The balances of non-current assets of
the company, at December 31,
2014.
N'000
Accounting carrying amount 937,500
Tax written down value 637,500

The balances above do not include a freehold building purchase in February 2014 for N750m. This
building was revalued to N985m on December 31, 2014. Accrued rental income on investment property
at December 31, 2014 amounted to N9.75m. This income was credited to statement of profit or loss asat year end bu
until three months after. Rental income is taxed by the FederalInlandRevenue
Service on actual basis when it is received.

No other temporary differences exist at December 31, 2104, income tax and Withholding taxes on
rental income are paid at 30% and 10% respectively six months after the year.

Required:
a) Discuss the conceptual basis for the recognition of deferred taxation by Limelight Plc using the
temporary difference approach in accordance with IAS 12, arising from the above transactions.

b) (i) Outline how the above transactions should be accounted for using
journal entries where appropriate, and
(ii) Calculate the provision for deferred tax after any necessary adjustments to the financial
statements at December 31, 2014, use journal entries
QUESTION 4

a) LALUPON Plc was incorporated on January 3, 2010 in Nigeria with N250m authorized and fully
paid share capital. As part of its initial capital, the company issued a 10% debenture bond. It
also agreed to the appointment of a trust manager who was charged with the responsibility
that the bond in dentures is faithfully kept. The indentures among others provided for:
- Bond amount N100m (2020)
- Yearly payment of interest and principal due
- Crystallization of the whole loan (Principal and interest and all incidental expenses) on
default.
- Discretional waiver of any term of the bond only at the instance of the bond holder.
On January 4, 2013, the Trust manager informed the bond holder of a default tin servicing the loan.
After a meeting of all stakeholders. The bond holder agreed to a waiver postponing the payment
till December, 2014. On June 3,
2014 because of the down turn in business activities LALUPON Plc felt a further waiver was
required. After another round of a meeting, the bond holder consented to a waiver till December
2015, when LALUPON Plc was confident 1 could make the payment. On December 31, 2014,
LALUPON Pl classified the loan as long-term debt in its statement of financial position on the basis
that the loan was not in default at the end of their porting period as the bond holder had issued
waivers an had not sought redemption.

Required:
Discuss. How the above events should be accounted for in the financial statements of
LALUPON (6 Marks)

b) LALUPON Plc owns a piece of land in a residential area. PONJEB ltd has leased the piece
of land from LALUPON Plc using it to store and dispense gas. The Federal government
has announced its intention to enact environmental legislation requiring property
owners to accept liability for environmental pollution. As a result, LALUPON Plc
introduced a hazardous policy and has begun to apply the policy to its properties.

LALUPON Plc has had are port of a gas leakage and subsequent fire out break which
damaged surrounding properties but no life was lost.
LALUPON Plc, has no right of recourse against PONJEBE ltd or its insurance company for
the clean-up and compensations to owners of properties destroyed. At April 30, 2014, it
is virtually certain that draft legislation requiring a clean-up of the land and payment of
compensations to victims will been acted.
Required:

Discuss how the above events should be accounted for in the financial statements of
LALUPON Plc. (6 Marks)

c. On May 1 2011, Yerokun limited granted 500 share appreciation rights (SARs) to its
300 managers. All of the rights vested on April 30, 2013 but they can be exercised
from May 1, 2013 up to April 30, 2015. At the grant date, the value of each SAR was
N10 and it was estimated that 5% of them an agers would leave during the vesting
period. The fair value of the SARs is as follows:

Date Fair Value of SAR


April 30, 2012 N9
April 30, 2013 N11
April 30, 2014 N12

All of the managers who were expected to leave employment did not leave the company as
expected before April 30, 2013. On the April 30, 2015, 60 managers exercised their options
when the intrinsic value of the right was N10.50 and were paid.
Yerokun Limited is confused as to whether to account for SARs under IFRS 2 share-based
payment or IFRS 13 Fair Value measurement and would like to be advised as to how the SARs
should have been accounted for from the grant date to April 30, 2014.

SECTION C: ANSWER ANY TWO OUT OF THE THREE QUESTIOS IN THIS SECTION

QUESTION 5
Umu Amaeshi Plc is conglomerate that that has diverse businesses cutting across some social and
environmental sensitive sectors listed on the Nigeria Stock Exchange. In compliance with financial
reporting regulatory directives of Nigeria, it has adopted IFRS in preparing its financial statements. The
board is aware that this step will enhance the transparency of its reporting and assist in attracting
foreign institutional investors who may be desirous of investing in Nigeria. However, in one of the
company's board meetings the CFO briefed members that given the social and environmental sensitive
nature of its operation, the adoption of IFRS may not be good enough to bring that transparency
relating to its policies and practices relating to social and environmental disclosures. He makes
reference to Para 14 of IAS 1 - presentation of Financial Statement which clearly stated that
“Many entities also present, outside the financial statements, report and statements such as
environmental reports and value added statements, particularly in industries in which
environmental factors are significant and when employees are regarded as an important user
group.Reports and statements presented outside financial statements are outside the scope of
IFRS.."
The board does not want to engage in social and environmental reporting disclosures since many who
do engage in what the business community see as marketing and the report filled with rhetoric. The CFO
has therefore suggested the use of Management commentary.

Required:
a) Briefly explain the purpose of Management Commentary and why it was not
made a mandatory requirement for all companies by IASB. (6 marks)

b) Identify the three most relevant elements of Management Commentary that Umu Amaeshi Plc
should focus on in its management commentary and explain how they will assist the company
to achieve the above objectives given that it does not want to engage in social and
environmental disclosure.
(9 Marks)
(Total 15 marks)

QUESTION 6
Corporations are realising that in this 21st century, firms' intangible assets and human capital are the
most important assets for value creation, production or rendering of services. A recent OECD report in
2006 attests to this and points to an emerging knowledge economy, human capital and intangible assets
lie at the core capabilities and competencies for innovation and business sustainability. There is
therefore the general feeling and perception that traditional corporate reporting do not meet the
capital allocation needs of providers of financial capital. One development has been the emergence of
Integrated Reporting (IR) being promoted by International Integrated Reporting Council (IIRC) supported
by IFAC and most professional accounting bodies globally. The framework issued in 2013 like IASB'S
Conceptual Framework is principles based and as such does not prescribe KPLs but has some guiding
principles and key content elements. Golden path Plc is desirous of employing IR to overcome the
present limitations of its traditional corporate reporting.

a) Write a report to the board of Golden Path Plc:


Advising them on why their financial statements may not meet the capital allocation needs of
providers of financial capital in 21st century firms given the limitations of traditional corporate
reporting which integrated reporting aims to address. (5 marks)
b) Briefly state why integrated reporting may still not resolve the main limitations identified above.
(1 mark)
(Total 15 marks)

QUESTION 7
a) 1BRO Plc provided the remuneration of its management board made up of executive and non-
executive directors (2 foreign nationals inclusive) viz-a-viz:
Annual basic salary
Bonus scheme (Annual compensation)

Four of the directors of 1BRO Plc. Obtained loans from the company at concessional rates while
2 directors are part of the bondholders of the company's loan stock with convertible features to
their advantage.

1n group financial statements with the related parties note under 1AS 24 (related Party
Disclosures). 1BRO Plc disclosure the total remuneration paid to directors and non-executive
directors. No further breakdown of the remuneration was provided. The remuneration of the
non-executive Directors, however, was not included in the key management disclosures.
1BRO Plc was of the opinion that in its jurisdiction, providing information about individual
director's remunerations would be a disservice to them especially, because they have served
the company meritoriously. Consequent upon this the CFO of the company is proposing to
disclose the related party information in the annual financial statements in an ambiguous
manner to prevent users of the financial statements from facing remuneration information back
to specific individual directors.

Discuss the appropriate disclosure for the above transactions within the context of 1AS 24 in the
financial statements of 1BRO Plc, for the year ended December
31, 2014. (7 marks)

b) KOKORO JOB1JOB1 group wishes to expand its operations. As part of this expansion, it has
granted options to employees of its subsidiaries GBANJA and GORO over its own shares as at
March 31, 2015. The awards vest immediately.

KOKORO JOB1JOB1 is not proposing to make a charge to the subsidiary for these options.

KOKORO JOB1JOB1 does not know how to account for this transaction in it own the subsidiaries
and the group financial statements.

Required:
Explain to KOKORO JOB1JOB1 how the above transactions should be dealt with in its own, the
subsidiaries and the group financial statements.
(8 marks)
(Total 15 marks)
SOLUTION 1

(a) Earnings Management

Earnings management can be viewed as attempt by management to influence or manipulate the entity's
reported earnings by using specific accounting methods or policies or changing them to achieve that
objective. In specific terms, earnings management techniques involve deliberate effort by management
to defer or accelerate recognition of expense or revenue transactions designed to influence shortterm
earnings. It could also take the form of using other methods designed to influence short-term earnings.

Aggressive earnings management results in stakeholders being misled to some extent about an entity's
profitability and by extension financial position.

Examples of how accruals could be employed in earnings management are:


• Recognition of invoices issued for goods not yet delivered or when revenue recognition criteria
had not been met which are reversed in subsequent year.
• Reduction in accruals for expenses incurred but not yet paid at end of the reporting period.
• Accruing for a liability that never occurred in the reporting year using provision of IAS 37 -
Provisions, Contingent Assets and Contingent Liabilities which may be reversed in subsequent
accounting year.
• Not accruing for expenses incurred, but not paid for in a given accounting year.
(b) Goodwill on acquisition of subsidiary/ total impairment

Fair value NCI/Existing AFS NCI Joy-land


80 20
Fair value of new consideration paid (60+60) 0 120
Fair value of Net Asset acquired Property, plant and 50 80 140
equipment

Intangible assets 52
Inventories 30
Trade receivables 20
Cash 28
Share of fair value of NA (NCI 40% & Joyland 60% 180 (72) (108)
(180) 8 32

Goodwill 8
\ 40

Total goodwill \
Impairment of Goodwill
Balance at the beginning 272
Purchase of subsidiary 40
312
Balance at the end of the year (192)
Total impairment 120

(c ) Joy-land Plc Group

Cash Flow Statement for the year ended 30 November 2015

Workings N'm N'm


Operating activities
Profit before tax 236
Adjustments to operating activities:
Associate profit (24)
Gain on disposal of property 10 (36)
Finance cost 18
Fair value gain Investment Property 9 (6)
Impairment of goodwill (a) 120
Depreciation on PPE note v 108
Amortisation of intangible assets 1 48
Defined benefit charge to profit or loss note vi 16
Defined benefit scheme paid note vi (28)
Loss on replacement of investment property 2 218
454
component part
Cash flow before working capital changes
Changes in working capital
Decrease in inventory (420-512-30) 122
Decrease in receivables (248-452-20) 224
Increase in trade payables (576-220-0) 356
Tax paid 2 (60)
Interest paid (finance cost as above) (18) 624
Net cash inflow from operating activities 1,078
Investing activities
Purchase of subsidiary 3 (40)
Purchase of associate 4 (192)
Purchase of PPE 5 (402)
Purchase of intangible assets 1 (48)
Additions to investment property (4)
Purchase of AFS financial asset 6 (24)
Proceed sale of property 60
Net Cash flows used by investing activities (650)
Financing activities
Rights issue NCI 8
Dividend to shareholders (20)
Long term loan repayment 7 (16)
Dividend to NCI 8 (44)
Net cash flows used by financing activities (72)
Net increase in cash and cash equivalent for the year 356
Cash and cash equivalent 1 Dec, 2014 572
Cash and cash equivalent 30 Nov, 2015 928

Workings
(1) Intangible
Assets

N’m N'm
Bal b/d 288 Amortisation 48
Bank (32+16) 48 Bal c/d 340
388 388

(2) ____________________________ Tax paid


N'm N'm
Bank- Tax paid (bal.) 60 Bal c/d - coy tax 120
Bal c/d- coy tax 132 Deferred tax 164
deferred tax 140 P or L- coy tax 44
Gain AFS - Deferred Tax 4
332 332
(3) Purchase of Subsidiary

This is based on cash that changed hands (68 - 28) = N40m

4) _______________________ Purchase of Associate


N'm N'm
Bal b/d 0
Share of prof. of 24
associate
Bank (bal. figure) 192 Bal c/d 216
216 216

(5) _____________________ Property, Plant & Equipment


N'm N'm
Bal b/d 1,016 Disposal 40
Talk -Peace 50 Depreciation 108
Disposal - plant 16 Revaluation 28
Bank - Additions
(Balancing) 402 Bal c/d 1,308
1,484 1,484

(6) AFS Financial Assets

N'm N'm
Bal b/d 360 Talk-Piece reclassified 20
Talk-Piece 4
Other AFS 8

Bank (balancing fig.) 24 Bal c/d 376

396 396

(7) Long Term Loan

N'm N'm
Bank (bal. fig.) 16 Bal b/d 284
Bal c/d 268
284 284

(8) Dividend to NC1

N'm N'm
Bank (balancing figure) 44 Bal b/d 144
Rights Issue 8
Acq. Talk-peace (a) 72
Bal c/d 220 Income statement 40
264 264
(9) _________________________ Investment Property
N'm N'm
Bal b/d 24 Disposal of Air- conditioner 2
Cash paid 4

Fair value gain 6 Bal c/d 32

34 34

(10) __________________________Sales of Land


N'm N'm
Land 40 Bank 60
P or L 36 Plant Trade-in 16
76 76

Marking Guide

A Earnings management 3

B Computation of goodwill on acquisition and impairment 3

C Consolidated Statement of Cash Flow and workings 24

30

EXAMINER'S REPORT
The question tests candidates' knowledge of preparation of consolidated statement of cash flows,
along with earnings management and goodwill computation. This would have been predictable, given
that the examiner has tested group statement of financial position and comprehensive income, while
almost all the candidates did well in computing the goodwill, earnings management could not be
explained by the students.

Major pitfalls of most of the candidates were their inability to determine the right figures for
components of the statement of cash flows, correct application of group's policy of valuing non-
controlling interest at full fair value, and explaining Earnings Management.

Candidates need to fully understand the provisions of the relevant IFRSs on consolidated financial
statements and how individual component items of the consolidated cash flow items are determined
but in particular the effect of the assets & liabilities of subsidiary purchased at the beginning of or
during an accounting year. Earnings Management is expected to feature in future examination and
could be a full 15 marks question given its potential to distort reported financial performance and
position.
SOLUTION 2
Date: 17/05/2016

The Managing Director


Ehis Marvel Plc.

Dear Sir,

Subject: The financial performance of Ehis Marvel Plc for the two years ended 31 March 2016

Introduction
Our discussion on the above subject matter refers. The report assesses the overall performance of Ehis
Marvel plc. This is subdivided into operational performance, liquidity/solvency and share price/dividend,
ending with a concluding remark.

Operating Performance

The overall performance of a firm is usually measured by its return on capital employed (ROCE).
Applying this to Ehis Marvel plc, this ratio has seen a significant deterioration from 33.9% in 2015 to a
low 9.3% in 2016. The cause of this deterioration can further be broken down into the component parts
of the ratios making up the ROCE which are asset turnover and profit margins. The asset turnover
(which measures the effectiveness of the firm in utilising its assets to generate income)fell from 3.3
times in 2015 to 2.1 times in 2016. This can be interpreted to mean increased inefficiency in asset
utilisation partly contributing to the deterioration in ROCE. However the company embarked on a huge
investment, acquiring five new stores which must have significantly increased net asset. It may be that
this new investment has not started to generate sales at the level of existing capacity. As such, asset
turnover may not be as bad as it appears and is bound to improve in future. The gross profit margin for
the clothing sales appears to be the major issue with deteriorating ROCE as it dropped by about 98%;
from 18.6% in 2015 to 9.4% in 2016. The effect of this appears more pronounced in terms of its huge
impact on ROCE because clothing sales accounted for about 70% (16,000/23,000 x 100%) of the
company's total sales in the year. In addition there has been a noticeable increase in the inventory
holding period of the clothing products to 68 days in the current year up from 39 days in 2015. This may
be interpreted to mean a deliberate policy on the part of the company to stockpile in order to attract
and meet more sales, but may in the alternative be seen as evidence of obsolete and slow-moving
inventory. One other concern with the clothing sales is its being exposed to changes in fashion. This has
implication on its sales and inventory given that there may be new designs that may fail to gain
acceptance in the
market. On the other hand, profit margin for food sales has witnessed a moderate improvement by
about 28%; increasing from 25% in 2015 to 32.1% in 2016. This has helped to reduce the impact of the
fall in the profit margin of the clothing sales and hence the overall effect on ROCE.

Engaging in a deeper comparative departmental analysis confirms the above position. For example
there has been an increase of 35% in total sales floor area that has resulted in a mere increase in total
sales of 17.3%. A further break down of this on a departmental basis shows that for the clothing
department, an increase in floor area capacity of 37% resulted in a mere increase in sales of 2.6% while
a smaller 20% increase in food sales area produced an impressive 75% sales increase.

Likewise, there has been deterioration in clothing sales per square metre this year to N333,000 down
from N446,000 in 2015 and this is in sharp contrast to the substantial increase in food sales per square
metre of N1,167,000 up from N800,000 in 2015. This resulted in overall total sales per square metre of
N426,000 down from N490,000 in
2015.

Not unexpectedly, the firm's net profit margin fell from 7.1% in 2015 to a mere 2.0% in
2015. This is an indication that overhead expenses increased substantially as a result of the additional
sales outlets and their attendant marketing costs.

Taken together, the insights gained from the above further analysis, point to food sales delivering
increasingly more profitable returns on investment than clothing sales, but the firm has invested more
in clothing floor area than food area. Therefore focusing more on clothing sales for capacity increase
rather than on food sales appears misplaced.

Liquidity/Long term solvency

Overall short term liquidity position as measured by current ratio shows a slight deterioration to 0.71:1
in the current year from 0.77:1 in 2015. Ability to meet short term obligations out of liquid assets is
another measure that could assist in gauging the firm's liquidity position but this may not help much
here as the firm is a retail company. 1t is generally agreed that it is difficult assessing the liquidity ratios
for retail companies as the bulk of their sales are in cash and the normal liquidity benchmark employed
for other companies may not be suitable for them. A far more reliable way to assess their liquidity is
looking at cash flow generated from operation in the cash flow statement. This reveals a healthy figure
of N5.55 million, which comfortably can finance the tax liability and dividend payments of the firm. No
doubt, it has also contributed immensely in funding the expansion acquisition of the five retail stores.
However it can be seen that the expansion has significantly impacted negatively on the bank balance
which fell from N450, 000 bank balance to overdraft of N930, 000. By inference, it may also explain why
the accounts payable period increased marginally from 50 days in 2015 to 55 days in the current year.
Long term solvency is assessed by looking at the company's gearing which has increased from 17% in
2015 to 28% in the current year. This increase is explained by the firm's borrowing of additional long
term loan of N2 million to partly fund the investment in the new stores and refurbishing existing ones,
having issued N3 million in equity to also support this. However, at its current level, the company's
gearing is still considered low, but what appears to be the worrying effect of the increased gearing is a
dramatic fall in interest cover from a very comfortable level of 25 times in
2015 to a very low level of 3.3 times in the current year. This may however be mitigated by the fact
that cash flow from operation can finance the interest amount as pointed out above.

Share price and dividends


The firm's share price lost almost half of its market value falling from N6.00 to N3.00 in the year under
consideration. The loss in share price can partly be explained by the dilutive effect of the firm issuing
two million shares at N1.50 to raise N3 million. This has the potential of reducing the unit market price
to N4.20. However a firm's market share price at any point in time represents market participant's
expectation of its future performance. Thus the other component of this fall in price can be located
within market expectation of Ethis Marvel's performance which in this instance is not better than
previous year. It is surprising that with after tax profit of N450, 000, the firm still went ahead to maintain
its dividends at N600, 000. This cannot be considered sustainable in the long run unless the after tax
profits exceeds N600, 000. It may be that this is a strategic decision aimed at maintaining the market
price of the shares or arresting its downward slide. It may also mean market signalling by management
whereby it aims to communicate a message of optimistic future earnings of the firm. Whichever way, it
does not represent true performance.

Conclusion
The above detailed analysis appears to convey mixed message about the operational performance of
the company. First, in spite of its heavy investment in new and refurbished sales stores, there was
relative reduction in overall operational performance. There is evidence of misplaced focus on clothing
sales expansion instead of food sales, and the overall impact has been an adverse effect on its share
price. On the other hand, it could be that it takes time for the expansion investment to start yielding
expected result given a gestation period and that current performance can be linked to the state of the
present wider economy that may improve in future. It should be noted that the ability of the company
to generate cash from its operating activities remains strong. If it continues at this rate, it has the
tendency to ameliorate the current poor liquidity position of the company.
Yours faithfully
For:

Signed
Chief Financial Officer
Appendix
The following additional Ratios can be calculated:
Clothing Food Overall

Increase in sales area (13,000/35000) 37% (1,000/5,000) 20% 14,000/40,000 35%


Increase in revenue (400/15,600) 26% (3,000/4,000 75% (3,400/19,600) 17.3%

Sales per sqmtr 2016 Sales per sqmtr 2015

N'000 N'000
Overall (23,000/54) (19,600/40)
426 490
Clothing (16,000/48) (15,600/35)
333 446
Food (7,000/6) (4,000/5)
1,167 800

Marking Guide

Commentary on Operating Performance 5

Commentary on Liquidity/Solvency 5

Commentary on Share price and Dividends 5

Summary/overall comments 1

Computation of additional Ratios 3

Presentation in report format 1

20

EXAMINER'S REPORT
The question examines candidates' ability to assess a company's overall operational performance using
financial ratios, breaking it further into departmental analysis. Candidates are expected to critically
evaluate comparative operational performance, liquidity/solvency and share price/dividend of a
company using comparative ratios, extracts from statement of profit or loss and statement of cash
flows. They are also expected to carry out further analysis that assesses the relative performance of the
two departments of the company using any further comparative ratios considered appropriate for this
purpose.
Most candidates attempted the question and performed very well as majority of them scored above
50% of the marks allocated to the question. The poor performance of the few candidates who
attempted the question was due to their inability to make relevant deductions from the ratios and
financial statement information provided. They were therefore unable to undertake informed
comprehensive evaluation of the company's comparative operational performance and financial status.
Most candidates in fact totally neglected to undertake appropriate comparative departmental
performance evaluation which would have increased the depth of their analysis of the company's
performance.

Candidates are advised to improve their analytical mind and understanding of the various aspects of
financial statement interpretations and comparative ratio analyses using 1CAN Study Manual and other
appropriate standard texts as well as practice wide with past 1CAN and equivalent examination
questions.

Q2 Alternative Solution

Operating Performance
The return on capital employed is the overall measure of profitability. The return on capital employed
has suffered a significant decrease from 33.9% to 9.3% due mainly to falling returns from the clothing
sector and the investment in PPE. The increase in PPE attributable to acquisition of five new clothing
stores may not yet have generated returns. The profit for the year margin has dropped from 7.1% to
2.0%. This is mainly as a result of the decrease in the gross margin of the clothing sector but is also
affected by the increase in finance costs and the additional depreciation on PPE acquired during the
year. Though there was a marked increase in the Food department gross profit margin from 25% to
32.5%, the fact that the sales of the Clothing department represents 70% of total sales with a decline of
about 100% in profit margin affected profit margin and assets turnover ratios considerably. A 37%
increase in the floor area of the clothing sector generated only increase in sales of 2.6%, whereas a
modest increase in the food sector floor area generated a significant increase of 75%. This shows that
the clothing department is responsible for the deterioration.

Working capital position:


1t does look as if ETH1S MARVEL PLC'S needs to improve working capital as the company's liquidity
position is below 1 and has deteriorated from 0.77:1 to 0.71:1. The receivables have increased in the
year (from N50,000 to N100,000), though the amounts are characteristically low for Ehis Marvel Plc's,
which is what would be expected for an entity in the retail sector, and it looks like the entity has in turn
withheld payment to payables with an increase of N950,000. Payables are being settled 9 days later
than in 2015. The increase in payables indicates that Ehis Marvel Plc's are using trade payables as a
means of funding working capital. The increase in receivables may be a deliberate attempt to secure
new customers by offering them
favourable credit terms but it is essential that good working capital management is not compromised.

The inventory holding period of clothing has increased significantly from 39 days to 68 days between
2015 and 2016. The increase in inventories has probably arisen in order to meet future expected
demand from the expansion but it may also be an indication that there is some slow moving obsolete
inventory. Given that Ehis Marvel operates in the retail sector, having inventories in stock for another 39
days is likely to be problematic and lead to obsolescence of out of trend items. The fashion industry is
vulnerable to changes too often and two quickly as such Ehis should be cautious with holding too much
inventory. The cash generation of the company is sound with a cash generated from operation of
N5,550,000 the company would be able to service its tax and dividend payments

Long-term stability and solvency:

It is clear from the cash flows from financing that a share issue has been supported by the shareholders
of Ehis Marvel Plc's. A good sign is that EHIS MARVEL PLC'S has managed to fund the acquisition of the
new stores by a modest increase of 11% (from 17% to 28%) in the overall gearing of the business, as
more amounts of equity (N3m) than debt (N2m) have been raised as new finance. It indicates good
stewardship of assets when long term expansion is financed by long term financing. EHIS MARVEL PLC'S
appear to have used a mixture of long term financing, bank overdraft and retained earnings generated
in the year, to fund the expansion. However the fact that the interest cover has fallen from 35 times to
33 times calls for caution as with further falls in profitability may hamper the business from servicing
interest from long term debts. On the other hand the dilution effect attributable to the additional
shares issued and probably the market's expectation of the company's performance have contributed to
the reduction in the share price by half (from N6 to N3).

Shareholders and market position:

The expansion and the dilution in shares are not to the detriment of shareholders as they have still
received, during the year, the previous year's dividend at N600,000 despite an after tax profit of only
N450,000. and it's possible that the new investments in the five stores and PPE will generate greater
returns in the future .In times of expansion, however, a more modest dividend may have negated the
need for long term financing and the interest costs associated with it However, maintaining the dividend
at the current level, the directors may be trying to convey to the market a feeling of confidence in the
future profitability of the company. It should be also noted that although the total dividends have been
maintained, the dividend per share will half due to the share issue during the year. Based on the latest
share price, the share
price of Ehis Marvel has fallen by 100% (from N6 to N3) over the reporting period which indicates that
the stock market has reacted to the decline in profitability.

Conclusion:
From the above analysis there is every indication that operating performance of the entity has declined
considerably despite the huge investment in the food sector and consequently in property, plant and
equipment. However, the cash generation position is quite sound, though the entity needs to worry
about the operating capital cycle particularly at it relates to the inventory days. Ehis Marvels is in the
process of expansion, the current investments in PPE due to the acquisition of the five retail stores may
be able to generate significant increase in sales and returns in future if the company is well managed.

Yours faithfully
Signed
Chief Financial Officer

SOLUTION 3

LIMELIGHT PLC

a. Conceptual basis for the recognition of deferred taxation in accordance with IAS 12:
Deferred tax is the estimated future tax consequences of transactions and events recognized in the
financial statements of the current and previous periods due to temporary differences. Temporary
differences are differences between the carrying amount of an asset or liability in the financial
statements and the value such an asset or liability is stated for tax purpose. Thus temporary differences
arise when income or expenditure is recognised in the financial statements in one year, but is charged
or allowed for tax in another year. It could also arise as a result of differences between the carrying
amount of asset or liability in the statement of financial position and its tax base. Deferred tax should be
recognised only in respect of those items where expense or income is recognised in both accounting and
taxable profit but in different periods.

Examples of temporary differences (not exhaustive) that have deferred tax implications include the
following:

I. Deductible temporary differences:


• Retirement benefit costs which are charged as an expense when incurred but may only
be allowed by the tax authorities when paid;
• Revaluation losses which are recognised in the financial statements when they occur ,
but may only be allowed by the tax authorities when assets are sold;
• Research costs charged as an expense in computing accounting profit, but which may
only be allowed by the tax authorities when paid;
• Unrealised profit on intra-group sales in the consolidated accounts of a group.

II. Taxable temporary differences


• 1nterest income recognised in the financial statements as it is earned ,which is taxed by
the authorities when received;
• Revaluation gains;
• 1nterest capitalised in producing or constructing an asset. This interest is normally
allowed by the tax authorities when it is incurred, but is charged against accounting
profit as the related asset is depreciated or armotised.

III. Temporary differences which can be either taxable or deductable


• Accelerated capital allowances, where the carrying amount of an asset differs from its
tax WDV. 1n this question, the carrying amount of the noncurrent assets is substantially
different from the tax WDV.
• Current assets (e.g. investments) restated to market value where the change in value is
recorded in accounting profit, but is included in taxable profit only on realisation.

The following measurement and recognition criteria are provided for deferred tax
under 1AS 12:

(i) Deferred tax assets and liabilities should be measured using tax rates which are expected to
apply when the asset is realised or the liability is settled, based on tax rates or laws that have
been enacted or substantially enacted by the end of the reporting period.

(ii) A deferred tax asset must be recognised for all deductible temporary differences to the extent
that taxable profit will be available against which the deductible temporary difference can be
utilized, unless the deferred tax asset arises from the initial recognition of an asset or liability in
a transaction that:

- is not a business combination, and


- at the time of the transaction, affects neither accounting profit nor taxable
profit (tax loss)

(iii) 1AS 12 does not permit deferred tax assets and liabilities to be discounted;
(iv) A deferred tax asset should be recognised for deductible temporary differences, unused tax
credits; but only to the extent that it is probable that taxable profit will be available against
which they can be offset.

(v) The carrying amount of deferred tax assets should be reviewed at the end of each reporting
period. A previously unrecognised deferred tax asset should be recognised to the extent that it
has become probable that future taxable profit will allow the deferred tax asset to be
recovered.

(vi) Deferred tax assets and liabilities should be offset only if:

- The entity has a legally enforceable right to set off current tax assets against current tax
liabilities; and
- The deferred tax assets and liabilities relate to income taxes levied by the same tax
authority.

b. (1) Accounting for the transactions.

Profit before tax

Total income tax charge including deferred tax for year will be deducted from Profit before tax

(N487.5m) to arrive at accounting period for the year.

Current tax liability

Current tax liability is obtained by applying the applicable tax rate on the taxable profit. The current tax
liability for Limelight for the year ended December 31, 2014 will be N39.39m (30% of N131.25m) which
will be accounted for as follows:

Dr Cr
N'000 N'000
Current Tax Charge - profit or loss 39,375
Current Tax Liability - SOFP 39,375

Being current tax on profits of the year ended Dec 31, 2014

Non-current Assets

Provision will be made for deferred tax on the temporary difference between the accounting carrying
amount (N937.5m) and tax written down value (N637.5m) of noncurrent assets. The related deferred
tax computation will be N(937.5 - 637.5) x 30%, which is N90m.
The Freehold building purchased in February 2014 for N750m and revaluation surplus of N235m N(985-
750)m will have to be recognised in the financial statements along with related deferred tax N70.5m
N(235 x 30%) on the revaluation surplus. The revaluation surplus and the related deferred tax will be
recognised directly in equity as other comprehensive income not in income statement as follows:

Dr Cr

N'000 N'000
Freehold 750,000
building
Bank 750,000
Being purchase of building in February 2014

Freehold building Revaluation 235,000


Surplus - OC1 Deferred tax - 164,500
OC1 70,500

Being revaluation surplus of building and related Deferred tax on 31

December, 2014

Marking Guide
a Definition of deferred tax and temporary differences
2
Explanation of examples of temporary differences (any 2 @ 1 mark each)
2
Identification of measurement and recognition criteria for deferred
tax under 1AS 12 (any 4 @ 1/2 marks each) 6

10

b.i Explanation of necessary accounting adjustments (any 4 @ 1/2 mark each) 2


Journal entries (6 @ y2 each) 3

b.ii Computation of deferred tax and journals (any 10 entries @ y2 each) 5 10

20

EXAMINER'S REPORT
The question tests the principles of the provisions of 1AS 12 - Income Taxes on deferred tax, requiring
the identification of transactions resulting in temporary differences; measurement, recognition and
accounting for deferred tax.

28
Most of the candidates attempted the question and performance was just above average. Most of those
who attempted the question failed to identify the measurement and recognition criteria for deferred
tax under IAS 12, thus losing valuable marks. They also displayed inability to correctly determine the
accounting treatment and compute the related deferred tax of the specified transactions as required in
part b. of the question.

It is very important for candidates to thoroughly understand the provisions and practical applications of
the various IFRSs covered in the syllabus.

SOLUTION 4

(a) LALUPON Plc.

The loan should have been classified as short-term debt. According to IAS 1, Presentation of financial
statements, a liability should be classified as current if it is due to be settled within 12 months after the
date of the statement of financial position. If an issuer breaches an undertaking under a long-term loan
agreement on or before the date of the statement of financial position, such that the debt becomes
payable on demand, the loan is classified as current even if the lender agrees, after the statement of
financial position date, not to demand payment as a consequence of the breach.

It follows that a liability should also be classified as current if a waiver is issued before the date of the
statement of financial position, but does not give the entity a period of grace ending at least 12 months
after the date of the statement of financial position. The default on the interest payment in January
represented a default that could have led to a claim from the bondholders to repay the whole of the
loan immediately, inclusive of incurred interest and expenses. As a further waiver was issued after the
date of the statement of financial position, and only postponed payment for a short period, LALUPON
PLC did not have an unconditional right to defer the payment for at least 12 months after the date of
the statement of financial position as required by the standard in order to be classified as long-term
debt.

Since the whole loan is repayable or crystallises upon a default, it means the annual 10% interest
charges of N10m should be recognised as finance cost for years ended December 31, 2013 and
December 31, 2014; accumulated liability (principal plus interest) of N110m and N120m should be
recognised as current liabilities.

LALUPON PLC. should also consider the impact that a recall of the borrowing would have on the going
concern status. If the going concern status is questionable, LALUPON PLC would need to provide
additional disclosure surrounding the uncertainty
and the possible outcomes if waivers are not renewed. 1f LALUPON PLC ceases to be a going concern
then the financial statements would need to be prepared on a break-up basis.

(b) LALUPON PLC

A provision shall be recognised under 1AS 37 Provisions, Contingent Liabilities and Contingent Assets
when there is a present obligation (legal or constructive) as a result of a past event; it is probable that
an outflow of resources embodying economic benefits or service potential will be required to settle the
obligation, and a reliable estimate can be made of the amount of the obligation. 1f the above conditions
are not met, no provision shall be recognised. 1n this case, the obligating event is the gas leakage and
subsequent outbreak of fire which destroyed surrounding properties because of the virtual certainty of
legislation requiring the clean-up and restoration. Additionally, there is probably going to be an outflow
of resources embodying economic benefits, because LALUPON has no recourse against the entity or its
insurance company. Therefore a provision is recognised for the best judgement or estimate of the costs
of rectification of environmental damage or restoration work. As LALUPON has no recourse against
PONJEBE, recovery of the costs of clean-up is not likely and hence no corresponding receivable should
be recorded. According to 1AS 37, the amount provided should be recognised as an expense in the
income statement and as a liability in the profit or loss and as a liability in the statement of financial
position.

(c) YEROKUN LTD

1FRS 13 applies when another 1FRS requires or permits fair value measurements or disclosures about
fair value measurements (and measurements, such as fair value less costs to sell, based on fair value or
disclosures about those measurements). 1FRS 13 specifically excludes transactions covered by certain
other standards including share- based payment transactions within the scope of 1FRS 2 Share-based
Payment and leasing transactions within the scope of 1AS 17 Leases.

Thus share-based payment transactions are scoped out of 1FRS 13.

For cash settled share-based payment transactions, the fair value of the liability is measured in
accordance with 1FRS 2 initially, at each reporting date and at the date of settlement using an option
pricing model. The measurement reflects all conditions and outcomes on a weighted average basis,
unlike equity settled transactions. Any changes in fair value are recognised in profit or loss in the period.
Therefore, the SARs would be accounted for as follows:
The liability has reduced by N217,500 and therefore the expense is the difference of
N97,500.

The fair value of the liability would be N1,350,000 at 30 April 2014 and the expense
for the year would be N97,500.

Tutorial note:

SARs exercised:

30 April2014.: 60xN10 50x 500 = N315,000


30 April2013: 60xN11 x500 = N330,000
Therefore a gain of N 15,000 is made on these SARs..

Unexercised SARs:

30 April2014: 225x N12x 500 = N1,350,000


30 April2013: 225x N11 x 500 = N1,237,500

Therefore a loss of N112,500 is made on the remaining unexercised SARs..

This results in an overall charge to profit or loss for the year ended 30 April 2014 of
N97,500.
Marking Guide
A Explanation of the relevance and provisions of IAS 1, Presentation
of financial statements 2

Classification of loan as short-term debt 2

Identification of need for going concern assessment 6

B Requirements of IAS 37 Provisions, Contingent Liabilities


and Contingent Assets regarding provisions 3

Identification of:
1
- Obligating event
1
- Probable outflow of resources
1 6
- Non-recourse to insurance company

C Determination of applicable IFRS 2


Computation of operating expenses and liabilities for the three years (6
entries @1mark each) 6 8

20

EXAMINER'S REPORT

This question tests candidates knowledge and ability to apply the provisions of IAS 1 - Presentation of
Financial Statements on classification of long-term loans; IAS 37 - Provision, Contingent Liabilities and
Contingent Assets on recognition of provisions; and IFRS 2 - Share-based Payment vs. IFRS 13 - Fair value
Measurement on share appreciation rights (SARs).

Most of the candidates did not attempt this question and the performance of those who attempted it
was poor. Most of those who attempted the question displayed a poor understanding of the relevant
provisions of the IFRSs. The candidates were especially unable to determine the appropriate provisions
for the treatment of SARs and were further unable to compute the charge/liability for each reporting
period/date in accounting for the SARs.

Candidates need to work harder to possess a deeper working knowledge of the provisions and
applications of IFRSs for good performance in Corporate Reporting examination.
EXAMINER'S REPORT
(a) 1) PURPOSE OF MANAGEMENTCOMMENTARY
Management commentary is a narrative report that provides a context within which to
interpret the financial position, financial performance and cash flows of an entity. It also
provides management with an opportunity to explain its objectives and its strategies for
achieving those objectives. Users routinely use the type of information provided in
management commentary to help them evaluate an entity's prospects and its general
risks, as well as the success of management's strategies for achieving its stated
objectives. Thus:

Management commentary should provide users of financial statements with integrated


information that provides a context for the related financial statements. Such
information explains management's view not only about what has happened, including
both positive and negative circumstances, but also why it has happened and what the
implications are for the entity's future.

Management commentary complements and supplements the financial statements by


communicating integrated information about the entity's resources and the claims
against the entity and its resources, and the transactions and other events that change
them.

Management commentary should also explain the main trends and factors that are
likely to affect the entity's future performance, position and progress. Consequently,
management commentary looks not only at the present, but also at the past and the
future.

ii) Management commentary was not made a mandatory requirement for all companies
by 1ASB as it is not an IFRS. This implies that entities applying IFRS are not required to
comply with the practice statement, unless specifically required by their jurisdiction.
This is because:
- Laws regulating business are local, that is, business laws vary from country to
country;
- Actual business management practice regulation like corporate governance
varies between countries;
- Nature of businesses vary such that social and environmental concern are not issues in
some businesses and therefore not material with social and environmental issues
- Size of business may be a factor. SMEs are unlikely to be concerned, while in unlisted
companies it can be voluntary. It may be mandatory for listed companies. In Nigeria, it
is not mandatory whereas it is, in EU.

(b) Elements of Management Commentary


Although the particular focus of management commentary will depend on the facts and
circumstances of the entity, management commentary should include information that is essential
to an understanding of:
i) The nature of the business;
ii) Management's objectives and its strategies for meeting those objectives
iii) The entity's most significant resources, risks and relationships;
iv) The results of operations and prospects; and
v) The critical performance measures and indicators that management uses to
evaluate the entity's performance against stated objectives

i) Nature of the business


Management should provide a description of the business to help users of the financial reports
to gain an understanding of the entity and of the external environment in which it operates. The
information serves as a starting point for assessing and understanding an entity's performance,
strategic options and prospects. Depending on the nature of the business, management
commentary may include an integrated discussion of the following types of information:

• Whether social and environmental issues are material to the operational activities of
the industry it operates example oil & gas industry, chemical etc.

• The legal, regulatory and economic impact of social and environmental concern as they
affect the business.

ii) Management objectives


To this end, management uses the medium to disclose information about how it intends to
manage the risks and exploit the opportunities inherent in social and environmental concerns
arising from its operation. Such information could also include success measurement criteria
inform of KPLs.

iii) Entity's most significant resources, risks and relationships


Management commentary should include a clear description of the most important resources,
risks and relationships that management believes can affect the entity's value and how those
resources risks and relationships are managed. To this extent management commentary could
be used to show how
social and environmental resources, risks and relationships affect the entity's value and are
managed.

iv) Results of operations and prospects


Management commentary should include a clear description of the entity's financial and non-
financial performance, the extent to which that performance may be indicative of future
performance and management's assessment of the entity's prospects. Non-financial
performance here could relate to social and environmental impact or its operation.

v) Critical performance measures and indicators


Performance measures are quantified measurements that reflect the critical success factors of
an entity. 1ndicators can be narrative evidence describing how the business is managed or
quantified measures that provide indirect evidence of performance. Management should
disclose performance measures and indicators (both financial and non-financial) that are used
by management to assess progress against its states objectives. Again non-financial
performance measures could be framed as social and environmental performance measures.

Thus management commentary could be used in such a way as to serve the purpose of social
and environmental reporting.

Marking Guide

a Purpose of management commentary and why not mandatory


(4 points @ 1.5 marks each) 6

b Elements of management commentary (Any 3 points @1 3


mark each)
Explanation of the usefulness of the elements for social and environmental
disclosures (Any 3 points @ 2 marks each) 6

15

EXAMINER'S REPORT
The question examines candidates understanding of management commentary and a practical
application of how it can be used to serve the purpose of social and environmental reporting.

The major problem is its application to serve the purpose of social and environmental reporting.
Majority of the candidates simply listed the content elements without
adapting them to the question asked forgetting they were not just asked to discuss the elements.

SOLUTION 6

a. Golden Path Plc - Report to Board of Directors

Date
The Chairman Board
of Directors Golden
Path Plc.

Dear Sir,

Introduction

Our discussions on the above subject matter refer. This report sets out the reasons why your financial
statements may not meet the capital allocation needs of providers of financial capital in 21 st century
firms. It also identifies the most relevant content elements of Integrated Reporting and how they
address the limitations of the financial statements earlier highlighted.
Limitations of financial statements in meeting capital allocation decision making
needs

Providers of capital and other different users of financial statements have criticised the traditional
corporate reporting based on its inability to meet their needs. The major limitations of financial
statements in meeting the information needs of capital allocation in 21st century firms are:

i. Historical performance- income, cash flows, earnings and statement of financial position are
historical as opposed to future performance that investors are interested in.
ii. Intangible assets such as internally generated goodwill, intellectual, human and social relationship
capital are not recognised. These form the important assets for value creation in 21 st century
firms.
iii. As a result of (ii) above, there may be misallocation of capital in 21st century firms. Reporting fails
to incorporate or show the core capabilities and competences, hence the most important assets in
value creation.
iv. B Financial reporting suffer information overload, have become complex and increasingly
cluttered because of too many disclosures that are boiler plates.
v. Financial reporting is based on estimate and the subjective nature of estimates reduces relevance
and usefulness
vi. 1n reporting, management do exercise judgmental discretion on how certain transactions are
accounted for. For example many standards allow judgment
vii. Fair value improves reporting and yet the standards allow historical cost as the basis of initial
recognition such as in 1AS 16, 38 etc.

The content elements of 1ntegrated Reporting that addresses some of the limitations of financial
statements:
1ntegrated Reporting (1R) provides greater context for performance data, clarifies how value relevant
information fits into operations or a business, and may help make company decision making more long-
term. While the contents of 1R will be of benefit to a range of stakeholders, they are principally aimed
at providers of financial capital allocation decisions:

i. Future outlook: 1R is expected to show what challenges and uncertainties the organization is likely
to encounter in pursuing its strategy, and what are the potential implications for its business
model and future performance? This will address the historical nature of financial information.
ii. Business model: 1R is expected to show what the organization's business model is and to what
extent it is resilient. The business model includes information on the capitals such as natural
capital, human capital, intellectual capital and social and relationship capital thereby addressing
the limitations of intangible assets and human capital.
iii. Strategy and resources allocation: 1R is expected to show where does the organization want to go
and how does it intend to get there? This provide context to the future outlook and helps
overcome the historic nature of financial reporting information, not recognised presently in
Financial Reporting.
iv. Opportunities and risks: 1R is expected to show what are the specific risks and opportunities that
affect the organization's ability to create value over the short, medium and long term, and how
the organisation is dealing with them. This information helps providers of financial capital in
assessing current historical and future performance.
v. Performance: 1R is expected to show to what extent the organization has achieved its strategic
objectives for the period and what its outcomes in terms of effects on the capitals are.
vi. Governance: 1R is expected to show how the organization's governance structure supports its
ability to create value in the short, medium and long term?
vii. Organizational overview and external environment: 1R is expected to show what the organization
does and what are the circumstances under which it operates?
viii. Basis of preparation and presentation: IR is expected to show how the organization determines
what matters to include in the integrated report and how such matters are quantified or
evaluated.

We hope the foregoing will be of benefit in your deliberations on the company's corporate
reporting requirements.

Yours faithfully

Signed
The Reporting Consultants

b. Limitations of Integrated Reporting


Based on the foregoing, it is clear that integrated reporting will only address certain limitations whereas
others will still remain, examples include judgement, clutter, estimates, etc.

Thus IR may still not address some of the limitations noted in traditional corporate reporting because
different organisations may use different approaches in compiling their report so as not to reveal more
information than required to competitors, for example on its strategies in creating value. Thus most
entities that use IR may still not disclose details of some of the content elements of IR.

Marking Guide

a i. Limitations of financial reporting (Any 5 points @ 1 mark


each) 5

ii. Content Elements of Integrated Reporting (8 points @ 1 mark each)


8
Presentation in Report format
1 14
B Limitations of Integrated Reporting

_1

15

EXAMINER'S REPORT
The question examines limitations of traditional financial reporting in meeting capital allocation decision
making needs a 21st century firms. It then requires students to discuss the content elements of
integrated reporting that addresses some of the limitations of traditional financial reporting and
whether integrated reporting itself can resolve all the issues.
Most candidates attempted the question but their performance was not impressive. Most of them
displayed a poor understanding of the nature, contents and limitations of both traditional financial and
integrated reporting which cover more than fifty percent of the available scores.

Candidates are advised to adequately cover all aspects of the syllabus and 1CAN Study Packs as well as
practice past questions of 1CAN and other appropriate examination.

SOLUTION 7

(a) 1BRO PLC

The exclusion of the remuneration of the non-executive directors from key management personnel
disclosures did not comply with the requirements of IAS 24 which defines key management personnel as
those persons having authority and responsibility for planning, directing and controlling the activities of
the entity, directly or indirectly, including any director (whether executive or otherwise) of that entity.
1BRO PLC. did not comply with paragraph 16 of the standard, which also requires key management
personnel remuneration to be analysed by category. The explanation of 1BRO PLC is not acceptable.

IAS 24 states that an entity should disclose key management personnel compensation in total and for
each of the following categories:

(i) short-term employee benefits;

(ii) post-employment benefits;

(iii) other long-term benefits;

(iv) termination benefits; and

(v) share-based payment.


Providing such disclosure will not give information on what individual board members earn as only
totals for each category need be disclosed, which will not breach any cultural protocol. However
legislation from local government and almost certainly local corporate governance will require greater
disclosure for public entities such as 1BRO PLC.

By not providing an analysis of the total remuneration into the categories prescribed by the standard,
the disclosure of key management personnel did not comply with the requirements of 1AS 24. The
standard also requires 1BRO PLC to disclose other
transactions like loans granted to directors at concessional rates and their dealing in the loan stock of
the company.

(b) KOKORO JOBIJOBI GROUP

IFRS 2 Share-based Payment includes within its scope transfers of equity instruments of an entity's
parent in return for goods or services. The standard provides a clear basis to determine the classification
of awards in both consolidated and separate financial statements by setting out the circumstances in
which group share-based payment transactions are treated as equity settled and cash settled. The entity
receiving goods or services should assess its own rights and obligations as well as the nature of awards
granted in order to determine the accounting treatment. The amount recognised by the group entity
receiving the goods or services will not necessarily be consistent with the amount recognised in the
consolidated financial statements. Group share-based payment transactions are treated as equity
settled when:

(i) the awards granted are the entity's own equity instruments, or

(ii) the entity has no obligation to settle the share-based payment transaction.

In KOKORO JOBIJOBI GROUP accounts, the transaction is treated as equity settled as the group is
receiving all of the services in consideration for the group's equity instruments. An expense is charged in
the group statement of profit or loss for the fair value of the share-based payment at the grant date
over the vesting period, with a corresponding credit in equity.

In the subsidiaries' accounts, the grant is treated as equity settled transaction as the subsidiaries do not
have an obligation to settle the award. An expense is charged in the subsidiaries' statements of profit or
loss for the fair value of the share-based payment at the grant date over the vesting period, with a
corresponding credit in equity. The credit in equity is treated as a capital contribution as KOKORO
JOBIJOBI is compensating the employees of GBANJA and GORO with no expense to the subsidiaries. In
this case the shares vest immediately, therefore the expense recognised in GBANJA'S and GORO'S
statement of profit or loss will be the full cost of the grant date fair value.

In the separate accounts of KOKORO JOBIJOBI, there is no share-based payment charge as there are no
employees providing services to the parent. KOKORO JOBIJOBI would recognise an increase in its
investment in the subsidiaries and a credit to equity.

The disclosure requirements of IAS 24 Related Party Disclosures by KOKORO JOBIJOBI should be applied
if any of the employees are key management personnel.
Marking Guide

a IAS 24 Definition of key management personnel 2

IAS 24 Disclosure requirements on key management personnel 2

Appropriate disclosure of specified transactions 3 7

B Terms of IFRS 2 2

Treatment of transaction in Parent Coy Account 2

Treatment of transaction in Subsidiaries' Accounts 2

Treatment of transaction in Group Account 2 8


15

EXAMINER'S REPORT
The question tests candidates' understanding of the principles and applications of IAS 24 - Related
Party Disclosures and IFRS 2 - Share-based Payment. For IAS 24, candidates are specifically expected to
understand the profile of key management personnel, disclosure requirements on key management
personnel and appropriate disclosure of specified transactions. On IFRS 2, candidates are expected to
explain the treatment of share options granted to employees of subsidiaries in the financial statements
of the parent company, the subsidiaries and the group; without making a charge to the subsidiaries.

Only a few of the candidates attempted this question and their performance was below average. The
major pitfall was their inability to properly account for the share options in the financial statements of
the group, the parent company and the subsidiaries.

Candidates are advised to properly study IFRSs and master the practical application of the relevant
principles in different scenarios and cases.
THE INSTITUTE OF CHARTERED ACCOUNTANTS OF NIGERIA PROFESSIONAL LEVEL

EXAMINATION - NOVEMBER 2016 CORPORATE REPORTING


Time Allowed: 3 hours
INSTRUCTION: YOU ARE REQUIRED TO ANSWER FIVE OUT OF SEVEN
QUESTIONS IN THIS PAPER

SECTION A: COMPULSORY (30 MARKS)


QUESTION 1

a. Bata Pic, which operates in the manufacturing sector, has been surviving the challenges
operating in the Nigeria economic environment. The draft Statements of Financial Position of
Bata Pic and its subsidiaries as at October 31, 2016 are as follows:

Bata Jewe Gaba


Non-current assets N'million N'million N'million

Property, plant and equipment 4,320 360 420


Investments in subsidiaries 1,110 600 -
Financial assets 500 - -
5,930 960 420
Current assets 1,050 570 540
Total assets 6, 980 1,530 960

Equity
Share capital - N1 ordinary shares 2,400 600 300
Retained earnings 3,410 540 390
Other components of equity 450 - -
Total equity 6,260 1,140 690

Current liabilities 720 390 270

Total liabilities and equity 6,980 1,530 960


The following information is relevant to the preparation of the group financial statements:

(i) Bata Plc acquired 60% of the share capital of Jewe Plc. on November 1, 2012 and 10% of Gaba
on November 1, 2013. The costs of the combinations were N852million and N258 million
respectively. Jewe Plc acquired 70% of the share capital of Gaba Plc on November 1, 2013.
(ii) The balances on retained earnings were

November 1, 2012 November 1, 2013


N'million N'million
Jewe Plc . 270 360
Gaba Plc 180 240

(iii) At the respective dates of acquisitions, the fair value of the net assets of Jewe Plc was
N930million and for Gaba Plc N660million. The difference in the fair value and book value of
the net assets at acquisition dates relates to non-depreciable land. The fair value of the non-
controlling interest in Jewe Plc at the date of acquisition was estimated at N390million and for
Gaba Plc, N330million. Bata Plc adopts the full goodwill method under IFRS 3 Business
Combinations to account for non-controlling interest.
(iv) Given the economic environment in the country, impairment test was carried out for the
subsidiaries and it was discovered that whereas Gaba Plc suffered no impairment loss due to its
line of business, Jewe Plc had suffered an impairment loss of N60million.
(v) During the year ended October 31, 2016, Bata Plc had sold inventory to both Jewe Plc and Gaba
Plc. The invoiced prices of the inventories were N480million and N360million respectively.
Bata Plc invoices goods to achieve a mark-up of 25% on cost to all third parties including
group companies. At the year-end, half of the inventory sold to Jewe Plc remained unsold but
the entire inventory sold to Gaba Plc had been sold to third parties.
(vi) Bata Plc purchased a deep discount bond, an innovative financial instrument in the Nigerian
Capital Market for N500million on November 1, 2015 switching from its equity holdings in
that market where equities had suffered huge losses. The bonds will be redeemed in 3 years
time for N740.75million and are carried at amortised cost in line with 1AS 39 Financial
Instruments - Recognition and Measurement. The Accountant is not clear as to the correct
treatment of amortised cost and as such has not passed the correct entry to give effect to
amortised cost valuation at year end in the financial statements.
As such the financial asset is shown at N500million.
Compound sum of N1: (1+r)n is as follows:
Year 12% 14%
1 1.1200 1.1400
2 1.2544 1.2996
3 1.4049 1.4815
4 1.5735 1.6890

Required:
Prepare a Consolidated Statement of Financial Position for Bata Plc. and its subsidiaries as at
October 31, 2016.
(25Marks)

b. As a result of the challenges in the foreign exchange market since the advent of the current
administration which has made it difficult to source foreign exchange to import raw materials,
the Directors of Bata Plc are now considering acquiring a foreign subsidiary which may
facilitate access to such foreign exchanges that may be needed. They are not fully aware of the
requirements of IAS 21 'The Effects of Changes in Foreign Exchange Rates in relation to
translating the financial statement of a foreign subsidiary.'

Required:
Briefly explain to the directors of Bata Plc how the assets, liabilities, income and expenses of a
foreign subsidiary including the resulting goodwill are translated for consolidation purposes
and the treatment of exchange difference arising from the translation. (5 Marks)
(Total 30 Marks)

SECTION B: YOU ARE REQUIRED TO ANSWER ANY TWO OUT OF THREE QUESTIONS
IN THIS SECTION (40 MARKS)

QUESTION 2
The objective of IAS 33 - Earnings Per Share is to improve the comparability of the performance of
different entities in the same period and of the same entity in different accounting periods. This is
done by prescribing the methods for determining the numbers of shares to be included in the
calculation of earnings per share. The management of Soar Plc had sought for your professional
advice on the application of IAS 33.
a. You are required to advise the management of Soar Pic on the:
i. Significance of earnings per share. (5 marks)
ii. Shortcomings of earnings per share
(5 marks)

b. The directors of Soar Pic have decided to replace most of the existing plant and machinery
which are now obsolete during the year ended September 30, 2015 in order to enhance
earnings. The costs of removing existing plant and acquiring and installing new plant have
been estimated at N750,000.
In order to improve liquidity, the directors decided to make a new issue of
800,0 ordinary shares at N2 per share fully paid on January 1, 2015 and a further
N600,000 4% convertible loan notes on June 1, 2015. The terms of issue would provide for
conversion into ordinary shares as stated below:

On September 30 Number of shares


per N100 of loan
2015 120stock
2016 125
2017 118
2018 122

The ordinary shares issued would rank for dividend in the current year. The following relates
to the company for the period ended September 30, 2015:
• Profit before interest and tax is N850,000.
• Effective rate of company tax on profit is 30% and the basic EPS for the year ended
September 30, 2014 was 48kobo.
• The company had issued as at September 30, 2014 the following:
• 2,000,000 ordinary shares of 50 kobo each fully paid
• 400,000 12% irredeemable preference shares of N1 each fully paid
• 300,000 10% redeemable preference shares of N1 each fully paid.
• N700,000 8% redeemable debenture (non-convertible)

Required:
Calculate for Soar Plc for the year ended September 30,2015:
i. Basic earnings per share (5 marks)
ii. Fully diluted earnings per share (5 marks)
(Total 20 marks)
QUESTION 3

Nationwide Plc is a conglomerate with subsidiaries in two geographical locations. Each of the
subsidiaries has stamped its foot in relevant subsectors and contributes to the group's gross e arnings.
Segment information are prepared on the basis of geographical areas as well as business lines.

Segment Information By Geographical Areas as at December 31,2012


Nigeria Europe Total
Subsidiary I N'm N'm N'm
Derived From External Customers 110,419 2375 112,794
Total Revenue 110,419 2,375 112,794
Interest And Similar Expenses (25,398) (271) (25,669)
Operating Income 85,021 2,104 87,125
Share Of Profit Of Equity Accounted Investee 1,850 - 1,850
86,871 2,104 88,975
Operating Expenses (75,507) (1,530) (77,037)
Net Impairment Loss On Financial Assets (2,772) (106) (2,878)
Profit Before Taxation 8,592 468 9,060
Income Tax Credit/(Expenses) (1,572) (113) (1,685)
Profit After Taxation 7.020 355 7.375
Assets And Liabilities

Total Assets 954,165 78,882 1,033,047


Total Liabilities (781,019) (57,630) (838,649)
Net Assets 173.146 21.252 194.398

Subsidiary II Nigeria Europe Total


(N'M) (N'M) (N'M)
Derived From External Customers 82,566 2,535 85,101
Total Revenue 82,566 2,535 85,101
Interest And Similar Expenses (34,049) (263) (34,312)
Operating Income 48,517 2,272 50,789
Share Of Profit Of Equity Accounted Investee 952 - 952
49,469 2,272 51,741
Operating Expenses (88,429) (1,468) (89,897)
Net Impairment Loss On Financial Assets (69,525) (3) (69,528)
(Loss)/Profit Before Taxation (108,485) 801 (107,684)
Income Tax Credit/(Expense) 25,346 (213) 25,133
(Loss)/Profit After Taxation (83.139) 588 (82.551)
Assets And Liabilities
Total Assets 899,434 155,300 1,054,734
Total Liabilities (711,678) (143,684) (855,362)
Net Assets 187.756 11.616 199.372
Required:
You are required to appraise the contributions of each of the geographical locations to the groups
performance through a vertical analysis from the segment information.
(Total 20 Marks)

QUESTION 4
(a)Prior to the advent of IFRS 13, many standards such as IAS 16, IAS 38, IAS 40 and IAS 39
among others require the use of fair value. These various requirements have been harmonised
with the introduction of IFRS 13 Fair Value Measurement.

Required:
Define fair value in accordance with IFRS 13 (2 Marks)

(b) One of the companies formally operating in Nigeria that had recently relocated
its operation to Ghana as a result of the challenging business environment in
Nigeria has access to both Lagos and Accra market for its product. The product
sells at slightly different prices (in naira) in the two active markets. An entity
enters into transactions in both markets and can access the price in those
markets for the product at the measurement date as follows:

Lagos Accra
Market Market
N'000 N'000
Sale price 260 250
Transaction cost (30) (10)
Transport cost (20) (20)
Net price received 210 220

Required
i. Briefly explain the principal market of an asset in accordance with IFRS 13
and determine what fair value would be used to measure the sale of the
above product if the Lagos market were the principal market?
(4 Marks)
How is fair value determined in the absence of a principal market and what
ii.
fair value would be used to measure the sale of the above product if no
principal market could be identified? (4 Marks)
(c) Megida Plc, a public limited liability company, has just acquired some hectares of land in Abuja
earmarked by government for economic empowerment program of citizens given the harsh
economic environment in Nigeria and so is only meant for commercial purposes. The fair value
of the land if used for commercial purpose is N100million. 1f the land is used for commercial
purpose it is expected that it will result in reducing unemplo yment. This will attract a tax credit
annually, which is based upon the lower of 15% of the fair market value of the land or
N10,000,000 at the current tax rate. The current tax rate as fixed by the government is 20%.
Megida Plc has determined that given the nature of Abuja's land, market participants would
consider that it could have an alternative use for residential purposes. The fair value of the land
Megida Plc has just acquired for residential purposes before associated costs is estimated to be
N148 million. 1n order to transform the land from its commercial purposes to residential use,
there is estimated legal costs of N4,000,000, a project viability analysis cost of N6,000,000 and
costs of demolition of the commercial buildings of N2,000,000. 1n addition, permission for
residential use has not been formally given by Abuja Municipal Authority. This has created
uncertainty in the minds of market participants. Consequently, the market participants have
indicated that the fair value of the land, after the above costs, would be discounted by 20%
because of the risk of not obtaining the planning permission from Abuja Municipal Authority.
Required:
Discuss the way in which Megida Plc should compute the fair value of the Abuja l and with
reference to the principles of 1FRS 13 Fair Value Measurement.
(10 Marks)
(Total 20 Marks)

SECTION C: YOU ARE REQUIRED TO ANSWER ANY TWO OUT OF THREE QUESTIONS
IN THIS SECTION (30 MARKS)

QUESTION 5

Manipulation of reporting entities book's and records have been termed in many quarters as "Creative
Accounting" and "Window Dressing". The Management of Wastage Plc requires clarification of these
two concepts.
You are required to write a report to the management of Wastage Plc,. Your report should include:

a. Definitions of the TWO concepts (2 Marks)


b. FIVE examples of each (5 Marks)
c. THREE possible reasons for Creative Accounting and Window Dressing.
(3 Marks)
d. Advise to management on FIVE possible preventive measures of Creative
Accounting (5 Marks)
(Total 15 Marks)

QUESTION 6

Maranathan Plc acquired a property for N4million on which annual depreciation is charged on a
straight line basis at the rate of 7.5%. An impairment loss of N350,000 was recognised at the end of
May 31, 2013 financial year when accumulated depreciation was N1million. Consequently, the
property was valued at its estimated value in use. The company planned to move to new premises,
before, the property was classified as held for sale on October 1, 2013. By this time, the fair value
less costs to sell was N2.4million. Maranathan Plc published interim financial statements on
December 1, 2013, by which time the property market had improved and the fair value less costs to
sell was reassessed at N 2.52million. At the year end, on May 31, 2014 it had improved further, so
that the fair value less costs to sell was N2.95million. The property was disposed off eventually on
June 5, 2014 for N3million.
Required:
a. Assess the above transactions based on the requirements of IFRS 5, Non-current
Assets Held for Sale and Discontinued Operations. (5 marks)
b. Evaluate the impact of the events occurring on the property over time and on the reported gain
in accordance with IAS 10, Events After the Reporting Period.
(10 Marks)
(Total 15 Marks)

QUESTION 7

a. ABC Plc, in accordance with the regulations of the Nigerian Stock Exchange on transition to
IFRS, prepared its first IFRS Financial Statement in 2012. The Financial Statement was
contained in a voluminous document of 155 pages. Some of the stakeholders found it difficult
to understand the essence of the voluminous document.
You are required to prepare a brief report, highlighting the essence and merits of the adoption
of 1FRS by Nigerian Companies and state some of the challenges that could be encountered.
(10 Marks)

b. Statements of Accounting Standards (SAS) in Nigeria have been replaced by 1nternational


Financial Reporting Standards (1FRS); however, some of these local standards relatin g to
industry specific rules which are not found in 1FRS are expected to be applied by companies in
the industries as far as they do not conflict with 1FRS.
You are required to examine the above statement and identify those statements of Accounting
standards that are still applicable after the adoption of 1FRS.
(5 Marks)
(Total 15 Marks)
SOLUTIONS
SOLUTION 1

a.
BATA AND ITS SUBSIDIARIES
CONSOLIDATED STATEMENT OF FINANCIAL POSITION AS AT OCTOBER 31, 2016

________________________________________________________ N'm _______ N'm


Non-current assets
Property, plant and equipment (wk2) 5,280.00
Goodwill (wk3) 540.00
Financial Asset (wk5) 570.00
Current Assets (wk4) 2,112.00
Total Assets 8.502.00

Equity and Liabilities


Equity
Share capital 2,400.00
Other components of equity 450.00
Retained earnings (wk 6) 3,631.20
Equity attributable to owners of the parent 6,481.20
Non-controlling interest (wk 7) 640.00
Current liabilities 1,380.00
Total Assets 8.502.00
Bata in Gaba is therefore

Direct 10%
Indirect 60% x 70% 42%
52%

Working 2
Property, Plant and equipment
N'm N'm
Property, plant and equipments:
- Bata 4,320
- Jewe - 360
Garba 420
5,100
Fair Value land
- Jewe 60
-Garba 120 180
5,280

The cost of investment paid by Jewe in Gaba belongs to Bata 60% x N600 million ie N360 million.

GOODWILL Jewe Gaba

N'm N'm N'm N'm


Purchase Consideration: Bata 852 258
Jewe 60% x 600 360
NCI (Note 1) at fair value 390 330
1,242 948
Net asset at acquisition
Share capital 600 300 300
Retained earnings 270 240 240
Fair value land 60 930 120 (660)
Goodwill 312 288
Impairment (60)
Goodwill to SOFP 252 288
Total Goodwill on Consolidation (252 + 288) = 540
(4) Unrealised profit on inventory
Since Gaba has sold all its inventory to third parties, the entire profit has been realised.
Unrealised profit applies to unsold intra-group inventory, hence only sales to Jewe:

25/125 X (N480m/2) = N48 million


Accounting entry will be Dr Group Retained earnings Cr Inventory.

(5) Deep Discount Bond


The bond has no coupon rate but it will be redeemed at an amount far higher than its purchase
price and as such its effective interest is first determined as follows:

740.75m/500m = 1.4815. As the bond is redeemable in 3 years time, we look at the compound
rate that corresponds with this number of years. That rate is 14%.
Since the valuation is just for 1 year, the value of the bond at the end of its first financial year
using amortised cost should be Cost + effective interest as there is no coupon rate.

Effective interest is 14% x N500 million = N70 million


Accounting entry will be Dr Financial Asset and Cr Retained Earnings with N70 million

(6) Consolidated Retained Earnings


Bata Jewe Gaba
N'million N'million N'million
As per question 3,410 540 390
Pre-acquisition retained earnings (270) (240)
Bata's share of post acq. Ret. Jewe 60% X 270 162 270 150
Bata's share of post acq. Ret. Gaba 52% X 150 78
Interest on Bond 70
Impairment of goodwill (60)
Unrealised profit on lnventory(.6 x48) (28.8)
3.631.2
(7) Non-controlling interest (NCI)
Jewe Gaba
N'million N'million
NC1 (fair value) at acquisition 390 330
Share of post acquisition retained earnings:
Jewe 40% X 270 108
Bata's share of post acq. Ret. Earnings: 48% X 150 72
Unrealised profit on Inventory (.4 X 48) (19.2) ---
NC1 Investment in Gaba (40% x 600) (240) —

Total NC1 (238.8 + 402) = 640.8 238.8 402

From: Financial Controller

To: The Directors,

Bata Plc

Translation of a foreign subsidiary's financial statements

According to IAS 21 "The Effects of Changes in Foreign Exchange Rates'' in


consolidating the financial statements of a foreign subsidiary, the following
items of the foreign subsidiary should be translated as follows:

1. Assets and liabilities are translated at the closing rate at the date of the statement of
financial position.
2. Income and expenses are to be translated at the exchange rate at the date of the
transaction. Average rates are allowed if there is no great fluctuation in the exchange
rates.
3. Goodwill, including any fair value adjustments are treated as assets and liabilities of
the foreign subsidiary and are therefore retranslated at each statement of financial
position date at the closing spot rate
4. Exchange differences are recognised in other comprehensive income

5. Any exchange that relates to the non-controlling interest is recognised in the statement
of financial position.

1. Dee
Financial Controller
MARKING GUIDE
Marks Marks
A. i. Statement of Financial position 9
ii. Goodwill computation 6
1
iii. Co 1
mputation of unrealised profit 5
iv. Co 3
v.
mputation of financial assets Computatio
n of Retained earnings
vi. Computatio
n of NCI 25
B. Asset & Liability translation rate 1
Income & Expenses translation rate 1
Goodwill translation rate Exchange
Difference translation rate Total 5
30

EXAMINER'S REPORT

The question tests preparation and presentation of consolidated financial statement of a mixed group
structure.

All the candidates attempted the question and the performance was average.

The commonest pitfall is their inability to calculate the deep discount bond.

This aspect of the syllabus has been examined on regular basis, therefore, candidates are advised to
study and understand the principles guiding various aspects of group financial statements.
SOLUTION 2
(a)

The Managing Director/CEO


Soar Plc

Dear Sir,
EARNINGS PER SHARE

1n response to our recent discussion on the above subject matter, below is the
significance and shortcomings of earnings per share:

Significance of Earnings Per Share (EPS)

i. EPS (especially growth in EPS) can be used to measure financial performance between
entities (or of different periods for the same entity).
ii. EPS (especially growth in EPS) can be used for investment decisions.

iii. EPS has a significant impact on the share price of an entity (as it affects the calculation
of share price).
iv. EPS can also serve as a means of assessing the stewardship and management role
performed by company directors and managers.
v. Diluted earnings per share alerts investors of risk attached to their shares as a result of
obligations entered into by the entity. That is, the possibility in reduction of their
return on investment in future periods.

Short comings of Earnings Per Share

i. Comparison of effective performance between entities may be hindered if the entities


operate under different accounting policies.
ii. Results may be misleading if the impact of inflation is not considered in the
preparation of financial statements.
iii. EPS is not a complete tool for investment analysis as it cannot provide information on
liquidity position of the entity.
iv. 1t is based on historical information and therefore does not necessarily provide
predictive value. High earnings and growth in earnings may be achieved at the
expense of investment which may generate increased earnings in the future.
v. Diluted EPS is only an additional measure of past performance despite looking at
future potential shares.
vi. Linking remuneration package to EPS growth may lead to pressure on management to
produce a favourable EPS, thereby distorting it.

Yours faithfully,

Financial Accountant

( b) SOAR PLC
Calculation of weighted average no. of shares

No. of shares

Existing number of shares (2,000,000 x 12/12) = 2,000,000

New issue at 1/1/2015 (800,000 x 9/12) = 600,000

BASIC 2.600.000

Calculation of equity earnings N'000


Profit before interest and tax 850,000
Less finance costs:
Non-convertible redeemable debentures (8% x 700,000) (56,000)
Convertible debentures [4% x 600,000 x (4mths/12mths)] (8,000)
Redeemable preference dividend (10% x 300,000) (30,000)
Profit before tax 756.000
Income tax expense @ 30% (226,800)
Profit after tax 529.200
Less irredeemable preference dividend (12% x 400,000) (48,000)
BASIC 481.200

EPS = Equity earnings


No. of ordinary shares

Basic EPS = N481,200/2,600,000 shares = 18.51kobo


Test for dilution

Incremental EPS =lnterest savings net of tax on convertible stock


No of ordinary shares that would be exchanged for the convertible stock

MARKING GUIDE
Marks
a. i. Significance of Earnings Per Share 5
ii. Shortcomings of Earnings Per Share 5

b. i. Calculation of Basic earnings per share 5


ii. Calculation of diluted earnings per share 5
Total 20

Examiner's Report
The question tests candidates' knowledge of Earnings Per Share.

About 85% of the candidates attempted the question and performance was above average.

Majority of the candidates understood the requirements of part 'a' of the question whilst there was
poor understanding of requirements of 'b'. Also some candidates were unable to compute the test for
dilution

Earnings Per Share is an important component of Corporate Reporting syllabus, therefore candidates,
should pay more attention on the aspect of the syllabus for better performance in future.
SOLUTION 3
VERTICAL ANALYSIS OF STATEMENT OF PROFIT OR LOSS AND NET ASSSETS

Nigerian Region Europe Region Regional


Contribution

Subsidiary Subsidiary Region Subsidiary Subsidiary Region Group Vertical Nigeria Europe
I II Total I II Total Total Analysis
as % of as % of
Total Total
Total Revenues 110,419 82,566 192,985 2,375 2,535 4,910 197,895 100.00 97.52 2.48

Interest And Similar-25,398 -34,049 -59,447 -271 -263 -534 -59,981 30.31 99.11 0.89
Expenses
Operating Income 85,021 48,517 133,538 2,104 2,272 4,376 137,914 69.69 96.83 3.17

Total Income 86,871 49,469 136,340 2,104 2,272 4,376 140,716 71.11 96.89 3.11

Operating -75,507 -88,429 -163,936 -1,530 -1,468 -2,998 -166,934 -84.35 98.20 1.80
Expenses
Net Impairment Loss On-2772 -69,525 -72,297 -106 -3 -109 -72,406 -36.59 99.85 0.15
Financial Assets

(Loss)/ Profit Before8,592 -108,485 -99,893 468 801 1,269 -98,624 -49.84 101.29 -1.29
Taxation
Income Tax Expense -1,572 25,346 23,774 -113 -213 -326 23,448 11.85 101.39 -1.39

(Loss)/ Profit After7,020 -83,139 -76,119 355 588 943 -75,176 -37.99 101.25 -1.25
Taxation
Total Assets 954,165 899,434 1,853,599 78,882 155,300 234,182 2,087,781 100.00 88.78 11.22

Total Liabilities 781,019 711,678 1,492,697 57,630 143,684 201,314 1,694,011 81.14 88.12 11.83
Net Assets 173,146 187,756 360,902 21,252 11,616 32,868 393,770 18.86 91.65 8.35
Appraisal of Nationwide Plc Operation
The Nigerian region contributed significantly to the performance of the group across the business
line.

Income statement
The Nigerian region contributed 97.52% of the entire revenue of the group. With interest and similar
expenses being 30.31% of the entire revenue of which Nigeria contributed 99.11% leaving an
operating income which is 69.69% of the group revenue.
The operating expenses net impairment was more than the operating income of which the contribution
from Nigeria was a major component.

Asset and Liabilities


The liabilities was 81.14% of the group assets. Nigeria contributed 88.78% of the group asset while
also having 88.12% of the entire group liabilities.
An overall analysis shows that the major activities of the groups are concentrated in Nigeria.
This is evident in the percentage of the revenue and assets contributed by the Nigerian region to the
group.
This may pose a concentration risk to the entire group.

Tutorial Note
Vertical analysis is the proportional analysis of a financial statement, whereby each line item on a
financial statement is listed as a percentage of another item within a single period. Vertical analysis
between the segments requires a comparison of segment's figure with the group figure i.e. expressing
contribution of the segment as a proportion of the group total.
Marks
MARKING GUIDE
Computation and Analysis Table 14

Discussion of computation and Analysis 6


20

Examiner's Report
The question tests performance evaluation of a group of Companies using vertical financial analysis.
About 25% of the candidates attempted the question and the performance was poor.
The pitfall was that candidates could not understand the requirements of the question by confusing
financial appraisal with segment information.
Candidates are advised to understand the requirements of the question for better performance in
future examination.
SOLUTION 4

(a) Fair value is "the price that would be received to sell an asset or paid to transfer a liability in
an orderly transaction between market participants at the measurement date."
According to IFRS 13 the principal market is the market with the greatest volume and level of
activity for the asset or liability. The price in the principal market used to measure the fair
value of the asset (liability) is not adjusted for transaction costs.

(b)
i. lf Lagos market is the principal market for the product, its fair value
would be measured using the price that would be received in that market, after taking
into account transport costs at N240, 000.
N'000
Sale price 260
Transport cost (20)
Fair value 240
ii. lf no principal market could be identified, the fair value of the product
would be measured using the price in the most advantageous market.
The most advantageous market is the market that maximises the amount that would
be received to sell the asset, after taking into account transaction costs and transport
costs (i.e. comparing the net amount that would be received in the respective
markets).
This is the Accra market where the net amount that would be received for the product
would be N220,000, given the fact that transaction costs is not considered in the
determination of fair value. The fair value of the asset is determined as:
N'000
Sale price 250
Transport cost (20)
Fair value 230

(b) IFRS 13 requires the fair value of a non-financial asset to be measured based on its highest and
best use from a market participant's perspective. This requirement does not apply to financial
instruments, liabilities or equity. The highest and best use takes into account the use of the
asset which is physically possible, legally permissible and financially feasible. The highest
and best use of a non-financial asset is determined by reference to its use and not its
classification and is determined from the perspective of market participants. It does not matter
whether the entity intends to use the asset differently. IFRS 13 allows mana gement to presume
that the current use of an asset is the highest and best use unless market or other factors suggest
otherwise.
In this case, the economic empowerment land appears to have an alternative use as market
participants have considered its alternative use for residential purposes. If the land acquired by
Megida is used for economic empowerment purposes, the fair value should reflect the cost
structure to continue operating the land for that purpose, including any tax credits which could
be realised by market participants. Thus the fair value of the land if used for commercial
economic empowerment purposes would be:
N'000
Fair Value 100
Tax credit 2
102

If used for residential purposes, the value should include all costs associated with changing the
land to the market participant's intended use. In addition, demolition and other costs associated
with preparing the land for a different use should be included in the valuation. These costs
would include the uncertainty related to whether the approval needed for changing the usage
would be obtained, because market participants would take that into account when pricing
value of the land if it had a different use. Thus the fair value of the land if used for residential
purposes would be:

N'000
Fair Value 148
Legal cost (4)
Viability Analysis Cost (6)
Cost of demolition (2)
136
Discounted at 80%(80% of 136) 108.8

In this situation, the presumption that the current use is the highest and best use of the land has
been overridden by the market factors which indicate that residential development is the
highest and best use.
A use of an asset need not be legal at the measurement date, but it must not be legally
prohibited in the jurisdiction.
MARKING GUIDE
Marks Marks
Details of Question

a. Definition of Fair Value in accordance with IFRS 13 2

b. i Brief explanation of principal market 2


Determination of Fair Vale if principal market is identified 2 4

ii. Explanation for determination of fair value in the absence of 2


principal market
Computation of fair Vale if principal market is not identified 2 4

c. Discussion of the way fair value of Abuja land can be


determined in accordance with the principles of IFRS 13,
fair
value measurement 4
Computation fair value of Abuja land based on commercial
and residential purposes in accordance with the principles of 6 10
IFRS 13
Total Marks 20

EXAMINER'S REPORT
The question tests the provisions and application of IFRS 13, Fair Value measurement.
More than 50% of the candidates attempted the question and some of them do not have clear
understanding of the requirement.
The commonest pitfall was the inability of the candidates to understand IFRS 13 requirements, that
the highest and best use of a non-financial asset is determined from the prospective of market
participant. As such they were unable to distinguish fair value of land if used for commercial and
residential purposes.
Candidates should endeavour to cover all relevant IFRS and IAS in the professional level of the
examination of the Institute.
SOLUTION 5

a.

To: The Board of Directors, WASTAGE Plc.

From: The Consultant

Date: November, 15 2016

Subject: Concept of Creative Accounting and Window Dressing

Creative Accounting
Creative accounting can be defined as the presentation of information in a manner that is
inconsistent with the underlying facts. It can also be the application of accounting policies to
structure particular transactions in such a way that the financial statements will portray a picture
of financial health that is in line with what the directors would like users to see rather t han the
true financial performance and position of the business. The main purpose of creative accounting
is to inflate profit figure though some companies may also reduce report profits in good years
either to smooth results or depending on what the directors want to achieve. Assets and
Liabilities may also be manipulated either to remain within limit such as debt covenants or to
hide problems.

Window dressing
The term, "window dressing" is strategy used to create a superficial or misleading presentation
of financial performance in an illegal and unethical manner. The manipulation of investment
portfolio performance numbers is window dressing and creative accounting.
In a nutshell, the objective of window dressing and creative accounting are the same, but th e
ways of achieving their results differs in the sense that creative accounting involves taking the
advantage of the loopholes in the accounting laws and standards but window dressing does not
necessarily take advantage of the loopholes but sometimes intentionally go against the provision
of the accounting laws and standards.

(b) Common examples of Creative Accounting

(i) Examples of creative accounting

There are many types of creative accounting practices. Some of these are discussed
below.
Earnings management
The earnings management literature outlines how management may engage in income
smoothing practices so as to reduce the risk perceptions of their firms:
• Smoothing through allocation over time. Management can manipulate the periods in
which expenses are recognized. For example, the useful life of assets can be adjusted,
thereby affecting the amount of depreciation charged as an expense.
• Use of discretionary accruals. Income can also be smoothed by the use of discretionary
accruals. For example, management can determine the amount to be provided as a
provision for bad debts, or the amount of a provision in respect of a warranty on a
company's products or services.

Off-balance-sheet financing
This involves a firm's debt being omitted from its statement of financial position. This was
the main creative accounting technique employed by Enron, prior to its bankruptcy in 2001.

Revenue recognition
This is a technique to recognize revenue before it is earned. One of such practice, known as
'channel stuffing', involves a distributor supplying more goods to retail outlets than can be
sold to customers.

Overstating assets
This involves the failure by a firm to record impairments relating to the value of assets such
as machinery, property, inventory, investments and receivables.

Aggressive capitalization of costs


Some firms may decide to capitalize costs (e.g. research & development) which others write
off routinely.

Capitalisation of expenses that are meant to be expensed in order to reduce operating costs
and improve profit and total assets. e.g. Capitalization of research costs and training cost that
are meant to be expensed in the profit or loss.

Engaging in accounting measures that encourages profit smoothing.

Inappropriate and constant changes in accounting policies with the main aim of improving
profitability performance.
Inappropriate classification of some statement of financial position items as set off statement
of financial position by complying with legal form rather than substance. For example where
the lease deliberate classifies a finance lease transaction as an operating lease so as to avoid
the recognition of lease obligation (lease liability) in the statement of financial position.

b(ii) Examples of window dressing


• Non-recognition of irrecoverable debts ( i.e. bad debts)
• Understatement of provisions for doubtful debts (i.e. allowance for receivables).
• Non provision for damaged and obsolete inventories and assets.
• Deliberate overstatement of closing inventories.
• Deliberate recognition of unrealized income and profit arising from inter company
transactions as if they were realized in the consolidated financial statements.
• Deliberate revaluation of non-current asset downward at the beginning of the period
so as to charge lower depreciation and subsequently revaluing the same non-current
asset upward at the end of the period in order to increase total assets and improve
gearing.
• Arrangement with customers to make sales towards the end of an accounting period
while the goods sold can be returned at the beginning of the next period.
• Deliberate understatement of expenses.
• Deliberate overstatement of income.
• Deliberate recognition of income in advance in the current period profit or loss while
accrued expenses are not recognized so as to improve performance.
• The use of special purpose entity: To improve the returns on capital employed and
gearing of the sponsor.
• Inadequate disclosure of items in the financial statement as required by the applicable
accounting standards. e.g service concessions arrangement.

c. Common reasons for creative accounting and window dressing


i. To incur a lower tax liability.
ii. To improve remuneration packages of management
iii. To improve the quality and value of assets
iv. To improve the gearing position of the reporting entity
v. To reduce costs and improve profitability and performance
vi. To improve the market value or worth of the company's shares so as to attract
investors.
vii. To attract further credit facilities from loan providers at favorable terms.
viii. To achieve competitive advantage in the industry.
ix. Tomeet an expected target or performance in order to enjoy an
expected reward.
x. Toavoid certain sanctions by any regulatory authority or body.
xi. Tocontrol dividend payment
xii. Big bath theory which maintains that new management will maximise losses and
blame them on their predecessors. This should result in the new management
taking credit for improved results going forward

d. Preventive measures for creative accounting


It is unlikely that creative accounting can never be eliminated entirely. A number of
approaches can, however, be effective in reducing its incidence.

i. Financial regulation
Legislation provides the basic weaponry in the war against creative accounting. In
prescribing the statutory regulations that must be compiled with the relevant statutory
and regulatory requirements which establish a solid platform for the prevention of
creative accounting.
ii. Monitoring, enforcement and penalties
An external audit of a company's financial statements plays a crucial role in the
prevention of creative accounting. This involves an independent assessment to
determine whether the financial statements give a true and fair view of a company's
performance and position.
iii. Corporate governance
Adherence to a Corporate Governance Code can enable a company to establish
appropriate structures to prevent the use of creative accounting policies. For example,
the appointment of a majority of non-executive independent directors on a company's
board acts as a control against abuses by management.

iv. Focus on ethics


An emphasis on ethics in education can also play a significant role in helping to
reduce the use of creative accounting practices by management.
v. Creating a culture of whistle-blowing in organisations
Insiders are often aware of creative accounting practices long before they become
known to the outsider. Management can help to eradicate creative accounting
practices by providing support for a culture of whistle-blowing in their organisation.
This can assist in bringing accounting irregularities to light at an early stage.

vi. Culture of good Citizen


Creating a culture of good citizenship in an organisation helps to extend
management's focus beyond the needs of the shareholder/investor group. Thus, a
commitment to corporate social reporting makes management accountable to a
broader range of stakeholders, thus reducing the motivation to engage in creative
accounting.

vii. Ensuring that adequate controls are in place to checkmate noncompliance with
relevant accounting standards.

viii. Ensuring that the substance of transactions as against its legal form is reflected in the
financial statements.

ix. Ensuring that adequate disclosures of items in the financial statements are made to
improve transparency, completeness, neutrality and reliability.

Yours faithfully

Financial Controller
MARKING GUIDE
Marks
Definition of Creative Accounting and window dressing 2

Five (5) examples of window dressing and Creative Accounting at 1 mark each 5

Five(5) preventive measures for creative accounting at 1 mark each 5

Three (3) reasons for Creative Accounting at 1 mark each 3


Total 15

Examiner's Report

The question tests the knowledge and understanding of the creative accounting and window dressing.
About 90% of the Candidates attempted the question and the performance was average.
The commonest pitfall is the inability of the candidate to clearly distinguish between Window
dressing and Creative accounting.
Candidates are required to understand every aspect of the syllabus for better performance in future.
SOLUTION 6

(a) In accordance with the requirement of IFRS 5- Non-current assets held for sale
and Discontinued Operations, Maranathan Plc transactions would be treated as
follows:
i. The property would be classified as held for sale from 1 st Oct, 2013
It is assumed that the directors have made a decision to sell the property from 1 st Oct,
2013 and that active steps to locate a buyer and the property is being marketed at a
reasonable price.
ii. The property would be removed from Non-current assets and presented separately
under current assets in the statement of financial position.
iii. The property will no longer be depreciated from the date of classification(i.e from 1 st
Oct 2013) as held for sale, hence, it will only be depreciated for the first four months
before classification as held for sale (i.e from 1/6/2013 - 1/10/2013).
iv. The property would be measured at the lower of carrying amount and fair value less
costs to sell.
v. Impairment must be considered both at the time of classification as held for sale and
subsequently as follows:
- Any impairment loss is recognized on profit or loss unless the assets had been
revalued before in which case the impairment is treated as a revaluation decrease.
- A gain from any subsequent increase in fair value less costs to sell of an asset is
recognized in the profit or loss to the extent that it is not in excess of the
cumulative impairment loss already recognized in line with IFRS 5.
(b)
i. At the time of classification as held for sale, depreciation needs to be
charged for the four months to 1 October 2013. This will be based upon the year end
value at 31 May 2013 of N2-65 million. The property has 10 years life remaining
based upon the depreciation to date and assuming a zero residual value, the
depreciation for the four months will be approximately N0-883 million. Thus, at the
time of classification as held for sale, after charging depreciation for the four months
of N0-883million, the carrying amount is N2-561 million (N4m - N 1 - N 0-883 m - N
0-35m) and fair value less costs to sell is assessed at N2-4 million. Thus, the initial
write-down on classification as held for sale is N161,667 and the property is carried at
N2-4 million.
ii. On 1 December 2013 in the interim financial statements, the property
market has improved and fair value less costs to sell is reassessed at N2 -52 million.
The gain of N120,000 (i.e N2-52 million - N2-4 million) is less than the cumulative
impairment losses recognised to date(N350,000 plus N161,667, i.e. N511,667).
Therefore, it is credited in profit or loss and the property is carried at N2 -52 million.
iii. On 31 May 2014, the property market has continued to improve, and fair
value less costs to sell is now assessed at N2.95 million. The further gain of N430,000
is, however, in excess of the cumulative impairment losses recognised to date
(N350,000 plus N161,667 - N120,000 - N430,000, i.e. N38,333). Accordingly, a
restricted gain of N391,667 is credited in profit or loss and the property is carried at
N2,911,667 which is (N2.9m minus N38,333).
iv. Subsequently, the property is sold for N3million at which point a gain of N88,333 is
recognised. This sale would be a non-adjusting event under IAS 10 Events after the
Reporting Period if deemed to be material.

MARKING GUIDE
Marks Marks

a. Criteria for classifying Non-current asset as held


- For sale & Discontinued Operation 3
- Treatment of asset so classified 2 5

b. Evaluation of Impact of given transaction over time 8


Identification of Reported Gain Treatment 2 10
15

EXAMINER'S REPORT
The question tests candidates ability to apply the provisions and the requirements of IFRS 5 – Criteria
to classify a non-current assets as held for sale and discontinued operations and IAS
10 - Events after the reporting date.
The percentage attempt was about 40% and most candidates that attempted the question demonstrated
very poor understanding and performance was below average.
The commonest pitfall was that there were poor evaluation/ presentation of solution by majority of
the Candidates.
Professional level candidates are required to understand the provisions and applications of IFRS at
this level of the Institute's examination.
SOLUTION 7

a.

To: The Managing Director


ABC Plc
Lagos - Nigeria

Report of the Adoption of International Financial Reporting Standards (IFRS) by Nigeria Companies

Introduction:
International Financial Reporting Standards (IFRS) are a set of accounting standards developed by the International
Accounting Standards Board (1ASB) that is becoming the global standard for the preparation of public company
financial statements.
Convergence of accounting standards refers to the goal of establishing a single set of accounting
standards that will be used internationally. This is also described as the international harmonisation of
accounting standards.

Highlighting the essence and merit of the adoption of ifrs

There are some strong arguments in favour of the harmonisation of accounting standards in all
countries of the world.
i. Investors and analysts of financial statements can make better comparisons between the
financial position, financial performance and financial prospects of entities in different
countries. This is very important, in view of the rapid growth in international investment by
institutional investors.
ii. For international groups, harmonisation will simplify the preparation of group accounts. 1f all
entities in the group share the same accounting framework, there should be no need to make
adjustments for consolidation purposes.
iii. 1f all entities are using the same framework for financial reporting, management should find it
easier to monitor performance within their group.
iv. Global harmonisation of accounting framework may encourage growth in cross border trading,
because entities will find it easier to assess the financial position of customers and suppliers
in other countries.
v. Access to international finance should be easier, because banks and investors in
the international financial markets will find it easier to understand the financial information
presented to them by entities wishing to raise finance.
vi. Harmonisation could also lead to reduction in cost of capital as a result of 4 and 5 above.
vii. Reduction in audit costs
viii. It makes the transfer of Accounting staff easier
ix. It enables relevant Tax Authority to have appropriate understanding of the accounting
information.

Challenges of Harmonization Of Accounting Standards


i. National legal requirements may conflict with the requirements of IFRSs. Some countries may
have strict legal rules about preparing financial statements, as the statements are prepared
mainly for tax purposes. Consequently, laws may need re-writing to permit the accounting
policies required by IFRS.
ii. Some countries may believe that their framework is satisfactory or even superior to IFRS.
iii. Cultural differences across the world may mean that one set of accounting standards will not
be flexible enough to meet the needs of all users.
iv. Different User Groups
v. Lack of strong accounting regulatory bodies
vi. High cost of adopting IFRS to Nigerian Companies

Yours faithfully

Financial Controller

b. SAS STILL IN USE IN NIGERIA


In Nigeria, the date of transition from local standard to IFRS was established by the
provisions in the National Road Map on adoption of IFRS. The opening IFRS Statement of
financial position is prepared by full retrospective application of all IFRS extant at the first
IFRS reporting date with the provision that:

> Some standards relating to industry specific rules not found in IFRS should be applied by
companies in the industries concerned as far as they do not conflict with the
provisions of IFRS. Such standards include:
i. SAS 14: Accounting in the petroleum industry: Down-stream
activities
ii. SAS 17: Accounting in the petroleum industry: Up-stream activities
iii. SAS 25: Telecommunications activities.
MARKING GUIDE
Marks Marks
a. 2 points of Essence of IFRS 2
5 merits of IFRS 5
3 Challenges of IFRS 3 10

b. SAS still in force after IFRS adoption 5 5


Total 15

EXAMINER'S REPORT

The question tests the essence, merits and challenges of IFRS adoption in Nigeria.

The candidates demonstrated a fairly satisfactory understanding of the question. About 50% of the
Candidates attempted the question but did not perform well in Part b.
Commonest pitfall was the inability of the candidate to recognise the statements of Accounting
Standards still solely applicable to Nigerian environment after the adoption of IFRS.
Candidates should endeavour to familiarise themselves with 1CAN Study Texts for Institute's future
examinations.
THE INSTITUTE OF CHARTERED ACCOUNTANTS OF NIGERIA

PROFESSIONAL LEVEL EXAMINATION - MAY 2017

CORPORATE REPORTING

Time Allowed: 3 hours


INSTRUCTION: YOU ARE REQUIRED TO ANSWER FIVE OUT OF SEVEN
QUESTIONS IN THIS PAPER

SECTION A: COMPULSORY (30 MARKS)

QUESTION 1

Rapuya Pic. is a Nigerian public limited company operating in the mining industry. The
draft Statements of Financial Position of Rapuya Plc., and its two subsidiaries, Puta Limited
and Soma Limited as at April 30, 2017 are as follows:

Rapuya Puta Soma


Non-Current Assets N'm N'm Defa’m

Property, Plant and Equipment 740 220 760


Investments in subsidiaries
Puta 226 - -
Soma 184 - -
Financial Assets 30 14 100
1,180 234 860

Current Assets 236 200 660


Total Assets 1,416 434 1,520
Equity 316 76 400
Share Capital
Retained Earnings 604 112 600
Other Components of Equity (OCE) 14 8 -
934 196 1,000
Non-Current Liabilities 112 84 320
Current Liabilities 370 154 200
Total Liabilities 482 238 520
Total Equity and Liabilities 1,416 434 1,520

The following information is relevant to the preparation of the group financial statements:

(i) On May 1, 2016 Rapuya acquired 52% of the ordinary shares of Soma Limited
a foreign subsidiary. The retained earnings of Soma Limited on this date were
220million defas. The fair value of the identifiable net assets of Soma
Limited on May 1, 2016 was 990million defas. The excess of the fair value over the net
assets of Soma Limited is due to an increase in the value of nondepreciable land.

Rapuya Plc. wishes to use the 'full goodwill' method to consolidate the financial
statements of Soma. The fair value of the non-controlling interest in Soma Limited at
May 1, 2016 was 500million defas.

Soma Limited is located in Tome, a small country in West Africa and operates a mine.
The income of Soma Limited is denominated and settled in defas. The output of the
mine is routinely traded in defas and its price is determined initially by local supply
and demand. Soma Limited pays 30% of its costs and expenses in naira with the
remainder being incurred locally and settled in defas. Soma's management has a
considerable degree of authority and autonomy in carrying out the operations of
Soma Limited and is not dependent upon group companies for financial support. The
Finance Controller is not certain from the above whether the defas or naira should be
taken as the functional currency of Soma Limited.

There have been no issue of ordinary shares and no impairment of goodwill since
acquisition.

(ii) Also on May 1, 2016, Rapuya Plc. had acquired 70% of the equity interests of Puta
Limited. The purchase consideration amounted to N226million which Rapuya Plc.
paid through bank transfer in compliance with the cashless policy of the Federal
Government of Nigeria. The fair value of the identifiable net assets recognised by
Puta Limited was N240million excluding the patent below. The identifiable net assets
of Puta Limited at May 1, 2016 included a brand which had a fair value of N8million.
This had not been recognised in the financial statements of Puta Limited. The brand
is estimated to have a useful life of four years. The retained earnings of Puta Limited
were N98million and other components of equity were N6million at the date of
acquisition. The remaining excess of the fair value of the net assets is due to an
increase in the value of non-depreciable land.

Rapuya Plc. wishes to use the 'full goodwill' method in consolidating the financial
statements of this subsidiary. The fair value of the non-controlling interest in Puta
Limited was N92million on May 1, 2016. There have been no issue of ordinary shares
since acquisition and goodwill on acquisition is not impaired.

(iii) The following exchange rates are relevant for the preparation of the group financial
statements:

Defas to N
May 1, 3:1 2016
April 30, 2017 2.5:1
Average for year to April 30, 2017 2.9:1
Required:
(a) Advise the Finance Controller on what currency should be taken as the
functional currency of Soma Limited applying the principles set out in IAS 21
- The
Effects of Changes in Foreign Exchange Rates. (5 Marks)
(b) Prepare a consolidated statement of financial position of the Rapuya Group
as at April 30, 2017, in accordance with International Financial Reporting Standards
(IFRS). (Show all workings) (25 Marks)
(Total 30 Marks)

SECTION B: YOU ARE REQUIRED TO ANSWER ANY TWO OUT OF THREE QUESTIONS
IN THIS SECTION (40 MARKS)

QUESTION 2

The following figures have been extracted from the financial statements of Lanke Plc and its
subsidiaries for the years ended December 31, 2014 and 2015.

GROUP COMPANY
2015 2014 2015 2014
N ‘m N ‘m N ‘m N ‘m
Revenue 21,843 17,519 15,848 12,198
Cost of Sales 15,676 12,571 11,165 8,588
Gross Profit 6,167 4,948 4,683 3,610
Other Income 234 308 612 34
Selling & Distribution Exps. (1,413) (1,368) (951) (844)
Admin. Expenses (1,817) (1,702) (1,150) (1,260)
3,171 2,186 3,194 1,540
Interest Expenses (499) (546) (298) (429)
Profit before taxation & Non-
controlling Interest 2,672 1,640 2,896 1,111
Tax expenses (481) (283) (313) (185)
Profit after tax but before non-
controlling interest 2,191 1,357 2,583 926
Non-controlling interest (7) (191) (-) (-)
Profit for the year 2,184 1,166 2,583 926
Property, Plant and Equipment 9,587 9,102 6,381 6,876
Investments 3,767 3,131 4,609 3,731
13.354 12.233 10.990 10.607
Current Assets 7,076 6, 221 3, 820 3,772
Current Liabilities 7,750 7,863 3,463 5,777
Net-Current Assets/(Liabilities) (674) (1,642) 357 (2.005)
12,680 10,591 11,347 8,602
GROUP COMPANY
2015 2014 2015 2014
N ‘m N ‘m N ‘m N ‘m
Share Capital 454 454 454 454
Capital Reserve 3,182 3,182 3,147 3,147
General Reserve 6.283 4.793 6.176 4.218
Equity 9,919 8,429 9,777 7,819
Non controlling interest 776 906 Nil Nil
10,695 9,335 9,777 7,819
Loan Notes 1,985 1,256 1,570 783
12.680 10.591 11.347 8.602

The directors of Lanke Pic. would like to know how the individual performance of the
company and that of the group compares with each other and over the two years. In
particular they are interested in performance measure around profitability, long term
solvency and asset utilisation (using only the ratios indicated below). They would also want
a brief explanation of why the analysis of the performance of a single company may differ
from that of a group company.
Required:
Prepare a performance report that addresses the needs of the directors of Lanke Plc for the
two year period 2014 and 2015.
Note: Limit your ratio computation to the following:
- Return On Capital Employed (ROCE)
- Profit margin
- Asset turnover
- Gearing
- Interest cover
(Show all workings)
(Total 20 Marks)

QUESTION 3
a. The economic environment in the country has been very harsh and it is now a common
knowledge that the economy is in recession. This in turn may impact on the income
generating capacity of assets of companies especially in those industries experiencing a
downturn in fortunes. This calls for the financial reporting regulators to pay attention
to evidence of impairment of assets in the financial statements submitted by these
companies.
Required:
Discuss briefly the reasons why the Financial Reporting Council of Nigeria (FRCN)
should focus on the impairment of non-financial assets and deferred tax assets of listed
companies in Nigeria in this period of slow economic growth, setting out the key areas
which entities should focus on when accounting for these items. (7 Marks)
b. IAS 36 stipulates how a company should test for impairment of assets.
A multinational oil marketing company operating in Nigeria is not sure how to test
for impairment of its assets especially those that do not generate cash flows that are
independent of other assets.

Required:
(i) Identify TWO external and TWO internal indicators that an asset of the multinational
oil company may have been impaired.
(2 Marks)
(ii) Briefly discuss how the multinational oil company should test for impairment
of assets that do not generate independent cash flows. (6 Marks)

c. A cash generating unit holds the following assets:

N'Million
Goodwill 160
Patent 320
Property, plant and equipment 480

An annual impairment review is required as the cash generating unit contains goodwill. The
most recent review assesses its recoverable amount to be N720million. An impairment loss
of N240million has been incurred and has been recognised in profit or loss.

Required:
Show how the value of the assets held by the cash generating unit will change after the
impairment test based on the information provided above.
(5 Marks)
(Total 20 Marks)

QUESTION 4

Dango Plc is a conglomerate company operating in Nigeria with diverse interest across
Africa. It prepares its financial statements in accordance with International Financial
Reporting Standards with year ending September 30. The following transactions relate to
Dango Plc.
(a) In February 2016, Dango Plc won a significant new contract to supply large quantities
of rice to the government of Guyama, a small West African Country for the next two
years. Under the terms of the arrangement, payment is made on delivery in cash once
goods have been cleared by customs. The rice will be delivered in batches four (4)
times every year, on April 1, July 1, October 1 and January 1. The batches for April 1,
2016 and July 1, 2016 amounting to N250million and N380million respectively were
delivered and paid. Dango incurred significant costs on customs duties for the first
batch of
delivery. The October 1 batch valued at N520million was shipped prior to the year -
end but delivered and paid for on October 1, 2016.
(8 Marks)

(b) On October 1, 2010 a 12-year licence was awarded to Dango Plc by the Federal
Government to be the sole manufacturer of a chemical used in the Nigerian
pharmaceutical industry. The licence was recognised on that date at its fair value of
N196million. The award of the licence motivated Dango Plc in 2011 to purchase a
division of another Nigerian competitor company making similar products. Goodwill
of N240million was recognised on purchase of the division. Dango Plc merged the
activities of the newly acquired division with its own to create a specialist chemical
sub-division which it now classified as a separate cash-generating unit. By 2016, the
revenue of this cash generating unit now amounts to 5% of the Group's revenue.(4 Marks)

(c) Dango Plc. buys raw materials from overseas suppliers. It has recently taken delivery
of 1,000 units of components X, used in the production of chemicals.
The quoted price of component X was N1,200 per unit, but Dango Plc. has negotiated
a trade discount of 5% due to the size of the order.
The supplier offers an early settlement discount of 2% for payment within 30 days
and Dango Plc. intends to achieve this.
Import duties of N60 per unit must be paid before the goods are released through
customs.
Once the goods are released, Dango Plc. must pay a delivery cost of N5,000 to have
the components taken to its warehouse.

Required:

Write a report to the directors advising them on the correct accounting treatment of
the above transactions in the financial statements for the year ended September 30,
2016 in accordance with the provisions of the relevant standards.

Note: You may consider the relevance of the following standards to the transactions:
IAS 20, IAS 2, IAS 38, IFRS 3 and IFRS 15. (8 Marks)
(Total 20 Marks)
SECTION C: YOU ARE REQUIRED TO ANSWER ANY TWO OUT OF THREE QUESTIONS
IN THIS SECTION (30 MARKS)

QUESTION 5

An annual report is a comprehensive report on a company's activities intended to give


information about the company's activities and financial performance. 1n addition to the
audited financial statements, annual reports contain a great deal of extra information which
could be financial and non-financial. The extra information provided may be required by
law, hence, it is mandatory. However, many companies provide additional information not
required by law, on voluntary basis.

Required:

a. Identify THREE of such reports that are voluntarily disclosed in annual


reports of Nigerian companies. (3 Marks)

b. Why would a company disclose information not required by law in its annual
report? Propose FOUR reasons for and give any TWO limitations of such disclosures. (7 Marks)

c. 1nstitutional investors, if not all investors, need information about corporate


governance in order to make rational and reasonable investment decisions. As such,
the Securities and Exchange Commission (SEC) of Nigeria requires that the annual
reports of all quoted companies should include a corporate governance report.

Required:

Identify the contents of such corporate governance report. (5 Marks)

(Total 15 Marks)

QUESTION 6

a. The International Accounting Standards Board (IASB) aims at enhancing the


guidance available for assessing fair value in order to increase consistency and
comparability in fair value measurements and related disclosures. To this end, fair
value measurements are categorised into a three-level hierarchy, based on the type of
inputs to the valuation techniques used in IFRS 13. IFRS uses the terms principal or
most advantageous market.

Required:

i. What are the fair value hierarchies under IFRS 13? (3 Marks)
ii. Distinguish between the principal and most advantageous market and state how
price is determined in the principal markettaking into consideration transport
and transaction costs. (5 Marks)
b. It is important for entities to understand and properly classify their financial
instruments. This is because some financial instruments may have features of both
debt and equity, which can lead to inconsistency in reporting. To this end, financial
reporting standards provide guidance on the difference between financial
instruments classified as equity and liabilities.

With relevant examples, distinguish between liability and equity under IAS 32,
Financial Instruments Presentation. (7 Marks)
(Total 15 Marks)

QUESTION 7

a. Megida hopes to obtain contracts from both the private and public sectors following
the new government economic initiatives. The company's revenue had always been
accounted for in line with IAS 18 since the company had adopted IFRS. Some directors
of Megida understand that with the introduction of IFRS 15 Revenue from Contracts,
the way revenue from contract is being recognised may change. In particular one of
them that attended an IFRS training organised by the Institute of Chartered
Accountants of Nigeria (ICAN) heard about IFRS 15 and its five step model for
revenue recognition but did not understand.

Required:

Itemise and briefly discuss the FIVE step model approach to revenue recognition
under IFRS 15. (9 Marks)

b. The directors of Duranga Plc. has learnt that corporate reporting could be improved by
adopting the International Integrated Reporting Council's Framework for Integrated
Reporting. The directors believe that International Financial Reporting Standards,
which the company has recently adopted following the decision of the Federal
Executive Council, are already extensive and provide stakeholders with a
comprehensive understanding of its financial position and performance for the year.
They believe that with over 100 countries adopting IFRS, their financial statements
speak international financial reporting language and practice. In particular, statements
of cash flows which the company prepares in accordance with IAS 7 enables
stakeholders to assess the liquidity, solvency and financial adaptability of a business.
They are concerned that any additional disclosures could be excessive and obscure the
most useful information within a set of financial statements. This is against the
backdrop of a recent effort by IASB on excessive disclosures in financial statements.
They are therefore unsure as to
the rationale for the implementation of a separate, or combined, integrated report.

Required:
Discuss the extent to which statements of cash flow provide stakeholders with useful
information about an entity and whether this information would be improved by the
entity introducing an Integrated Report. (6 Marks)
(Total 15 Marks)
SOLUTION 1

(a) ADVICE ON THE FUNCTIONAL CURRENCY OF SOMA LTD

The financial controller should note the following for consideration:

According to IAS 21 - The Effects of Changes in Foreign Exchange Rates, the functional currency
is the currency of the primary economic environment in which the entity operates.
This has the implication that the functional currency should be determined at the
entity level. The primary economic environment in which an entity operates is
normally the one in which it primarily generates and expends cash.

Soma's functional currency should be determined by considering the following


factors in accordance with IAS 21:

(i) The currency:


• that mainly influences the determination of the sales prices for goods
and services (this will often be the currency in which sales prices for
its goods and services are denominated and settled); and

• of the country whose competitive forces and regulations mainly the


sales prices of its goods and services.

(ii) The currency that mainly influences labour, material and other costs of
providing goods or services (this will often be the currency in which such
costs are denominated and settled).

Other factors that may also provide evidence of an entity's functional currency
include:

• The currency in which funds from financing activities (i.e. issuing debt and
equity instruments) are generated.

• The currency in which receipts from operating activities are usually retained.

Additional factors to be considered are:

• The degree of autonomy of a foreign operation. An example is when the


foreign operation only sells goods imported from the reporting entity and
remits the proceeds to it.
• Whether transactions with the reporting entity are a high or a low proportion
of the foreign operation's activities.

• Whether cash flows from the activities of the foreign operation directly affect
the cash flows of the reporting entity and are readily available for remittance
to it.
• Whether cash flows from the activities of the foreign operation are sufficient
to service existing and normally expected debt obligations without funds
being made available by the reporting entity.

In Soma's case, sale prices are influenced by local demand and supply, and are
traded in defa.

Analysis of the revenue stream points to the defa as being the functional currency.
The cost analysis is variable as the expenses are influenced by the defa and the Naira.

Soma's management operates with a considerable degree of authority and autonomy


in carrying out the operations of Soma and is not dependent upon group companies
for financial support. Consideration is given to whether the foreign operation
generates sufficient functional cash flows to meet its cash needs, which in this case
Soma does, as it does not depend on the group for finance.

Therefore, the functional currency of Soma will be the defa as the revenue is clearly
influenced by the defa, and although the expenses are mixed, secondary factors point
to the fact that the functional currency is different to that of Rapuya Plc.

(b) Rapuya Plc


Statement of financial position as at April 30, 2017
N'm
PPE 1,472.00
Goodwill (wk 3&4) 94.80
Intangible asset -patent wk 5 6.00
Financial asset 84.00
1,656.80
Current assets 700.00
2.356.80
E quity and liabilities
Share capital 316.00
Retained earnings (wk 7) 727.91
Other components of equity (wk 8) 15.40
1,059.31
Non-controlling interest (wk 9) 369.49
1,428.80
Non-current liabilities 324.00
Current liabilities 604.00
2.356.80

11
Workings 1 .

Group structure
RAPUYA Plc

Puta Limited Soma Limited

2 . Translation of the financial statements Soma


Item Defas Rate N'm
Property, plant and equipment 760.00 2.50 304.00
Financial assets 100.00 2.50 40.00
Current assets 660.00 2.50 264.00
1,520.00 608.00

Share capital 400.00 3.00 13 3.33

Retained earnings:
- pre-acquisition 220.00 3.00 73.33
- Post acquisition (including exchange gain) 380.00 193.34
Non-current liabilities 320.00 2.5 0 128.00
Current liabilities 200.00 2.50 80.00
1.520.00 608.01
Goodwill-Puta N'm
Fair value Consideration transferred 226.00
Fair value of NCI 92.00
318.00
G et assets acquired
Total Fair of Net asset at acq 240.00
Add: Patent 8.00
Fair value of consideration transferred 248.00
Goodwill 70.00
Goodwill-Soma Defas Rate N'm
Consideration transferred (N184 x 3) 552.00 3.00 184.00
NCI at fair value 500.00 3.00 166.67
1,052.00 350.67
Less: Fair value of net asset at acq. 990.00 3.00 (330.00)
Goodwill May 1, 2016 62.00 3.00 20.67
Exchange gain 4.13
Goodwill April 30, 2017 62.00 2.50 24.80
Fair value Adjustments - Puta
Fair value of identifiable assets 240.00
Carrying amount net assets
Share capital 76.00
Retained earnings 98.00
Other components of equity (8 - 2) 6.00 180.00
Fair value land 60.00
Dr Land and Cr OCE with N60m

6. Fair value Adjustments - Soma Defas Rate N'm

Fair value of identifiable assets 990.00


Share capital 400
Retained earnings 220 620.00
Fair value - Adj. Land May 1, 2016 370.00 3.00 123.33
Fair value Adj. - Land April 30,2016 370.00 2.50 148
Exchange gain as the land is non-depreciable 24.67

7. Fetained earnings Rapuya Puta Soma

N'm N'm N'm


As per question 604.00 112.00 266.67
Amortisation patent (2.00)
Fair value Land Exchange Gain Soma 24.67
Pre-acq - per question (98.00)
Pre-acq - as translated (wk 2) (73.33)
Post acquisition retained profit 12.00 218.01
Puta share - 70% x 12 8.40
Soma share - 52% x 218.01 113.36
Share of Exchange gain on goodwill 52% x 4.13 2.15
727.91
8. Other Components of Equity (OCE) Rapuya Puta

N'm N'm
As per question 14.00 8.00
Pre-acquisition (6.00)

Group's share 70% x 2 1.40

15.40

9. Non controlling Interest Puta Soma NCI

N'm N'm
At acquisition (wk 3 and wk 4) 92.00 166.67
NCI Share of post acquisition profit
Puta (wk 7) 30% x 12 3.60
Soma (wk 7) 48% x 218.01 104.64
Post acq OCE 30% x 2 0.60
Exchange gain on goodwill (48% x 4.13) 1.98
Acquisition of 10% of NCI
96.20 273.29
369.49
EXAMINER'S REPORT

The question tests principles that determine functional currency in accordance with IAS 21
and preparation of consolidated statement of financial position in accordance with IFRS.

Candidates' understanding of Part (a) of the question was fair. However, all the candidates
attempted part (b) of the question and most of them performed above average.

The commonest pitfall of the candidates in part (a) was their inability to mention the factors
that determine the functionality of currency. That of part (b) was the inability to cal culate
goodwill arising on consolidation and the related noncontrolling interest (NCI).

Candidates are advised to pay more attention to this section of the syllabus for better
performance in future examination.

MARKING GUIDE

SOLUTION 1 MARKS MARKS


a. Identification of any four factors from IAS 21 @ 1
4
mark each
1 5
Conclusion/Advice based on above

b.

Correct postings in the consolidated statement of


financial Position @ y 2 mark each 5

c.
Correct entries in the working notes @ y4 mark
20 25
each
Total marks 30
SOLUTION 2

To: The Board of Directors, Lanke Plc

From: Finance Director

Date: May 16, 2017


Re: Performance Evaluation Report of Lanke Plc for the Two Years Ended
December 31, 2014 and 2015

The purpose of this report is to review the performance of the group and its subsidiaries for
the two years ended December 31, 2014 and 2015. The information
used for the report was extracted from the financial statements for the year ended
December 31, 2015. The calculated ratios are as shown in the appendix attached to this
report. Our comments and observations on the calculated ratios are as follows:

Returns On Capital Employed (ROCE)


The group's return on capital employed has improved from 20.64% in 2014 to 25.01% in 2015
while that of the company also improved significantly from 17.90% in 2014 to 28.15% in
2015. This increase could be attributable to improvements in both asset turnover and profit
margin. The company's calculated ratio is better than that of the group.

Profit Margin
The company's profit margin improved significantly from 12.63% in 2014 to 20.15% in 2015
indicating that costs control measures of the company is more efficient in 2015 than in 2014.
The group's profit margin increased slightly from 12.48% in
2014 to 14.52% in 2015. Company's profitability is better than that of the group for the two
years.

Asset Turnover
For the group, the asset turnover improved marginally from 1.65 times to 1.72 times between
year 2014 and year 2015. This shows that the group is more efficient in t he use of its assets to
generate revenue. However, for the company there is a marginal drop in asset turnover from
1.42 times to 1.40 times. The group could be said to utilize asset more efficiently than the
company.

Gearing
The gearing of the group has increased from 11.86% to 15.65% during the period under
review. The increase in gearing is attributable to the increase in the loan notes from N1,256m
to N1,985m in 2015. The gearing of the company also increased from 9.10% to 13.84% which
is also due to the significant increase in the loan notes from N783 million to N1,570 million
in 2015. The gearing of the company is lower than that of the group; however, both are in a
good leverage position.
The Interest Cover
The interest cover calculated confirms that the company and the group would be able to pay
interest costs as the operating profit covers the interest cost 4 times and
6 times in 2014 and 2015 respectively for the group; 3.59 times and 10.72 times in
2014 and 2015 respectively for the company.

The interest cover for the group was slightly better than that of the company in
2014 but the ratio for the company was tremendously improved in 2015 making the
company to be better in 2015 than the group. However, the interest covers for both the group
and the company is good as they are able to cover interest payment several times with their
earnings. The calculated ratios are not less than 3.0 times which is usually considered to be
low.

Differences between Company and Group Operating performance


The differences between the company and the group performance as analyzed above could
be attributed to the presence of the non-controlling interest as part of the shareholders of the
group. For instance, when calculating the capital employed for the group and the gearing
ratio, non-controlling interest will have to be included. This invariably affects the figures
calculated.

Conclusion
The overall performance of the group and the company for the periods was good but the
group's performance was weakened by the poor performance of other group members. The
reduction in the overall operating performance of the group when compared to that of the
company shows that some other members of the group must have operated at a loss.

Signed

Mallam Megida
Finance Director
APPENDIX

1. Returns on capital employed


= Profit before interest and tax 100 (PBIT) x
Capital employed

Capital employed = Share capital + Reserves + loan notes

Note: For a group, non-controlling interest is included in the capital


Employed

2015
Group N3.171 x 100 = 25.01%
N12,680

Company N3.194 x 100 = 28.15%


N11,347

2014
Group
N2.186 x 100 = 20.64%
N10,591
Company
N1.540 x 100 = 17.90%
N8,602

2. i. Profit margin

Net Profit Margin


Profit before interest and tax (PBIT) 100
x
Revenue

2015
Group = N3.171 x 100= 14.52%
N21,843

Company =N3.194 x 100 = 20.15%


N15,848

2014
Group
N2.186 x 100 = 12.48%
N17,519
Company
N1.540 x 100 = 12.63%
N12,198
ii. Gross Profit Margin = Gross Profit x 100
Revenue

2015
Group N6.167 x 100 = 28% N21,843

N4.683 x 100 = 29% N15,848


Company

2014
Group
N4.948 x 100 = 28%
N17,519
Company
N3.610 x 100 = 29%
N12,198

3. Asset turnover
= Revenue ____
Capital employed

= Revenue
Share capital + reserves + loan notes

2015
Group N21.843= 1.72 times
N12,680

Company N15.848 = 1.40 times


N11,347
2014
N17.519
Group = 1.65 times
N10,591

Company N12.198 = 1.42 times


N8,602

4 Gearing ratio
Gearing, also called leverage, measures the total loan note of a company as a
percentage of either:

(a) the equity capital in the company


Gearing ratio = ______ Loan Note x 100
Share capital + reserves (Equity)
OR

(b)
2015
Group = N1.985 x 100 = 15.65%
N1,985 + N10,695

Company = N1.570 x 100 = 13.84%


N1,570 + N9,777

2014
Group = N1.256 x 100 = 11.86%
N1,256 + N9,335

Company = N783 x 100 = 9.10%


N783 + N7,819

5. Interest cover
= Profit before interest and tax (PBIT)
Interest expenses

2015
Group = N3.171 = 6.35 times
N499

Company = N3.194 = 10.72 times


N298
2014
Group = N2.186 = 4.00 times
N546

Company = N1.540 = 3.59 times


N429

EXAMINER'S REPORT

The question tests candidates' knowledge of financial analysis relating to group of


companies with the aid of financial ratios.

About 92% of the candidates attempted the question. Most candidates have a good
understanding of the requirements of the question and performance was above average.

Commonest pitfall was the inability of the candidates to identify and apply appropriate
formulae.

Candidates are advised to pay more attention to ratio analysis for better performance in
future examinations.

MARKING GUIDE

Mark Mark
Report Format 1
Formulas (V2 mark for each formula) 3/
1
2

Calculations 5
Discussion on each ratio (1.5 X 5) 7/
1
2

Differences between Group and the Company 1


Conclusions 2
Total 20
SOLUTION 3

a. Impairment of non-financial assets and deferred tax assets

The following are the reasons why Financial Reporting Council of Nigeria
(FRCN) should focus on the impairment of non-financial assets and deferred
tax of listed companies:

• Particular attention should be paid to the valuation of goodwill and


intangible assets with indefinite life spans.

• The entity should focus oncertain specific areas including cash flow
projections, disclosure ofkey assumptions and judgements, and
appropriate disclosure of sensitivity analysis for material goodwill and
intangible assets with indefinite useful lives.

• IAS 36Impairment of Assets points out that greater weight should be


given to external evidence when determining the best estimate of cash flow
projections.

The following are the key areas which entities should focus on when
accounting for these items:

• A continuous period of slow economic growth could indicate to


regulators that non-financial assets will continue to generate lower than
expected cash flows, especially in those industries experiencing a downturn in
fortunes.

• In measuring value-in-use, cash flow projections should be based on


reasonable and supportable assumptions which represent the best estimate of
the range of future economic conditions.

• Each key assumption should be consistent with external sources of


information, or there should be disclosure of how these assumptions differ
from experience or external sources of information.
• Such an economic climate could result in the recognition of tax losses or the
existence of deductible temporary differences where perhaps impairments are
not yet deductible for tax purposes.
• The recognition of deferred tax assets requires detailed consideration
of thecarry forward of unused tax losses, whether future taxable
profitsexist, and the need for disclosing judgements made in these
circumstances.
• IAS 12 Income Taxes limits the recognition of a deferred tax asset to the extent
that it is probable that future taxable profits will be available against which
the deductible temporary difference can be utilised.
• IAS 12 states that the existence of unused tax losses is strong evidence that
future taxable profit might not be available. Therefore, recent losses make the
recognition of deferred tax assets conditional upon the existence of convincing
other evidence.
• The probability that future taxable profits will be available to utilise the
unused tax losses will need to be reviewed and if convincing evidence is
available, there should be disclosure of the amount of a deferred tax asset and
the nature of the evidence supporting its recognition.

It is particularly relevant to disclose the period used for the assessment of the
recovery of a deferred tax asset as well as the judgements made.

b. IAS 36 "Impairment of assets".

(i) Indications that an asset or group assets may have been impaired
External factors
• A significant decrease in the market value of an asset in excess of normal passage
of time.
• A significant adverse changes in the market or business in which the asset is
used.
• Adverse changes to the technology, economical or legal environment of the
business.
• An increase in market interest rates likely to affect the discount rate used in
calculating value in use.
• The carrying amount of the entity's net assets exceeds its market capitalisation.
• Where interest rates increase adversely affecting recoverable value
• Unexpected changes in government policies that could affect the business
adversely.

Internal factors
• Physical damage or obsolescence has occurred.
• Adverse changes to the method of use of the asset
• Where indications suggest that the economic performance of the asset will be
worse than expected.
• Where management intends to re-organise the entity.
• Where actual cash flows are less than estimated cash flows if an asset is appraised
in terms of value.
(ii) Test for impairment of an asset that does not generate independent cash flows

Where the oil company's asset does not generate independent cash flows i.e. cash
inflows that are largely independent of those from other assets, it will be assumed
that:
• The recoverable amount of that individual asset cannot be determined;
• If it is not possible to estimate the recoverable amount of an individual asset, an
entity shall determine the recoverable amount of the cash- generating unit to
which the asset belongs (the asset's cash-generating

unit);

• In such cases, value in use and, therefore, recoverable amount, can be


determined only for the asset's cash-generating unit, and impairment is then
tested at the level of cash generating unit for such asset;
• An impairment loss shall be recognised for a cash-generating unit if, and only if,
the recoverable amount of the unit is less than the carrying amount of the unit or
group of units;
• The impairment loss shall be allocated to reduce the carrying amount of the
assets of the unit or in the following order:
- first, to reduce the carrying amount of any goodwill allocated to the
cash-generating unit, and
- then, to the other assets of the unit; pro rata on the basis of the carrying
amount of each asset in the unit.
These reductions in carrying amounts shall be treated as impairment losses on
individual assets and recognised; and
• In allocating an impairment loss in accordance with IAS 36, an
entity shall not reduce the carrying amount of an asset below the highest
of:

its fair value less costs to sell (if determinable);


its value in use (if determinable); and
zero.

(c)
Impairment is calculated as the higher of
Carrying amount Recoverable amount 960
Impairment 720
240
Allocation of the impairment
Asset Carrying Impairment Carrying
amount amount
after
impairment
N'million N'million N'million
Goodwill 160 160 —

Patent 320 32 288


Property, plant and Equipment 480 48 432

MARKS MARKS
a. Impairment of non-financial assets and deferred tax asset

(any 7 points @1 mark each) 7

b. Identify 2 external and 2 internal indicators

i. Two external indicators


(any 4 points @ %% mark each) 1

Two internal indicators


(any 4 points @ %% mark each) 1
2

ii. Test for impairment of an asset that does not


generate independent cash flows
(6 points at 1 mark each) 6

c. Calculation of the value of the assets held after


the impairment test 5
13
Total 20
SOLUTION 4

REPORT

From: Mr. ABC

To: The Directors


Dango Plc Date:
May 16, 2017

Subject: Relevant accounting treatments of transactions

The purpose of this report is to explain the relevant accounting treatments on mentioned
transactions in accordance to the applicable accounting standards and International Financial
Reporting Standards (IFRS) such as IAS 20, IAS 38, IFRS 3 and IFRS 15. The explanations are as
follows:
(a) Revenue recognition: In accordance with IFRS 15, Revenue from Contracts with Customers,
the core principle is delivered in a five step model framework viz:

Applying the above to the Dango case, the performance obligation and transfer of control
will be deemed to take place on delivery of the rice of the government of Guyama. This is
when the customer takes on the risks and rewards and accepts the goods.
Therefore, in the year ended September 30, 2016, revenue should be recognised for the April
1, 2016 (N250million) and July 1, 2016 deliveries (N380million) making total revenue of
N630million.
Based on the above, revenue will not be recognised for the October 1, 2016 delivery as
it does not take place until after the year end.

The significant costs incurred by Dango on customs for the first batch of delivery
should be expensed in line with the matching concept.

Costs incurred in relation to the October 1, 2016 delivery should be held as work in
progress in the statement of financial position until revenue is recognized in the year
ended September 30, 2017.

(b) In compliance with IAS 38, the license awarded by the Federal Government to Dango
Plc on October 1, 2010, which was recognized on that date at its fair value i.e.
N196million, should be amortized over its useful life of 12 years. It should be noted
that fair value implies the price that would be received to sell an asset or paid to
transfer or settle a liability in an orderly transaction between market participants at
the measurement date.

Goodwill of N240million was created as a result of business combination (purchase of


division). The goodwill created in this manner should be recognized in the financial
statements. Under IFRS 3 "Business combination" goodwill is the excess of the
purchase consideration over the fair value of the acquiree's identifiable net assets.
Once recognized, goodwill is held indefinitely and will not be amortized but will be
subjected to annual impairment review.

Also, according to IAS 38, all internally generated goodwill should not be recognized
in the financial statements.

A cash generating unit according to IAS 38 is the smallest identifiable unit that
generates independent cash-flows. There is no requirement by IAS 38 that it must
constitute a minimum revenue threshold before such classification as a cash
generating unit can be made.

(c) Valuation of Inventory as at September 30, 2016

According to IAS 2 "Inventory", inventory should be valued or measured at the lower


of cost and net realisable value. Based on IAS 2 "Inventory", cost of inventory
includes the purchase price and all directly attributable costs incurred in bringing the
item of inventory to its current location and saleable condition.

IAS 2 states that "the cost of inventories shall comprise all cost of purchase, cost of
conversion and other costs incurred in bringing the inventories to their present
location and condition".
Calculation of cost of inventory N
Purchase price (1,000 units x N1,200) 1,200,000
Less trade discount (5% x N1,200,000) (60,000)
Net purchase price Import duties (N60 x 1,140,000
1,000 units) Delivery costs 60,000
5,000
1.205.000

Note: It should be noted that purchase price excludes any settlement


discounts and is the cost after deduction of trade discount. This implies that
purchase cost of inventory involves deduction of trade discount, but
settlement discount is irrelevant.

The cash discount of 2% represents discount received and should not affect
inventory valuation but should be recognised as an income in the statement
of profit or loss.

Conclusion

The provisions of IFRS on the correct treatment of revenue for contracts wit h customers,
the recognition of internally generated goodwill and valuation of inventory has been
dealt with in the above explanation for Dango Plc. to apply in the financial statements.
MARKING GUIDE

MARKS MARKS
a. Core Principle of Revenue recognition
Main points listed on IFRS 15
(any 3@ 1 mark each) 3
(any five @ 1 mark each) 5 8

b. Explanation relating to Federal Government


license (any three parts @ 1 mark each) 3
Goodwill created under business combination 1 4

c. Explanation on IAS 2 "Inventory" 1%


Computation of Purchase cost of inventory 5
Explanation regarding trade discount and
settlement discount 1% 8
Total 20
SOLUTION 5

a. Identification of reports that are voluntarily disclosed in annual reports of


Nigerian companies:
• Statement of corporate governance;
• Environmental and social report;
• Report on corporate and social responsibility/Sustainability Report;
• Five-year Financial Summary;
• Key Performance Indicator (KPI);
• Value added statement;
• Chairman's report; and
• Statement of Director's responsibilities.

b. A company would disclose information not required by law in its annual report
for the following reasons:
• As a marketing and public relations tool;
• It enhances the level of transparency upon which the financial information is
produced;
• It helps in projecting an entity as a better managed company;
• It enhances the demand for an entity's shares as investors tend to allocate
more capital to a more transparent company;
• To build goodwill or enhance reputation; and
• As a competitive advantage strategy.

Limitations:

• The company can decide what to include in the report and what to leave
out; the reality you see is the reality the company wants you to see;
• Since such voluntary disclosures are not regulated by laws or
standards, such disclosures tend to lack uniformity and
standardization;
• The information is often presented in a very positive form, as public
relations for investors, and might not be entirely reliable;
• Information overload; and
• Window dressing (Creative Accounting).

c. The contents of corporate governance report are:


• Compliance with laws and regulations;
• The Board Composition;
• Role of the Board;
• Directors' Interest in Contracts;
• Board Meetings;
• Board changes;
• Committees of the Board;
• Roles and responsibilities of the Board committees and the discharge of
such;
• Control Environment;
• Induction and Training;
• Performance Evaluation process;
• Code of Business Conduct and Code of Governance for Directors;
• Human resource policies, internal management structure, relations with
employees, employee share ownership schemes and other work place
development initiatives;
• Statement of Company's Risk Management Policies and Practices; and
• Dealings in Securities Code.

MARKING GUIDE

MARKS MARKS

a. i. Identification of Voluntarily disclosed report


(any three @ 1 mark each) 3

b. Reasons for disclosing information


i. Not required by law (any
four @ 1 mark each) 4
ii. Limitation of such disclosure
(any three @ 1 Mark each) 3
7

c. General requirements of contents of corporate


governance report (any five @ 1 mark each)
5

Total 15
SOLUTION 6

(i) Fair Value Hierarchies


The hierarchy categorizes the inputs used in valuation techniques into three levels. The
three levels are as contained below:

- Level 1 input
Level 1 inputs are quoted prices in active markets for identical assets or liabilities
that the entity can access at the measurement date.

A quoted market price in an active market provides the most reliable evidence of
fair value and is used without adjustment to measure fair value whenever
available.

- Level 2 inputs
Level 2 inputs are inputs other than quoted market prices included within Level 1
that are observable for the asset or liability, either directly or indirectly.

Level 2 inputs will include:


i. quoted prices for similar assets or liabilities in active markets;
ii. quoted prices for identical or similar assets or liabilities in markets that are
not active; and
iii. inputs other than quoted prices that are observable for the asset or liability.

- Level 3 inputs
Level 3 inputs are unobservable inputs for the asset or liability.

Unobservable inputs are used to measure fair value to the extent that relevant
observable inputs are not available, thereby allowing for situations in which there
is little, if any, market activity for the asset or liability at the measurement date

(ii) Principal market


The market with the greatest volume and level of activity for the asset or liability
Most advantageous market
It is the market that maximizes the amount that would be received to sell the asset or
minimizes the amount that would be paid to transfer the liability, after taking into
account transaction costs and transport costs.

In determining fair value at the principal market the following must be borne in
mind
- Transaction cost - This cost is not considered in principal market fair value
determination.

- Transport cost - This cost must be considered in arriving at the principal market
fair value.

b. A financial liability is any liability that is:

(a) a contractual obligation :


i) to deliver cash or another financial asset to another entity; or
ii) to exchange financial assets or financial liabilities with another entity
under conditions that are potentially unfavourable to the entity; or

(b) a contract that will or may be settled in the entity's own equity
instruments and is non derivative.

Debt is a typical example of a liability. An instrument will be classified as debt if:

• Redemption is at the option of the instrument holder;

• There is limited life to the instrument;

• Redemption is triggered by a future uncertain event which is beyond the control


of both the holder and the issuer; and

• Dividends are non-discretionary.

An equity instrument is any contract that evidences a residual interest in the assets of
an entity after deducting all of its liabilities.

An instrument will be classified as equity if:

• The instrument is non-redeemable; and


• Dividends are discretionary.
EXAMINER'S REPORT

The question tests candidates' knowledge of IFRS 13 and IAS 32. About 50% of the
candidates that sat for the examination attempted the question and their performance was
below average.

Commonest pitfall was candidates' inability to understand the fair value hierarchies level of
inputs as identified by the standards.

Candidates are advised to concentrate more on the ICAN Study Text for this level of
examination for better performance in future.

MARKING GUIDE

MARKS MARKS
a. i. 3 hierarchies (1 mark each) 3

i. (a) Distinction between principal market and


most advantageous market
(1% mark each) 3

(b) Price determination in Principal Market


- Transport cost 1
1 5
- Transaction cost
8

b. I mark each for seven points mentioned out of


eleven points above Total 7
15
SOLUTION 7
The five steps model approach systems under IFRS 15 are:
a. Step 1 - Identify the contract with the customer
A contract can be written, verbal or implied. It is within the scope of IFRS 15 when:
(i) Both parties have approved it and are committed to it;
(ii) Each party's rights regarding the goods and services to be transferred can be
identified;
(iii) The payment terms can be identified;
(iv) The contract has commercial substance; and
(v) It is probable that the consideration will be received.

Step 2 - Identify the separate performance obligations


A performance obligation is a promise in a contract with a customer to
transfer to the customer either:
(i) a good or service (or a bundle of goods or services) that is distinct; or
(ii) a series of distinct goods or services that are substantially the same
and that have the same pattern of transfer to the customer;
(iii) A good or service, which has been delivered, may not be distinct if it
cannot be used without another good or service which has not yet
been delivered; and
(iv) Similarly, goods or services which are not distinct should be combined with
other goods or services until the entity identifies a bundle of goods or services
which is distinct.

Step 3: Determine the transaction price


(i) An entity must consider the terms of the contract and its customary
practices in determining the transaction price
(ii) The transaction price assumes transfers to the customer as promised
in accordance with the existing contract and that the contract will not be
cancelled, renewed or modified.
(iii) The transaction price is not adjusted for effects of the customer's credit
risk.
(iv) For this purpose, an entity shall consider the effects of all the following:
❖ variable consideration;
❖ the constraint on variable consideration;
❖ time value of money;
❖ non-cash consideration;
❖ consideration payable to the customer.

Step 4: Allocate the transaction price to the performance obligations


(i) The entity allocates a contract's transaction price to each separate
performance obligation within that contract on a relative stand-alone selling
price basis at contract inception.

(ii) A stand-alone selling price is the price at which an entity would sell a
promised good or service separately to a customer.

(iii) IFRS 15 suggests that any of the following three methods will be suitable
for estimating the stand-alone selling price:
■ adjusted market assessment approach;
■ expected cost plus margin approach; and
■ residual approach.

Step 5: Recognizerevenue when oras an entity satisfies performance


obligations
(i) Revenue is recognised when or as the promised goods or services are
transferred to a customer.
(ii) A transfer occurs when the customer obtains control of the good or
service.
(iii) Indicators of control include:
■ The entity has a present right to payment for the asset;
■ The customer has legal title;
■ The customer has physical possession (exceptions for
bill and hold, consignment sales and repos);
■ The customer has the significant risks and rewards of
ownership of the asset; and
■ The customer has accepted the asset.
(iv) Benefit of an assets are the potential cash flows obtainable directly or
indirectly from the assets.
b.
i. The extent to which statements of cash flow provide stakeholders with
useful information about an entity

• Statements of cash flows provide valuable information to stakeholders


on the financial adaptability of an entity.

• Cash flows are objective and verifiable and so are more easily
understood than profits.

• Profits can be manipulated through the use of judgement or choice of a


particular accounting policy. Operating cash flows are therefore useful
at highlighting the differences between cash and profits.

• The cash generated from operations is a useful indication of the quality


of the profits generated by a business.

• Good quality profits will generate cash and increase the financial
adaptability of an entity. Cash flow information will also have some
predictive value. It may assist stakeholders in making judgements on
the amount, timing and degree of certainty on future cash flows.

• Cash flow is useful as an input in the valuation model.

• The classification of cash flows can be manipulated between operating,


investing and financing activities. It is important therefore not to
examine the cash flow information in isolation.
• It is only through an analysis of the statement of financial position,
statement of comprehensive income and notes, together with cash
flow, that a more comprehensive picture of the entity's position and
performance develops.

• It is true that International Financial Reporting Standards are extensive


and their required disclosures very comprehensive.
This has led to criticism that the usefulness may be limited where the
most relevant information is obscured by immaterial disclosures.
ii. Extent to which statement of cash flows provides stakeholders with
useful information and whether this information would be improved by the
entity introducing an Integrated Report.
- Integrated Report (IR) presents a company's performance in
terms of both financial and other value relevant information.
• IR reflects the broad and longer-term consequences of the decisions
organizations make, based on a wide range of factors, in order to create
and sustain value.
• IR enables an organization to communicate in a clear, articulate way
how it is drawing on all the resources and relationships it utilizes to
create and preserve value in the short, medium and long-term.
• IR system helps investors to manage risks and allocate resources most
efficiently.
• An integrated reporting system would increase disclosure as well as
imposing additional time and cost constraints on the reporting entity.
• Integrated reporting will provide stakeholders with valuable
information which would not be immediately accessible from an entity's
financial statements especially the cash flow statement.
• Financial statements are based on historical information and may lack
predictive value.
• Financial Statements are essential in corporate reporting, particularly for
compliance purposes but do not provide meaningful information
regarding business value.
• IR is a strategic direction which is lacking from a traditional set of
financial statements and will be valuable to stakeholders to make a
more informed assessment of the organisation and its prospects.

Judging from the benefits of IR as enumerated above, it is very clear that IR


cannot obscure the relevance of statement of cash flows in a comprehensive
reporting framework but it will rather enhance its importance.

EXAMINER'S REPORT

The question tests the provision of IFRS 15. Part (a) of the question tests provision of IFRS 15
with regards to revenue from contract which hitherto have been accounted for under IAS 18.
Part (b) of the question tests candidates' knowledge of the usefulness of statements of cash
flow in a Financial Statements and the probable improvement the introduction of Integrated
Report could have on the usefulness of statement of cash flow.
About 70% of the candidates attempted the question and performance was very poor.

The commonest pitfall was the inability of the candidates to relate integrated reporting to
cash flow statements.

Candidates are advised to ensure a good coverage of the syllabus directing attention to
provisions of various standards (IFRS) in the ICAN Study texts for better performan ce in
future examinations.

MARKING GUIDE

MARKS MARKS
THE INSTITUTE OF CHARTERED ACCOUNTANTS OF NIGERIA
PROFESSIONAL LEVEL EXAMINATION - NOVEMBER 2017
CORPORATE REPORTING
Time Allowed: 3V4 hours (including 15 minutes reading time)

INSTRUCTION: YOU ARE REQUIRED TO ANSWER FIVE OUT OF SEVEN


QUESTIONS IN THIS PAPER

SECTION A: COMPULSORY QUESTION (30 MARKS)

QUESTION 1
The following are the financial statements of Papa, Tata and Chebe, all Plcs.as at March 31, 2017:

Papa Tata Chebe


N'm Nm Nm
Assets:
Tangible non-current assets 1,280 440 280
Investment in Tata 413 - -
Investment in Chebe 60 - -
Current assets 531 190 130
Total assets 2.284 630 410
Equity and liabilities:
Share capital of N1 each 800 240 200
Share premium 150 20 30
Revaluation reserve 90 - -
Retained earnings 390 210 94
Total equity 1,430 470 324
Non-current liabilities 640 30 16
Current liabilities 214 130 70
Total equity and liabilities 2.284 630 410
Papa acquired the following shareholdings in Tata and Chebe
Date of Holding Fair value of Purchase acquisition
acquired net assets consideration
N'm N'm

Tata April 1, 2014 30% 325 120


April 1, 2016 50%460 260

Chebe April 1, 2016 25% 200 60

You are also provided with the following information, which will be relevant to the
consolidated financial statements of Papa Plc.
(i) None of the companies has issued any additional share capital since April 1, 2014.
(ii) The financial statements of Papa have not yet been adjusted for the gain or loss arising on
gaining control of Tata.
(iii) At April 1, 2014, the carrying value of the net assets of Tata was the same as their fair value
of N325million.
(iv) Papa Plc. wishes to use the full fair value method of accounting for the acquisition of Tata,
and at April 1, 2016 the estimated value of goodwill attributable to non-controlling interests
was N3million. The estimated fair value of the initial investment in 30% of the shares of Tata
was N150million at March 31, 2017.
(v) Included in the tangible non-current assets of Tata is land, valued at cost which on March 31,
2017 had a fair value of N25million in excess of its carrying value. There has been no
subsequent significant change in that value.
(vi) At April 1, 2016 the fair value of Chebe's land was N16million in excess of its carrying value.
There has been no subsequent significant change in that value.
(vii) Goodwill arising on acquisition is tested for impairment at each year end. At March 31, 2017
an impairment loss of N15million was recognised for Tata.
(viii) There has been no impairment of the investment in Chebe.

Required:
Prepare the consolidated statement of financial position of Papa group as at March
31, 2017. (Total 30 Marks)
SECTION B: YOU ARE REQUIRED TO ANSWER ANY TWO OUT OF THREE QUESTIONS IN THIS
SECTION (40 MARKS)

QUESTION 2
The summarised comparative financial statements of Odua Plc. for the year ended December 31,
2016 and 2015 are as follows:

Statements of profit or loss and other comprehensive income for the year ended December 31

2016 2015
(Nm) (Nm)
Revenue 550 400
Cost of sales (400) (200)
Gross profit 150 200
Operating costs (72) (60)
Operating profit 78 140
Investment income - (Note 2)
(Loss)/Gain on revaluation of investments held at fair value Profit/Loss (10) 20
Finance costs (10) (6)
Profit before taxation 58 154
Income tax expense (8) (30)
Profit for the year 50 124
Other comprehensive income (Amounts that will not be
reclassified to profit or loss)
Revaluation losses on property plant & equipment (90) -_
(Loss)/total comprehensive income for the year (401 124

Statement of Financial Position as at 31 December


Assets N N N N
Non-current assets:
Property, plant and equipment 430 490
Investments at fair value through profit or loss 70 80
500 570
Current assets:
Inventory 80 38
Trade receivables 104 56
Bank L 184 20 114
Total assets 684 684
Equity and liabilities
Equity:
Equity shares of N0.50 each 240 240
Revaluation reserve 2 110
0 130
Retained earnings 180
440 480
Non-current liabilities: Bank
loan Current liabilities: 100 100
Trade payables Bank
overdraft Current tax payable 10 78
Total equity and liabilities 0
4
04 144 26 104
684 684

The following additional information is relevant:

(i) The Managing Director is of the view that the company has retained its book value and
therefore has not suffered any deterioration in performance from
2015 to 2016. This statement was made in the process of appraising the new strategy
introduced by the company which he believes has not failed.

(ii) Odua Plc. has traditionally been very profitable, but in recent years has been finding it
difficult to keep up its sales level due to the effects of online shopping. Basically, it finds out
that more customers are buying directly online from suppliers and cutting out the
middleman, which includes Odua as a wholesaler. To cope with this situation, on January 1,
2016 Odua launched a

strategy of cutting its prices in the hope that this would generate additional sales volume
and profits.

(iii) To support the new strategy and allow faster movement of goods, a new product movement
and control system was commissioned and installed on January 1, 2016 at a cost of
N40million. This is being depreciated over a 5 year useful economic life. The old system was
disposed off for zero consideration on the same date, but had been carried at N15million at
the date of disposal. The loss was taken to cost of sales, as well as the depreciation. No
other non-current assets were acquired or disposed in either of the two years.

(iv) The share price has declined from N2.80 per share on December 31, 2015 to N1.60 per share
on December 31, 2016 and the Managing Director is unable to attribute reasons for the
decline in the share prices.

You are required to:


Analyse the financial statement of Odua Plc. by evaluating and interpreting the underlisted
ratios under relevant headings of profitability, efficiency, short term liquidity and long term
solvency and stability as well as stock market performance for each financial year taking into
consideration the additional information in (i) - (iv) above:

Gross Margin Inventory Days


Net Margin Receivables Days
ROCE ROE Payables Days
Current Ratio Earnings per share
Acid Test Price earnings ratio
Interest cover Gearing
(Total 20 Marks)

QUESTION 3
Funda Plc. is a listed entity based in Nigeria, a utility service company and is involved in the supply
of water, electricity and cable services to domestic and industrial consumers. The directors of Funda
Plc. have prepared draft financial statements for the year ended June 30, 2017 and have stated that
these have been

prepared in accordance with IFRS. The financial statements are to be used in support of a loan
application. Funda Plc. employees own 5% of the ordinary shares of the company. The employees'
representatives have expressed concern about the loan application and are seeking advice on
certain of the policies Funda Plc. has used in drafting the financial information.

The draft income statement for the year ended June 30, 2017 is:
N'm
Revenue 410.0
Cost of sales (275.0)
Gross profit 135.0
Other operating costs (65.0)
Profit before taxation 70.0

The employees' representatives require an explanation on the following:

Sale of water filters


Funda Plc. manufactures industrial water filters. On December 31, 2016 it sold a batch of 30 filters
to a steel maker. Funda Plc. faced stiff competition to secure this
order. As part of its marketing offer, Funda Pic. provided a volume discount of 20% from the market
price of the filters. A further term of the contract was that Funda Pic. has granted the steel maker a
put option on the filters. The option entitles the steel company to require Funda Plc. to repurchase
the filters after 6 years for 35% of the price paid on the initial transaction. The expected economic
life of the filters is 10 years. At the date of the sale transaction, the repurchase option was expected
to be „in the money'. The filters normally sell for N625,000 each at which price Funda Plc. achieves a
profit margin of 36%. The steel company has paid for the filters in full and Funda Plc. has recognised
the revenue and costs of manufacture in profit or loss.
Connection fees
Funda Plc. supplies electricity to domestic consumers and charges customers a connection fee
before it will connect the customer to the electricity supply network. This connection fee is
refundable to the customer if and when the customer decides that the electricity supply is no longer
required. There is no minimum notice period that the customer must give Funda Plc. of its request
to disconnect. Funda Plc. may deduct the cost of disconnecting the customer from the amount
refundable. Funda Plc. has many long-established customers that are not expected to request
disconnection in the foreseeable future. Fees charged in the year ending 30 June,
2017 were N5m (2016: N4m). These amounts have been credited to profit or loss.

Activation fees
Funda Plc. supplies a range of digital cable services including telephone, internet access and cable
TV. The company charges its customers a one-time activation fee to enable them to get a number
and access to the cable network. This fee is non- refundable. N7m in such fees have been credited to
profit or loss in 2017 (2016: N4m).
Deposits
Funda Plc. supplies domestic electrical goods to customers. Customers must pay a deposit of 25% of
the purchase price when placing an order. The remaining 75% is payable on delivery of the
appliance. Funda Plc. retains the deposit if the customer cancels the order. If Funda Plc. is unable to
fulfil the order the deposit is repayable in full. Funda Plc. recognises the deposits as revenue in profit
or loss when the order is placed. Revenue includes N10m in respect of these deposits relating to
orders in the year. 90% of these orders have been filled.
Required
Prepare a report which explains the suitability of Funda Plc. accounting policies for the transactions
listed above and recommend the correct treatment where appropriate. (Total20 Marks)

QUESTION 4

Enugun Industries Limited


Atikun has recently been appointed as Financial Controller to Enugun 1ndustries Limited. Until a
month ago, Enugun Industries had a Finance Director, who resigned suddenly, due to ill health. Since
Atikun joined the company, he has learned that his resignation was related to stress caused by a
series of disagreements with the Managing Director about the performance of the business. The
directors have not yet appointed a replacement.
It is now March 2015 and you have been asked to finalise the financial statements for the year
ended December 31, 2014. The draft statement of profit or loss extract and statement of financial
position are shown below:

Draft statement of profit or loss for the year ended December 31, 2014

N'000

Profit before tax 2,500

Draft statement of financial position as at December 31, 2014


N'000
Property, plant and equipment 12,000
Current assets 3,500
Total assets 15,500
Share capital 2,000
Retained earnings 6,000
Equity 8,000
Non-current liabilities 5,000
Current liabilities 2,500
Total equity and liabilities 15,500

During the year ended December 31, 2014 Enugun Industries entered into the following
transactions.
(i) Just before the year-end Enugun Industries signed a contract to deliver
consultancy services for a period of 2 years at a fee of N500,000 per annum. The
full amount of this fee has been paid in advance and is non-refundable.
(ii) Enugun Industries has constructed a new factory. The construction has been
financed from the pool of existing borrowings. Land at a cost of N1.8million was
acquired on February 1, 2014 and construction began on June 1, 2014.
Construction was completed on September 30, 2014 at an additional cost of
N2.7million. Although the factory was usable from that date, full production did
not commence until December 1, 2014. Throughout the year, the company's
average borrowings were as follows:

Amount Annual
interest rate
N %
Bank overdraft 1,000,000 9.75
Bank loan 1,750,000 10
Loan notes 2,500,000 8

An amount of N450,000 has been included in property, plant and equipment in


respect of borrowing costs relating to the construction of the factory. The useful
life of the factory has been estimated at 20 years. No depreciation has been
charged for the year. The reason for this is that the factory has only been in use
for one month and that the depreciation charge would be immaterial.

(iii) A blast furnace with a carrying amount at January 1, 2014 of N3.5 million has
been depreciated in the draft financial statements on the basis of the remaining
life of 20 years. In December 2014, the directors carried out a review of the
useful lives of various significant items of plant and machinery, including the blast
furnace and came to the conclusion that the useful life of the furnace was 20
years at December 31, 2014. The reasoning behind this judgement was that the
lining of the furnace had been replaced in the last week of December 2014 at a
cost of N1.4 million. Provided that the lining is replaced every five years, the life
of the furnace can be extended accordingly. You have found a report,
commissioned by the previous Finance Director and prepared by a firm of asset
valuation specialists, which assesses the remaining useful life of the main
structure of the furnace at January 1, 2014 at 15 years and the lining of the
furnace at 5 years. You have also found evidence that the Managing Director has
seen this report.
Atikun has had a conversation with the Managing Director who told him, "We
need to make the figures look as good as possible so 1 hope you're not going to
start being difficult. The consultancy fee is non-refundable so there's no reason
why we can't include it in full. I think we should look at our depreciation policies.
We're writing off our assets over far too short a period. As you know, we're
planning to go for a stock market listing in the near future and being prudent
and playing safe won't help us do that. It won't help your future with this
company either."
Required:

a. Explain the required IFRS accounting treatment of these issues,


preparing relevant calculations where appropriate. (16 Marks)

b. Discuss the ethical issues arising from your review of the draft financial
statements and the actions that you should consider.
(4 Marks)
(Total 20 Marks)

SECTION C: YOU ARE REQUIRED TO ANSWER ANY TWO OUT OF THREE


QUESTIONS IN THIS SECTION (30 MARKS)

QUESTION 5

Tinubun Plc., a public limited company. operates two pension plans.

Pension Plan 1

The terms of the plan are as follows:

(i) Employees contribute 6% of their salaries to the plan


(ii) Tinubun Plc. contributes, currently, the same amount to the plan for the
benefit of the employees
(iii) On retirement, employees are guaranteed a pension which is based upon
the number of years' service with the company and their final salary.
(iv) This plan was closed to new entrants from October 31, 2016 but which was open to
future service accrual for the employees already in the scheme.

The following details relate to the plan in the year to October 31, 2017:
N'm

Present value of obligation at November 1, 2016 200


Present value of obligation at October 31, 2017 240
Fair value of plan assets at November 1, 2016 190
Fair value of plan assets at October 31, 2017 225
Current service cost 20
Pension benefits paid 19
Total contributions paid to the scheme for year to October 31, 2017 17
Actuarial gains and losses are recognised in the Statement of Other Comprehensive Income.

Pension Plan 2
Under the terms of the plan, Tinubun Plc. does not guarantee any return on the contributions paid
into the fund. The company's legal and constructive obligation is limited to the amount that is
contributed to the fund. The following details relate to this scheme
N'm

Fair value of plan assets at October 31, 2017 21


Contributions paid by company for year to October 31, 2017 10
Contributions paid by employees for year to October 31, 2017 10

The discount rates for the two plans are:


October 31, 2017 November 1, 2016
Discount rate 6% 5%

Required:
a. Explain the nature of and differences between a defined contribution plan
and a defined benefit plan with specific reference to the company's two schemes. (7 Marks)
b. Show the accounting treatments for the two Tinubun Plc. pension plans for the year ended
October 31, 2017 under IAS 19 'Employee Benefits'.
(8 Marks) (Total 15 Marks)

QUESTION 6

Eko Exports Limited


The following information pertains to Eko Exports Limited (EEL) for the financial
year ended December 31, 2016:
(i) A customer who owed N1million was declared bankrupt after his warehouse was destroyed
by fire on February 10, 2017. It is expected that the customer would be able to recover 50% of the
loss from the insurance company.

(ii) An employee of EEL forged the signatures of directors and made cash withdrawals of
N7.5million from the bank. Of these, N1.5million were withdrawn before December 31, 2016.
Investigations revealed that an employee of the bank was also involved and therefore, under a
settlement arrangement, the bank paid 60% of the amount to EEL on January 27, 2017.
(iii) EEL has filed a claim against one of its vendors for supplying defective goods. EEL's legal
consultant is confident that damages of N1million would be paid to EEL. The supplier
has already reimbursed the actual cost of the defective goods.
(iv) A suit for infringement of patents, seeking damages of N2million, was filed by a third
party. EEL's legal consultant is of the opinion that an unfavourable outcome is most
likely. On the basis of past experience, he has advised that there is 60% probability that
the amount of damages would be N1million and 40% likelihood that the amount would
be N1.5 million.
Required:
Advise EEL about the amount of provision that should be incorporated and the
disclosures that are required to be made in the financial statements for the year ended
December 31, 2016.

(Total 15 Marks)

QUESTION 7

Corporate reporting by listed companies in Nigeria is evidenced by the annual


report defined as a comprehensive report on a company's activities throughout the
preceding year.
The directors of Mugono Plc would want to know the content of an annual report.
They are not sure of the difference between mandatory and voluntary disclosures in
the annual report of the company.

Required:

Write a report to the directors of Mugono Plc

a. Highlighting the componentsincluded in annual report. (4 Marks)


b. Showing the following:
i. THREE advantages of mandatory disclosures in annual report;
(3 Marks)
ii. FOUR advantages of voluntary disclosures in annual report.
(4 Marks)

iii. TWO limitations of information provided on a voluntary basis.


(2 Marks)
(For clarity in presentation) (2 Marks)
(Total 15 Marks)
SOLUTION 1

PAPA PLC
Consolidated statement of financial position as at 31 March 2017
Nm
Assets
Tangible non-current assets (W4) 1,745
Intangible non-current assets - goodwill 45 - 15 (W3) 30
Investment in associate (W5) 95
Current assets (531 + 190) 721
Total Assets 2.591
Equity and liabilities:
Share capital of N1 each 800
Share premium 150
Revaluation reserve 90
Retained earnings (W6) 438
1,478
Non-controlling interest (W7) 99
1,577
Non-current liabilities (640 + 30) 670
Current liabilities (214 + 130) 344
Total Equity and liabilities 2,591

Workings
(W1) Tata retained profits

Nm
Fair value of net assets at 1 April 2016 460
Fair value adjustment for land (25)
Carrying value of net assets 435
Share capital 240
Share premium _20 (260)
Therefore retained earnings at 1 April 2016 175

Nm

Carrying value of net assets at 1 April 2014 325


Share capital 240
Share premium .20 (260)
Therefore retained earnings at 1 April 2014 65
Nm Nm

(W2) Gain or loss on acquiring control of Tata

Fair value of initial investment in Tata Initial 150


cost of investment 120
Share of retained earnings 1 April 2014 - 1 April 2016 (= 30% x 33
(175 - 65) - see W1 Carrying value of investment in Tata as
associate Loss recognised on gaining control of Tata 153

This loss has not yet been recognised in the individual financial statements of Papa Plc; it
must therefore be included in the calculation of group reserves (see Working 8).
(W3) Goodwill in Tata at acquisition

Nm
Fair value of initial investment at acquisition Cost 15
of additional shares Total cost 0
Fair value of net assets acquired (80% x 460) 26
41
0
Goodwill at acquisition attributable to Papa 0
36
Goodwill attributable to NCI Total goodwill at 84
acquisition date 2
3

Goodwill in statement of financial position: There has been impairment of4 N15 million in
goodwill. This is apportioned between the interests of the equity owners of 5Papa and NCI in
the ratio 80:20.
Impairment of goodwill attributable to parent = N15m x 80% = N12 million Impairment of
goodwill attributable to NCI = N15m x 20% = N3 million.

(W4) Tangible non-current assets

Nm
Papa 1,28
Tata 0
Fair value adjustment 440
25
1,74
5
(W5) Net Assets of the Subsidiary - Chebe

Reporting Acquisition Post


DateDate Acquisition
N'M N'M N'M
Share capital of N1 200 200 -
Share Premium 30 30
Retained Earnings 94 94
Fair vale adjustment 16 16
324 216 140

Investment in Chebe (Associate)


N'm
Costs 60
Share of post acquisition
retained profit (25% x 140) 35

Investment 95
OR N'm

Share of Chebe net assets (24% x 324) 81 Fair


value adjustment (25% x 16) 4
Goodwill (60 - (200 x 25%) 10

95

(W6) Consolidated retained earnings


Nm
Papa (given) 390
Tata post-acquisition retained earnings
(210 - 175 (W1)) x 80% 28
Loss on acquiring control (W2) (3)
Goodwill impairment attributable to parent (W3) (12)
Share of post acquisition profits of associate (W5) 35
438
(W7) Non-controlling interest in Tata
Nm
Book value (20% x 470) 94
Fair value adjustment (20% x 25) 5
Goodwill (3 - impairment 3) (W3) 0
99
EXAMINER'S REPORT
This question tests candidates' understanding of the preparation of consolidated statement of
financial position involving steps acquisition and associates.

About 99% of the candidates attempted the question and the performance was below average.

Commonest pitfall is the inability of the candidates to compute goodwill on acquisition, retained
earnings and non-controlling interest. Also, candidates were not able to distinguish an associate
company from subsidiary company.

Candidates are advised to master all aspects of group financial statements as this is always the
compulsory question in this paper. 1CAN Study Text should be deeply studied to improve
performance in future examinations.

Marking guide
Marks

i. Statement of Financial Position 10


ii. Working Notes 20
Total mark obtainable 30

SOLUTION 2
Computation Of Relevant Ratios For Odua Plc
Price earning mps 1.6 2.8
ratio EPS 0.104 0.258

= 15.38 times = 10.84 times

PBIT = Profit before interest and tax


PAT = Profit after Tax
MPS = Market Price Per Share

Interpretation and discussion of the Ratios

Profitability:
All profitability ratios declined in 2016. Gross margin dropped from 50% in 2015 to 27.27% in 2016.
This implies that the cost of sales of the company has increased in
2016 than it was in 2015. There was also a drop in net profit margin from 40% in
2015 to 12.36% in 2016. Return on Capital Employed declined from 27.59% to 12.59% while ROE
dropped from 25.83% in 2015 to 11.36% in 2016.

Gross profit has declined from N100m to N75m, the latter figure being N90m if the loss on disposal
were excluded from cost of sales. This is an underlying decline of 10%. This is not a disaster. It is
possible that the new strategy and system took some time to gain attraction in the market and it
would be very useful to see month by month figures to ascertain if there was an improvement over
the years. It is also entirely possible that the result would have been much worse had the new
strategy not been implemented. After all, the real comparison is not with last year, but with what
this year would have been, had the change not occurred. This is probably impossible to know.

Net margins are also down substantially (from 41% to 12.4%, the latter increasing to 17.8%
excluding the loss on disposal). If we further adjust these figures to exclude the gains and losses on
fair value investments, the figures are 19.6% (2016) and 36% (2015). This is a smaller decline, but
still significant.

The increase in operating costs is not surprising considering that revenue has increased by 37.5% in
value terms and much more in unit terms due to the reduction in prices. Hence, we can conclude
that operating cost efficiency had not materially deteriorated.

Short term Liquidity:


The current ratio has actually improved from 1.1:1 to 1.28:1, however the levels are still very poor.
The acid test ratio looks even worse, declining from 0.73:1 to 0.72:1. It seems that the company's
cash position is bad, with a positive N20m cash balance turning into a negative N40m over the year.
Apart from the bank overdraft, trade payables seem to be financing much of the company's liquidity
needs.

However, it is important to bear in mind that the company bore large one-off investments in 2016,
particularly the new N40m control system. This was financed from cash flow, as neither equity nor
borrowings have increased.

Also, it is likely that the increase in inventory and receivable (both absorbing cash) are related to the
expansion of sales levels, and may not be repeated.

However, that said, it is vital that the company examines closely its future cash flow needs and
considers raising new equity or debt as protection against unexpected events. The company is
dangerously exposed at present.

It would be important to know when the bank loan is due for repayment, as the company is not in a
position to repay it at present. Presumably the investments could be sold if necessary. This provides
a cushion of support.

Long term Solvency and Stability

The interest cover of the company is about 27 times in 2015 which shows a good performance while
the interest cover dropped drastically to 6.8 times in 2016 which must have arisen as a result of poor
performance that occurs in the period. Furthermore, the gearing ratio of the company increased
from 17.24% in 2015 to 18.52% in 2016. This still depicts a better performance of being lowly
geared.

Efficiency:

Although there have been significant increases in the figures for inventory, receivables and payables,
when expressed as a percentage of revenue and cost of sales as appropriate, the increases are not
that large.

The inventory days in 2016 was 73 days compared to 2015 of 69 days in 2016, while the receivable
days was 69 days in 2016 which is higher compared to 2015 of 51 days. Payables are being settled
more quickly than 2015, although from a very high base. It is important to guard against risks of
being charged higher prices if payment is slow. In the case of inventory, obsolescence may be a risk,
depending on the type of product being stored. Tighter inventory management reduces this risk.
Stock market performance

Earnings per share and price earnings ratio are measures of how well the company performs in the
market. The earnings per share of the company declined in 2016 when compared the figure of 0.10
kobo in 2016 with 0.26kobo of 2015. The company experienced decline in both market price and
earnings per share thereby showing an increase in price earnings ratio from 10.84 to 15.38.

Overall Comment:

From the analysis of the financial statements supplied and the ratios calculated, it is clear that the
performance of Odua Plc has deteriorated in 2016. Therefore, the company should focus on
increasing sales, maintaining its margins, and controlling liquidity very tightly through strong
working capital management.

EXAMINER'S REPORT
The question tests candidates understanding of ratio analysis regarding the identification of relevant
ratios, computation and interpretation of identified ratios.
About 94% of the candidates attempted the question and the performance of some of them was
above average.

Commonest pitfall for candidates in answering the question was their inability to appropriately
classify the ratios under the specified categories. The overall comment and conclusion were ignored
by most candidates.

Candidates are advised to prepare very well on this area of analysis and interpretation of the ratio as
this is a required knowledge at the final level of the examination.

Marking guide
Marks
a. Classification 374
b. Formula 6 1/2
c. Calculation (each year) 6 1/2
d. Interpretation 374
e. Conclusion/Overall comment 1 /2

20
SOLUTION 3

Date:

To: Employees representative of Funda Plc.

From: Financial Controller

Subject: Accounting policies used by Funda Plc


(a) Suitability of accounting policies
For an accounting policy to be suitable, it had to meet up with the following criteria.
i. It has to conform with regulatory procedures of Standards
Organisation of Nigeria (SON), NAFDAC, etc.
ii. The policy must meet up with current accounting treatment as laid down in IAS, IFRS, etc. as
follows: (IAS 8 - Accounting Policies, changes in Accounting Estimates and Errors). 1 write to
explain the suitability and treatment of specific accounting policy matters which were used
in the computation of the financial information meant for loan application.
- Relevant to the decision making needs of users, and
- Reliable in the financial statements:
- Represent faithfully the results and financial position of the entity.
- Reflect the economic substance of transactions and other events, and not
merely the legal form;
- are neutral, i.e free from bias
- Are prudent; and
- Are complete in all materials respects
iii. Documentation procedures as specified in the operating manual and Auditors
suggestions should be reflected in the accounting policies of the company.
iv. Funda Plc accounting policies should facilitate accounting system audit.
v. Areas of efficiency improvement in the system should be reflected in the accounting
policy.
vi. Accounting policy would aid better understanding by the users of financial
statement.
vii. Further explanations, clarifications in the treatment of specific transactions
mentioned in the account.
viii. Accounting policies should be consistently applied every year and reflect changes (if
any).
ix. Since no company operates in isolation accounting policies should
form the basis of comparison with the financial statements of companies in the
same industry.
(b) Accounting treatment
Sales of water filter - (IAS - Leases)
i. It is a sale and repurchase arrangement
ii. Volume discount should be deducted to arrive at the net income (that is N625,000)
less N125,000.
iii. Put option is an example of a financial instrument that provides finance for six years.
iv. N500,000 net payment can be recognised as revenue/sales/income.
v. Put-option can be defined as a contract that gives the purchaser the right but not the
obligation to sell a specified quantity of a particular financial instrument,
commodity, foreign currency etc at a specified price (strike price) during a specified
period.
vi. N175,000 (i.e. 35% of N500,000) is a premium to compensate the buyer for the risk
of payment under the option.
vii. N29,166.67 (N175,000 6) should be debited to Profit or Loss
annually for six years in respect of a filter. This amount should be provided for and
debited to Profit or Loss and such provision should be recognised as part of current
liability in statement of financial position.
viii. Depreciation should be provided and debited to Profit or Loss.
ix. Depreciation of N50,000/year/unit of filter in each year for six years should be
debited to Profit or Loss.^(500,000 10 = N50,000)
x. The item is part of inventory when it is collected back at the end of six years and
classified under current asset in the statement of financial position.
xi. The inventory should be valued at N200,000 (N500,000 - N300,000) (50,000 x 6
years) = N300,000 at net realisable value (NRV)

Connection fee

- The refundable connection fee should not be credited to Profit or Loss


because of its nature as refundable. (IAS 18 - Revenue)

- Only the cost of disconnection should be credited toProfit or Loss as


income/revenue/sales.

- The amount of N4m in 2016 and N5m in 2017 should be recognised as


contingent liability in the Statement of Financial Position (IAS 37 -
Provisions, contingent Liabilities and contingent Assets).
- Amount credited to Profit or Loss in the account is wrong.

Activation fees

- The treatment gives to the transaction as credit to Profit or Loss is in order/correct.

- DR: Bank/cash CR: Activation fee account

Deposits
- Recognition of deposits as revenue in the statement of profit or loss when orders are placed
is wrong.

- N9m (90% of N10m) should be recognised as sales (fulfilled order).

- N1m (unfulfilled order), i.e. 10% of N10m is not revenue but a liability and should be shown
under appropriate heading in the Statement of Financial Position.

Conclusion/Closing

Would you demand for further clarification do not hesitate to contact me Name and

Signature

EXAMINER'S REPORT
This question tests candidates' knowledge of report writing, suitability of the accounting policies on
practical situation and correct treatment of specified transactions.

About 36% of the candidates attempted the question and performance was generally poor and
below average.

The commonest pitfall was that most candidates ignored the report aspect of the question and
could not explain the suitability of accounting policies.

Candidates are advised to pay particular attention to reporting aspect of this subject and to deeply
understand the application of accounting standards to practical situations.
SOLUTION 4

(a) Scenario 1

Criteria for revenue recognition

For revenue to be recognised the following criteria must be satisfied:


• The entity had transferred to the buyer the significant risk and reward of ownership.
• The amount of revenue is capable of being measured reliabily
• The cost incurred can be measured reliably.
• The stage of completion of the transaction at the reporting date can be measured
reliably.
The following criteria are not satisfied in the consultancy contract, hence, the
full amount of N1,000,000 is to be recognised as LIABILITY: OR NON-CURRENT LIABILITY.
NONE is to be recognised in the REVENUE ACCOUNT

Scenario 11

Borrowing costs directly attributable to construction of an asset which takes long period to
get ready for its intended use should be capitalised as part of the cost of the asset. Although,
borrowing costs were incurred throughout the year, capitalisation commences when project
for which the borrowing was contracted is complete. 1n the case of general borrowings, the
calculation of amount to capitalise is based on weighted average cost of borrowing.

Capitalisation by using Weighted Average of the borrowing costs


Type of borrowings Amount Rate(%) Hash
N total
N
Bank overdraft 1,000,000 9.75 97,500
Bank loan 1,750,000 10 175,000
Loan notes 2.500.000 8 200.000
5.250.000 472.500

Capitalisation Rate = 4 72, 50 0


x100
^ 5,250,000

= 9%

Computation of interest on borrowing:


N N
Land @ cost 1,800,000
Interest (9% x 1,800,000 x 8/12) 108,000
Construction work 2,700,000
Interest (9% x 2,700,000 x 4/12) 81.000
Total interest capitalised 189.000 89.000
Total cost of assets 4,689,000
Scenario 111
Depreciation of the blast furnace has been based on an estimated useful life of 20 years.
This is at variance with a report by a qualified expert.

• Where estimates have been prepared by qualified professional, then the


professional valuers estimates to be used is 15years.

Carrying amount of assets N3,500,000


Estimated useful life 20 years
Depreciation charged N175.000

Using professional valuer's estimate of 15 years the depreciation to be changed is


3.500.000
15

Depreciation to charges N233.333.33


Depreciation undercharged = N(233,333.33 - 175,000)
= N58.333

Additional depreciation of 58,333 is to be charged to the account in 2014


The lining which was replaced in the last week of the year will not attract
any depreciation for the year.

Revised Financial Statements


Statement of Profit or Loss Extract for the year ended December 31, 2014

N'000 N'000
Profit before tax 2,500
Consultancy agreement 1,000
Over capitalised interest 261
Undercharged depreciation 58.33
(1,319.33)
Revised profit 2.180.67
Statement of Financial Position as at December 31, 2014

Draft Adjustment Revised


N'000 N'000 N'000
Property, Plant & Equipment 12,000 (319.33) 11,680.67
Current assets 3r500 3.500
Total assets 15.500 1.518.67
Share capital 2,000 2,000.00
Retained earnings 6r000 L,319.33) 4,680.67
8,000 6.680.67
Non-current liability 5,000 500 5,500.00
(b)
Current liability 2r500 500 3.000.00
15,500 15,180.6
7

Ethical Issues

The following are ethical issues arising from the scenario above:

• Previous Financial Controller (FC) resigned as a result of pressure from Managing Director
(MD).
• MD pressured the FC to compute depreciation with an unrealistically long useful life
• MD pressured FC to present the favourable financial statements in favourable light
preparatory to stock market listing.

Possible courses of actions by the FC

• Discuss with the MD about compliance with reporting standards


• Explain to the MD that as a professional, you are bound by ethical code
• Consider speaking with other Directors or Audit Committee (if any)
• If all action fails, then consider resigning/seeking alternative employment.

EXAMINER'S REPORT

The question tests the candidates' understanding and application of criteria for revenue recognition,
change of accounting policies, computation of average, cost of borrowings and ethical issues
confronting accountants in the preparation of financial statements.

About 57% of the candidates attempted the question but they demonstrated poor understanding of
the requirement. Their performance was generally poor.
The commonest pitfall was the inability of the candidates to recognise the applicable timing of
recognition for cost of borrowing and the applicable depreciable time of the acquired assets.

Candidates are advised to get acquainted with the provision of International Financial Reporting
Standards (IFRS) and be able to apply them to practical situations.

Marking Guide
Marks
(a) (i) Criteria 2
Recognition of N1m 2
(ii) Computation of cost of capital
and Depreciation 6
(iii) Computation of depreciation
of furnace 4
Redraft of financial statement _2
16

(b) Ethical issues 2


Courses of action 2
20

SOLUTION 5
(a) i. A defined contribution Plan is a post-employment plan under which
an entity pays fixed contribution into a separate entity (a fund) and will have no
legal or constructive obligation to pay further contributions if the fund does not hold
sufficient assets to pay all employee benefits relating to employee service in the
current and prior periods.

A defined benefit plan is any post-employment benefit plan other than a defined
contribution plan. This plan uses actuarial technique (projected unit credit method)
to estimate the ultimate cost to the entity of the benefits that employees have
earned in return for their service in the current and prior periods.

Differences between a defined contribution plan and a defined benefit plan

i. Under a defined contribution plan, there are no actuarial assumptions


to make, while under a defined benefit plan actuarial technique is relevant;
ii. Defined contribution plan does not usually affect statement of financial position
whereas defined benefit plan will always affect both statement of financial position
and statement of profit or loss and other comprehensive income;

iii. Under a defined contribution plan, the entity is ONLY responsible for the amount of
annual contribution payable into the fund, whereas in a defined benefit plan the
amount of benefits receivable by the employees is key;

iv. Under a defined contribution plan, the obligation is determined by the amount paid
into the plan in each period whereas under a defined benefit plan, actuarial
technique is used to estimate the ultimate cost to the entity of the benefits that
employees have earned in return for their service in the current and prior periods;

v. In defined contribution plan, amount received by the employee is not pre-


determined while under defined benefit plan, the amount received by employee is
pre-determined;

vi. In contribution plan, the entity is not required to make good any short falls if pension
doesn't have enough assets to pay whereas under benefit plan the entity i.e.
employer bears the short fall if the pension funds are insufficient to pay the
retirement of the employees;

vii. In contribution plan the employees bear the risk while in benefit plan the employer
bears the risk; and

viii. No actuarial assumption to make in contribution plan while actuarial technique is


relevant in defined benefit plan.

(b) i. Accounting treatment for Pension Plan I: The accounting for the
defined benefit plan is as follows:

Financial position as at October 31, 2017

31/10/2017 1/11/2016
N'm N'm
Present value of obligation 240 200
Fair value of plan assets (225) (190)
15 10

30
Expense in statement of Profit or Loss for year ended 31/10/17
N’m N'm
Current service cost 20.0
Net interest expense:
Interest expense - 5% x N200m 10
Interest earned - 5% x N190m (9.5) 0.5
Total expenses 20.5

Computation of Actual Gain or Loss

Actuarial Loss on Obligation (Working 1)


N'M
Present value of obligation 1/11/16 200
Interest expense (5% x N200m) 10
Current service cost 20
Benefits paid out (19)
211
Present value of obligation 31/10/17 (240)
Actuarial loss of obligation (29)

Actuarial Gain on Plan Assets (Working 2)


N'M
Fair value on plan assets 1/11/16 190.0
Interest earned (5% x N190m) 9.5
Contribution paid 17.0
Benefit paid out (19.0)
197.5
Fair value of plan assets 31/10/17 225.0
Actuarial Gain on Plan Assets 27.5

Analysis of amount in statement of Other Comprehensive Income (OC1)


N'M
Actuarial loss on obligation (working 1) 29.0
Actuarial Gain on plan assets (working 2) 27,5
Actuarial Loss on obligation (Net) 1.5
Working 3

Movement in net liability in statement of financial position as 31/10/17

N'M
Opening liability 10.0
Expenses 20.5
Contributions paid (17.0)
Actuarial loss (Net) 1.5
Closing liability 15

Accounting Treatment for Pension Plan 2


The accounting for a defined contribution plan is fairly simple because the employer's
obligation for each period is determined by the amount that had to be contributed to the
plan for that period.

The company does not recognise any assets or liabilities for the defined contribution
scheme, but charges the contribution payable for the period (N10m) to operating profit. The
contributions paid for the employees will be part of the wages and salaries cost and when
paid will reduce cash.

Tinubun Plc

Statement of contribution paid

N'm
Contribution paid by employer 10
Contribution paid by employees
10
20

Statement of Profit or Loss


Expenses:
N'm
Contribution paid by employer
10

EXAMINER'S REPORT
This question tests the understanding of the concept of 1AS 19, its interpretation and practical
applications.
About 46% of the candidates attempted the questions and the performance was below average.
Many of the candidates who attempted the question misunderstood the requirements.
The major pitfall encountered by the candidates was wrong Interpretation and application of the
concept of IAS 19 especially on the difference between a defined contribution plan and a defined
benefit plan.

Candidates are advised to study and understand the requirement of IFRS in depth for application at
this level of examination. They are to use the 1CAN study texts for better performance in future
examinations.
SOLUTION 6

EKO EXPORTS LIMITED (EEL)

i. The debt owed by the customer existed at the reporting date, but the customer's inability to
pay did not exist at that point. This condition only arose on February 10, 2017. (IAS 10 -
Events occurring after the reporting date)

Thus, this a non-adjusting event


However, for its materiality to the financial statements, the following disclosure should be
made:

• Nature of the event


i. e. a description of the fire incident on the customer warehouse and its
subsequent declaration as a bankrupt.

• An estimate of its financial effect


Since 50% is recoverable from insurance, the balance of 50% i.e. N500,000 should be
disclosed as the effect of the incident by way of note.

ii. The amount withdrawn before the year-end i.e. N1.5million is an adjusting event, though it
was discovered after year-end, it existed at year-end. (IAS
10 - Events occurring after the reporting date).

Since 60% has been recovered (N1.5m x 60% - N0.9m) therefore only N0.6m would be
provided in the financial statements.

The further withdrawal of N6.0million is a non-adjusting event, as it occurred after the year-
end.

For its materiality the following should be disclosed by way of note.

• Nature of event (i.e. the employee fraud)


• The gross amount of contingency (i.e. N6.0million)
• The amount recovered subsequently (i.e. N6.0m x 60% = N3.6m)

iii. EEL should not recognise the contingent gain until it is realised (A contingent asset). (IAS 37-
Provisions contingent liabilities and contingent Assets)

If recovery of damages is probable and material to the financial statements, EEL should
disclose the following in the notes:
• Brief description of the nature of the contingency asset (i.e. description of suit for
claim of damages again a vendor for supply of defective goods).
• An estimate of the financial effect: (N1million the probable claim).

iv. EEL should make a provision of the expected amount i.e. N1.2million (N1.0m x 60% + N1.5m
x 40%) (IAS 37- Provisions contingent liabilities and contingent Assets) because
- It is a present obligation as a result of past event;
- It is probable that an outflow of resources embodying economic benefits will be
required to settle the obligation; and
- A reliable estimate can be made of the amount.

In addition, EEL should disclose the following in the notes to the financial statements:

- Brief nature of the contingent liability (claim against EEL for infringement of patent);
- The amount of contingency (I.e. N1.2million); and
- An indication of uncertainties relating to the amount or timing of any out flows.

EXAMINER'S REPORT
This question tests the application of relevant provisions of standards to a given scenario, in a real
life situation where a qualified professional is expected to handle on the job.

About 58% of the candidates attempted the question and their performance was very poor as they
were not able to apply the provisions of the standards to the requirements of the question.

Commonest pitfall of the candidates was inability to interpret the question as practical application
of the standards. Theoretical answers were provided by majority of the candidates.

Candidates are advised to familiarise themselves with the provisions of the standards, study the
financial reports of organisations for more knowledge to perform better in future examinations.
Marking guide Marks Marks
i. Scenario i
Identification of non adjusting events 1
Nature of the event 1
Estimate of financial effect 1
Identification IAS 10 as the required standard 1 4

ii. Scenario ii
Identification of N9 million as an adjusting event 1
Calculation of amount of provisions 1
Identification of N6m as non-adjusting event 1
Any one of materiality disclosure 1 4

iii Scenario iii


Identification of Contingent asset 1
Two disclosures materiality 2 3

iv Scenario iv
Calculation of correct amount for provisions 1
Disclosures in the financial statements(any three disclosures) 3 4
Total 15

SOLUTION 7

(a) The Directors,


Mugono Plc

Dear Sir,

Component Reports included in Annual Report

In response to your request on the above subject, below is our report:

Mandatory disclosure reports:


1. A statement of financial position as at the end of thefinancial period.
2. A statement of comprehensive income for the period (made up of a
statement of profit or loss and other comprehensive income).
3. A statement of changes in equity for the period.
4. A statement of cash flows.
5. Notes to the financial statement, consisting of a summary of significant accounting
policies and other explanatory notes.
6. Directors' report
7. Independent Auditors' report.

Voluntary disclosure reports


8. Audit Committee report.
9. Corporate social responsibility report.
10. Brief description of the business activities in the financial year.
11. Chairman's report.
12. Corporate Governance report.
13. Sustainability/Environmental report.
14. Financial summary
(b) i. Three advantages of mandatory disclosures include:
- The statements provide information that are of interest to users and other
stakeholders;
- The mandatory statements shows the financial position as at the end of a
reporting period;
- Mandatory reports shows the financial performance for a
particular period;
- 1t improves communication to stakeholders;
- 1t improves public image of the company; and
- Decision making of the stakeholders is improved with
mandatory disclosures.

ii Advantages of Voluntary Disclosures are as follows:


- 1t protects the company against any litigation of concealment;
- 1t provides the potential investors detailed information that will assist them
in appraising the viability of the proposed investment;
- 1t improves the fluidity of capital market towards an efficient capital
allocation, thereby bringing down the average cost;
- 1t provides the public with more robust view and perception of the
company future performance;
- 1t motivates for better performance;
- It reinforces company's communication to shareholders;
- It protects the entity's goodwill and reputation; and
- 1t is a means of passing across the performance level of an entity's
corporate social responsibility.
iii. Limitations of information provided on a voluntary basis are as
follows:

- The entity can decide on what to include in the report and what to leave out;

- The information is often presented in a very positive form, as


public relations for investors, and might not be entirely reliable;

- 1t increases public expectation regarding future disclosure;

- 1t increases the involvement of shareholders and might lead to


unnecessary conflicts; and

- The information can be false if not controlled.

We are of the opinion that the above meets your request. However, do not hesitate
to get in touch with the undersigned for clarification, if necessary.
Yours faithfully,

Financial Consultant

EXAMINER'S REPORT
The question tests candidates' knowledge of the components of annual reports with special
attention on the mandatory and voluntary disclosures requirements.

About 96% of the candidates attempted the question and their performance was above average.
However, only about 50% of the candidates who attempted the questions were able to score above
50% of the allocated marks.

The commonest pitfall is the inability of the candidates to correctly state the advantages of
mandatory and voluntary disclosures.

Candidates are advised to have a better understanding of all the components of Annual Report and
the Mandatory disclosures. The art of report writing should also be improved upon for better
performance in future examinations.
Marking guide Marks
(a) Any correct FOUR Components of Annual Report 4
(b) i. Three advantages of mandatory disclosures 3
ii. Four advantages of voluntary disclosures 4
iv. Two limitations of voluntary disclosures 2
Presentation 2
15
THE INSTITUTE OF CHARTERED ACCOUNTANTS OF NIGERIA PROFESSIONAL LEVEL EXAMINATION -

MAY 2018
CORPORATE REPORTING
Time Allowed: 3V4 hours (including 15 minutes reading time)

INSTRUCTION: YOU ARE REQUIRED TO ANSWER FIVE OUT OF SEVEN


QUESTIONS IN THIS PAPER

SECTION A: COMPULSORY QUESTION (30 MARKS)

QUESTION 1
Komolafe Group carries on business as a distributor of warehouse equipment and importer of fruit
into the country. Komolafe was incorporated in 2008 to distribute warehouse equipment. It
diversified its activities during the year 2010 to include the import and distribution of fruit, and
expanded its operations by the acquisition of shares in Kelvins in 2012 and Kelly in 2014.

Accounts for all companies are made up to December 31.

The draft statements of profit or loss and other comprehensive income for Komolafe, Kelvins and
Kelly for the year ended December 31, 2016 are as follows:

Komolafe Kelvins Kelly


N'000 N'000 N'000
Revenue 91,200 49,400 45,600
Cost of sales (36.100) (10.926) (10.640)
Gross profit 55,100 38,474 34,960
Distribution costs (6,650) (4,274) (3,800)
Administrative expenses (6,950) (1,900) (3,800)
Finance costs (650) 0 0
Profit before tax 40,850 32,300 27,360
Income tax expense (16,600) (10,780) (8,482)
Profit for the year 24,250 21,520 18,878
Other comprehensive income for the year: Items that400 200 0
will not be reclassified to profit or loss in subsequent period
Revaluation of property

24.650 21.720 18.878


The draft statement of financial position as at December 31, 2016 are as follows:

Komolafe Kelvins Kelly


Non-current assets N'000 N'000 N'000

Property, plant and equipment (carrying amount) 70,966 48,546 26,126


Investments
Shares in Kelvins 13,300 000 000
Shares in Kelly 0 7,600 0
84,266 56,146 26,126
Current assets 3.136 18.050 17.766
Equity 87,402 74,196 43,892

Ordinary shares 16,000 6,000 4,000


Retained earnings 45,276 48,150 39,796
Current liabilities 26.126 20.046 96
87,402 74,196 43,892

The following information is available relating to Komolafe, Kelvins and Kelly.

(i) On January 1, 2012 Komolafe acquired 5,400,000, N1 ordinary shares in Kelvins for
N13,300,000 at which date there was a credit balance on the retained earnings of Kelvins of
N2,850,000. No shares have been issued by Kelvin since Komolafe acquired its interest
(ii) At the date of acquisition, the fair value of the identifiable net assets of Kelvins was N10m. The
excess of the fair value of net assets is due to an increase in the value of non depreciable land.
(iii) On January 1, 2014 Kelvins acquired 3,200,000, N1 ordinary shares in Kelly for N7,600,000 at
which date there was a credit balance on the retained earnings of Kelly of N1,900,000. No
shares have been issued by Kelly since Kelvins acquired its interest. The fair value of the
identifiable net assets of Kelly at the date of acquisition approximates their book values.
(iv) During 2016, Kelly had made intra-group sales to Kelvins of N960,000 making a profit of 25%
on cost and N150,000 of these goods were in inventories at December 31, 2016.
(v) During 2016, Kelvins had made intra-group sales to Komolafe of N520,000 making a profit of
25% on sales and N120,000 of these goods were in inventories at 31 December 2016
(vi) An impairment test conducted at the year end did not reveal any impairment losses.

(vii) It is the group's policy to value the non-controlling interest at fair value at the date of
acquisition. The fair value of the non-controlling interests in Kelvins
on January 1, 2012 was N1,000,000. The fair value of the 28% non-controlling interest (direct
and indirect) in Kelly on January 1, 2014 was N1,800,000.
Required:
Prepare for Komolafe Group:
a. A consolidated statement of profit or loss and other comprehensive income
for the year ended December 31, 2016. (13 Marks)

b. A consolidated statement of financial position as at December 31, 2016

(12 Marks)

c. In business combination, the consideration given by the acquirer to gain control of the
acquiree can be in different forms including deferred and contingent considerations. While
deferred and contingent considerations represent amounts of consideration to be transferred
in the future, the two differ in nature and form.

Required:
Briefly distinguish between deferred and contingent consideration.
(5 Marks)
(Total 30 Marks)

SECTION B: YOU ARE REQUIRED TO ANSWER ANY TWO OUT OF THREE


QUESTIONS IN THIS SECTION (40 MARKS)

QUESTION 2
Set out below are the draft accounts of Wole-Adura Plc and subsidiaries and of
Maseru Associates. Wole-Adura acquired 40% of the equity capital of Maseru
Associates three years ago when the latter's retained earnings stood at N140m.

Abridged statement of financial position

Wole-Adura Plc Maseru


& Subsidiaries Associates
Nm Nm
Property, plant and equipment 990 595
Investment in Maseru Associates at cost 290
Loan to Maseru Associates 70
Current assets 450 175
Loan from Wole-Adura Plc. - 170
1800 700
FINANCED BY:
Ordinary shares of 50k each 1,125 350
Retained earnings 675 350
1800 700

Abridged statements of profit or loss


Wole-Adura Plc Maseru
& Subsidiaries Associates
Nm Nm
Profit before tax 427.50 280.00
Tax expense (157.50) (105.00)
270.0 175.00

Additional information:
(i) Wole-Adura proposed a dividend of N225m.
(ii) Total market capitalisation is N5,625m.

Required:
a. Calculate each of these ratios for Wole-Adura Plc. and subsidiaries.
i. Earnings per share.
ii. Dividend cover.
iii. Earnings yield.
iv. Dividend yield. (4 Marks)

(i) Using the equity method, compute the earnings of the group

incorporating the associates. (4 Marks)


(ii) Compute the ratios in (a) above for the group. (4 Marks)

c. Comment on the ratios calculated in (a) and (b) above by pairwise


comparison. (3 Marks)

d. Extracts from the financial statements of Ikoku Plc. recently published are as follows:

Statement of profit or loss for the year ended December 31, 2017
Operating profit 160 5
Interest expense (10) -
Tax expense (60) (2)
Profit for the year 90 _3_

Statement of financial position as at December 31, 2017


2017 2016
N'm N'm
Non-current assets
Property, plant & equipment 80 20
Current assets
Inventory 200 40
Trade receivables 70 25
Bank (50) 30
Total assets 300 115

Equity & liabilities


Ordinary shares of N1 each 60 40
Current liabilities
Trade payables 190 60
Current tax 50 15
300 115

Required:
Discuss the liquidity challenges of Ikoku Plc. during the year ended December 31, 2017
from the extracts of the published financial statements.
(5 Marks)
(Total 20 Marks)

QUESTION 3
a. “IFRS 5 Non-current Asset held for Sale and Discontinued Operations" sets
out the principles governing the measurement and presentation of noncurrent assets that
are expected to be realised through sale rather than through continuing use. The standard
also deals with reporting the results of operations that qualify as discontinued.
Required:
Discuss the conditions which must be met for a non-current asset to be classified as being
“held for sale" and explain the accounting treatment that applies when such a classification is
deemed appropriate. (7 Marks)
b. Bamgbose Plc. is a long-established travel agent, operating through a network of retail outlets and
online store. In recent years, the business has seen its revenue from the online store grow
strongly, and that of retail outlets decline significantly. On July 1, 2017, the board decided to
close the retail network at the financial year end of December 31, 2017 and put the
buildings up for sale on that date. The directors are seeking advice regarding the treatment
of the buildings in the statement of financial position as well as the treatment of the trading
results of the retail division for the year. The following figures are available at December 31,
2017.
Carrying amount of buildings &30.0 million
Fair value less costs to sell of buildings N25.8 million
Other expected costs of closure N5.85 million
Trading results:

Year ended December 31, 2017 Year ended December 31, 2016

Online store Retail Online store Nm Retail


Nm outlet outlet
Nm Nm
Revenue 58.5 13.5 48 18
Cost of sales 19.5 10.5 16.5 13.5
Gross profit 39 3 31.5 4.5
Operating costs (15) (7.5) (12) (7.5)
Profit before tax 24 (4.5) 19.5 (3)

Required:
i. Outline the conditions which must be met in order to present the results of an
operation as "discontinued" and the accounting treatment that applies when such a
classification is deemed appropriate. (5 Marks)

ii. Draft the statement of profit or loss for Bamgbose Plc. for year ended December 31,
2017 together with the comparative figures for 2016, taking the above information
into account. (8 Marks)
(Total 20 Marks)

QUESTION 4

a. Recording the substance of transactions, rather than their legal form, is an important principle
in financial reporting. The use of off-statement of financial position financing arrangement
enables companies to obtain financing without showing debts in their books.
Required:
Describe how the use of off-statement of financial position financing can mislead users of
financial statements, making specific reference to THREE user
groups and giving examples where recording the legal form of transactions may mislead them.(6 Marks)

b. Waasimi entered into the following transactions during the year ended March 31, 2018:
In March 2018, Waasimi factored some of its trade receivables to Asejere, a finance house.
Based on selected account balances, Asejere paid Waasimi 80% of its book value. The
agreement was that Asejere would administer the collection of the receivables and remit a
residual amount to Waasimi depending upon how quickly individual customers paid. Any
balance not collected by Asejere after six months will be refunded to Asejere by Waasimi.

On April 1, 2017, Waasimi's freehold building had a carrying amount of N15million and an
estimated remaining useful life of 20 years. On this date, Waasimi sold the building to
Gbajumose for a price of N24million and entered into an agreement with Gbajumose to lease
back the building for an annual rental of N2.6million for a period of five years.

The auditors of Waasimi have commented that in their opinion the building had a market value
of N20million at the date of its sale and to rent an equivalent building under similar terms to
the agreement between Waasimi and Gbajumose would cost N1,600,000 per annum. Assume
finance cost of 10% per annum.
Required:
i. Briefly explain the major accounting issues involved in the above
transactions using the principles of substance over form. (5 Marks)

ii. State the appropriate accounting treatments of the various elements


identified. (6 Marks)

iii. State the classes of charges to be incurred and their appropriate


accounting treatments. (3 Marks)
(Total 20 Marks)

SECTION C: YOU ARE REQUIRED TO ANSWER ANY TWO OUT OF THREE QUESTIONS
IN THIS SECTION (30 MARKS)

QUESTION 5
a. Deferred tax can be determined adopting two perspectives that may result in
different numbers in the financial statements and tax computations. These are statement of
comprehensive income and statement of financial position, perspectives.
Required:
Distinguish between the TWO perspectives of identifying deferred tax balances in the
financial statements. (4 Marks)

b. The following information relates to Tola Plc. as at December 31, 2017:

Note Carrying amount Tax base


Non-current assets N
Plant and equipment 250,000 218,750
Receivables:
Trade receivables 1 62,500
Interest receivable 1,250

Payables
Fine 12,500
Interest Payable 2,500
Note 1
The trade receivables balance in the accounts is made up of the following amounts:
Balances 68,750
Doubtful debt provision (Specific) (6,250)
62.500

Further information:
(i) The deferred tax balance as at January 1, 2017 was N1500.
(ii) Interest is taxed on a cash basis.
(iii) Allowances for doubtful debts are not deductible for tax purposes. Amounts in respect of
receivables are only deductible on application of a court order to a specific amount.
(iv) Fines are not tax deductible.
(v) The tax rate is 30% for 2017. The government has not announced the tax rate
for 2018 but it is expected to rise to 36%.

Required:
Compute the deferred tax provision which is required as at December 31, 2017 and the
charge to profit or loss for the period in accordance with IAS, 12 - Income Taxes. (11
Marks)
(Total 15 Marks)
QUESTION 6

Omotola Nigeria Plc is a conglomerate which operates in different sectors of the economy. The
company has many subsidiaries and associates across the six continents of the world and its head
office is located in Lagos, Nigeria. The shares of the company are listed on the Nigerian Stock
Exchange.

The company is trying to finalise its financial statements for the year ended April 30, 2018 and the
following accounting issues are being considered by the chief accountant based on the submission
by the assistant accountant who is yet to complete her professional examinations with the Institute
of Chartered Accountants of Nigeria. The functional and presentation currency of Omotola Nigeria
Plc. is Naira. The following transactions relate to the company:

(i) On May 1, 2017, Omotola Nigeria Plc. bought an investment property in United States for
$1,000,000. The company uses fair value model of IAS 40 to account for the investment
property and the fair value at April 30, 2018 is determined to be $1,200,000. The assistant
accountant is unsure which exchange rate to use in translating the investment property at
the year end and how to recognize any exchange difference that may arise.

(ii) On May 1, 2017, Omotola Nigeria Plc. acquired a wholly owned subsidiary in United States
of America. The goodwill that arose on the acquisition of this subsidiary is $400,000.

In addition, the company invested in an equity instrument on the same date which is
measured at fair value through other comprehensive income (OCI) in accordance with the
requirements of IFRS 9.

Required:
a. In accordance with the requirement of IAS 21 - Effect of Changes in
Foreign Exchange Rates, discuss the treatment of foreign currency transactions and the
gain or loss arising therefrom. (7 Marks)

b. Discuss how the transaction in (i) will be accounted for in the financial
statements of Omotola Nigeria Plc. for the year ended April 30, 2018 in accordance
with IAS 21 (4 Marks)

c. Discuss how the transaction in (ii) will be accounted for in the financial
statements of Omotola Nigeria Plc. for the year ended April 30, 2018 in accordance
with IAS 21. (4 Marks)
(Total 15 Marks)
QUESTION 7

Some shareholders in Nigeria are becoming increasingly interested in the environmental policies,
impacts and practices of business entities given the activities of some oil and gas and
telecommunication companies. However financial statements have not traditionally provided this
information. As a result, there is early indication that some listed companies in Nigeria are beginning
to publish sustainability report complying with the Global Reporting Initiative ("GR1"), an
organisation set up in 1997, to develop a sustainability reporting framework for businesses. The GRI
Sustainability Reporting Guidelines give guidance to entities on how to measure and report on
managements' approach to the economic, environmental and social aspects that impact on their
businesses.

Required:
a. Identify and explain the principal arguments against voluntary disclosure by business
entities of their environmental policies, impacts and practices.
(8 Marks)
b. E
xplain the nature of the information that could be disclosed by entities in
their external reports in respect of the economic, environmental and social aspects, in order
to comply with the GRI guidelines. (7 Marks)
(Total 15 Marks)
SOLUTION 1

(a) KOMOLAFE GROUP


CONSOLIDATED STATEMENT OF PROFIT OR LOSS AND OTHER COMPREHENSIVE
INCOME FOR THE YEAR ENDED DECEMBER 31, 2016
NOTE N'000
Revenue 2 184,720
Cost of sales 3 (56,246)
Gross profit 128,474
Distribution costs 4 (14,724)
Administrative expenses 5 (12,650)
Finance costs (650)
Profit before tax 100,450
Income tax expense 6 (35.862)
Profit for the year 64,588
Other comprehensive income for the year
Items that will not be reclassified to profit or loss in subsequent period

Revaluation of property 7 600


Total comprehensive income for the year 65.188

Profit attributable to:


Equity holders of the parent 8 57,162
Non-controlling interest 8 7,426
64.588
Total comprehensive income attributable to:
Equity holders of the parent 10 57,742
Non-controlling interest 9 7.446
65.188

(b) KOMOLAFE GROUP


CONSOLIDATED STATEMENT OF FINANCIAL POSITION AS AT DECEMBER 31,
2016
Note N'000
Non-current assets
Property, plant and equipment (carrying amount) 11 146,788
Intangible asset- Goodwill 13 7,040

Current assets 15 38,892


Total assets 192.720
Equity and Liabilities
Equity
Ordinary share 16,600
Retained earnings 16 113.283
Equity attributable to owners of parent 129,283
Non-controlling interest 14 17,169
Total equity 146,452
Current liabilities 17 46,268
Total equity and liabilities 192.720

Working notes
1. Group Structure
10. Total Comprehensive Income Attributable to Owners of
Parent N'000

Profit for the year (see 8 above) 57,162


Other comprehensive income:
Revaluation of property (400 x 100%) 400
Revaluation of property (200 x 90%) 180
57.742

11. Property, plant and equipment (carrying amount) N'000


KOMOLAFE 70,966
KELVINS 48,546
KELLY 26,126
Fair value adjustment 1.150
146.788

12(a) Net Assets of the subsidiary - KELVINS


Reporting Acquisition Post
Date Date Acquisition
N'000 N'000 N'000
Share capital 6,000 6,000 -
Retained earnings 48,150 2,850 45,300
Fair value adjustment - Land 1,150 1,150
Unrealised profit - Inventory (30) ______ (30)
55.270 10.000 45.270

12(b) Net Assets of the subsidiary - KELLY


Reporting Acquisition Post
Date Date Acquisition
N'000 N'000 N'000
Share capital 4,000 4,000 -
Retained earnings 39,796 1,900 37,896
Unrealised profit - Inventory (30) ______ (30)
43.766 5.900 37.866
13. Goodwill KELVINS KELLY TOTAL
N'000 N'000 N'000
Consideration transferred by the parent 13,300 6,840 20,140
Fair value of NCI at acquisition 1.000 1.800 2.800
14,300 8,640 22,940
100% of net assets of subsidiary at
acquisition (note 12) (10,000) (5,900) (15,900)
Goodwill 4.300 2.740 7.040

Note: the consideration transferred for the acquisition of KELL Y is N7,600x 90% i e N6,84m

GOODWILL - ALTERNATIVE SOLUTIONS


14. Current Assets N '000
KOMOLAFE 3,136
KELVINS 18,050
KELLY 17,766
Less: Unrealised Profit in Kelvins (30)
Unrealised Profit in Kelly (30)
38.892

15. Retained Earnings N'000

Komolafe 45,276
Add: Share of Post-acquisition profit
in Kelvins (45,270 x 90%) (Wn 12) 40,743
in Kelly (37,866 x 72%) (Wn 12b) 27.264
113.283

N'000
16. Current Liabilities
26,126
KOMOLAFE
20,046
KELVINS
96
KELLY
46.268

(c) Deferred and contingent considerations


Deferred consideration arises when all or part of the cost of an acquisition is deferred and
does not become payable until a later date. In this case, the amount of any deferred
consideration is discounted to its present value at the acquisition date to arrive at fair value
as required by IFRS 3. Deferred consideration should be included in the cost of acquisition
(i.e. as part of the consideration to acquire the acquiree).

Contingent consideration, on the other hand, arises when the final cost of the consideration
is contingent on (dependent on) certain future events. For instance, an acquirer could agree
to pay an additional amount if the acquired subsidiary's profits exceed a certain level within
a specified period after the date of acquisition. Contingent consideration should be included
in the cost of acquisition (discounted to present value if the payment will occur more than
12 months in the future). Contingent consideration must be recognised at acquisition date
as long as the fair value can be measured reliably even though it is not probable that
amount will be paid.
While both deferred and contingent considerations are measured at acquisition and their
date fair value recognized as part of cost of acquisition, the two differ as follows:

- While subsequent settlement of contingent consideration is dependent on the


occurrence of certain future events, settlement of deferred consideration is not
dependent on any future event.

- While subsequent measurement of deferred consideration, that is not equity is easier to


determine the value is arrived at by unwinding of discount, (systematic release of
discount to finance cost) subsequent measurement of contingent consideration that is
not equity is not straight forward as it is based on fair value which is determined in
accordance with IFRS 13.

- Whereas deferred consideration will surely be settled in the future, contingent


consideration may or may not be paid in the future.

- Contingent consideration is recognised at the date of acquisition as long as the fair value
can be measured reliably even though it is not probable that the consideration will be
paid (i.e. that the future event will occur). However, there is no such requirement for
deferred consideration.

EXAMINER'S REPORT

The question tests the candidates' knowledge of preparation of Consolidated statement of profit or
loss and other comprehensive income. Consolidated statement of financial position and the
distinction between deferred and contingent considerations.
Being a compulsory question, all the candidates attempted the question and performance was
above average.

Commonest pitfalls include:

- Lack of knowledge of how to analyse profit for the year and total comprehensive income
between owners of parent and NCI.

- Candidates inability to understand the difference between deferred and contingent


consideration.

- Poor calculation of goodwill, NCI, unrealised profit and retained earnings.


Candidates are advised to practice more questions before examination, as this is a regular
examinable section of the syllabus. Candidates are also advised to pay more attention to the
theoretical aspect of group accounting when preparing for future examinations.

MARKING GUIDE Marks Total


C Comments
(i) There is an increase of 3.11 kobo in earnings per share of the group. This is in
excess of 12kobo EPS of Wole Adura Plc and subsidiaries.
(ii) There is an increase of 0.31 times in dividend cover of the group. This is in excess
of 1.2 times Dividend Cover of Wole Adura Plc and subsidiaries.
(iii) There is an increase of 1.244% in earnings yield of the group. This is in excess of
4.8% earnings yield of Wole Adura Plc and subsidiaries.
(iv) The dividend yield for the two entities remains constant.
(v) The inclusion of the N70m share of profit in Maseru Associates impacted
positively on all ratios computed.
Pairwise Comparison

Wole Adura Plc. and Subsidiaries Changes


Group

EPS 12kobo 15.11kobo 3.11k

Dividend Cover 1.2 times 1.51 times 0.31 times

Earnings Yield 4.8% 6.044% 1.244%

Dividend yield 4% 4% -
D To discuss the liquidity challenges during the year ended December 31, 2017, the following
computations are necessary:
2017 2016 Change
Current Asset Ratio 300 - 80 115 - 20
300 - 60 115 40
0.92 1.27 (0.35)

Quick Ratio 220 - 200 95 - 40


300 - 60 115 -40
0.08 0.73 (0.65)

Both the current ratio and quick ratio reduced drastically in 2017 compared with 2016 by
28% and 89% respective probably due to:
(i) Bank overdraft of N50m compared with positive position of N30m in 2016.
(ii) Higher inventory tied up of N200m compared with only N40m in 2016.
(iii) Trade payables increased by 216% in 2017 compared with 2016 from N60m to
N190m.
(iv) Larger volume of business requires larger working capital which is lacking here. Working
capital dipped from N20m in 2016 to - N20m in 2017.

(v) Interest expense of N10m in 2017 as a result of heavy borrowing.

EXAMINER'S REPORT

The question tests the computation of ratios and the interpretation of same for decision making.
About 81% of the candidates attempted the question and performance was below average.
The commonest pitfall was the inability of the candidates to interprete the computed ratios
correctly. Also, some candidates could not correctly applied equity method to calculate the
earnings of parent in associate company.
Candidates are advised not to limit their knowledge only to computation of ratios, but also to
interpretations of financial information in order to improve their performance in future
examinations.
SOLUTION 3

(a) The conditions which must be met for a non-current asset to be classified

as being "held for sale" are:

(i) Management is committed to a plan to sell;

(ii) The asset is available for immediate sale;


(iii) An active programme to locate a buyer has been initiated;

(iv) The sale is highly probable;

(v) The sale should be completed, or expected to be so, within 12


months from the date of the classification;

(vi) The actions required to complete the planned sale would have been made, and it is
unlikely that the plan will be significantly changed or withdrawn;

(vii) The asset is being actively marketed at a sales price that is reasonable in relation to
its fair value.
Accounting treatment

• Assets classified as held for sale must be presented separately on the face of the
statement of financial position and included in current assets.

(b) (i) The condition which must be met in order to present the results of an
operation as "discontinued" are that the discontinued operation:

- represents either a separate major line of business or a geographical area of


operations;

- is part of a single co-ordinated plan to dispose of a separate major line of


business or geographical area of operations; or

- is a subsidiary acquired exclusively with a view to resale.

Accounting treatment

The total (or sum) of:

- the post-tax profit or loss of the discontinued operation, and


- the post-tax gain or loss recognised on the measurement at fair value less
costs to sell (or on the disposal) should be presented as a single amount in the
statement of profit or loss.

(ii) Bamgbose Plc


Statement of Profit or loss and other comprehensive income for the
year ended December 31, 2017
2017 2016
N'm N'm
Revenue 58.5 66
Cost of sales (19.5) (30)
Gross profit 39 36
Operating costs W2 (20.85) (19.5)

Profit for the year from continuing operations 18.15 16.5


Loss for the year from discontinued operations (8.7) -
W1
Profit for the year 9.45 16.5
Working W1
N'M
The post-tax loss of the discontinued operation 4.5
The post-tax impairment loss recognized on the measurement
of the buildings at fair value less costs to sell (30 - 25.8) 4.2
Loss for the year from discontinued operations 8.7

Working W2
Operating costs:
As per the accounts 15
Other expected cost of closure 5.85
20.85

EXAMINER'S REPORT
The question tests the knowledge of the provisions of "IFRS 5" and its application
in practical terms.

About 86% of the candidates attempted the question and performance was average.

Candidates' pitfalls was lack of understanding of the provisions of IFRS on


discontinued operations aspect of the standard and its practical application.

Candidates are advised to prepare very well for this stage of the examination and
completely cover the syllabus by using the ICAN Study Text which covers
extensively the required standards, including IFRS 5.

MARKING GUIDE Marks Total


(a) IFRS 5 conditions for an asset to be classified as "held
for sale" - any 6 points @ 1 mark each
Accounting treatment - any 2 points @ y2 mark

(b) i. IFRS 5 conditions for an operation to be reported


as "discontinued" - any 2 points @ 1 y2marks each

Accounting treatment - any 2 points @ 1 mark each

ii. Application: Statement of profit or loss and other


comprehensive income
Workings
Total
SOLUTION 4

(a) Substance over form states that economic or commercial substance of a transaction
override the legal form. Therefore, organization should always record the economic
substance of a transaction rather than their legal form.

The main problem of off-statement of financial position finance is that it results in financial
statements that do not faithfully represent the transactions and events that have taken
place. This implies that such financial statements cannot be relied upon and as a result, any
decision made on the basis of the information contained in the financial statements will be
incorrect and misleading.

How the major user groups can be misled


i. Lenders of capital are especially concerned about the entity's gearing position. When
the borrowing is high, it increases risk.

ii. Suppliers are concerned with liquidity position of the entity. The
existence of consignment inventories may be relevant to trade suppliers.
Sometimes, consignment inventories and their related current liabilities are not
recorded on the statement of financial position as the wording of the purchase
agreement may be such that legal ownership of the goods remains with the supplier
until specific event(s) occur.

iii. Investors are concerned with the entity's profitability performance.


Where borrowing is low, it becomes inexpensive and tax efficient with promising
returns to the shareholders.

iv. Management may not be able to use the asset to claim capital
allowance with the relevant tax authority, if it is off financial statement, thereby
understating asset by ignoring substance over form rules.

v. Employees are also interested in the financial statements because of their job
security. If a facility that is not recorded in the company's book attracts huge finance
charges, if this continues for a long period, it can negatively affect the organisation
and may lead to staff rationlisation in the future.

(b) i. Major accounting issues involved in the two transactions using the
principles of substance over form:
• Factoring
This is a common method of entities releasing the liquidity of their trade
receivables. The issue is whether the trade receivables have been sold, or
the income, from the finance house for their 'sale' should be treated as a
short-term loan.
The main "substance issue" with this type of transaction is to
identify which party bears the risk relating to the asset. If the risk lies with
the finance house (Asejere), the trade receivables should be removed from
the statement of financial position. In
this case, it is clear that Waasimi still bears the risk relating to
slow and non-payment of trade receivables. The residual payment by
Asejere depends on how quickly the receivables are collected, the longer it
takes, the less the residual payment. Any balance uncollected by Asejere
after six months will be refunded by Waasimi, which reflects the non-
payment risk.
• Sales and Leaseback of freehold building
This is sales and lease back transaction. The substance was that Waasimi
needed loan, if not, an asset cannot be sold and leased back immediately. It
implies that the excess purchase consideration of N4million (N24m - N20m)
is "in substance" a loan rather than sales proceeds (legal form) which is
being repaid through the excess (N1million per annum) of the rent
payments.
It should be indicated that the lease is an "Operating Lease", hence, the
property should be treated as sold and derecognised.

ii. Appropriate accounting treatment

Factoring
Cash received from Asejere (80% of the selected receivables) should be treated
as a current liability (a short-term loan).
The difference between the gross trade receivables and the amount received
from Asejere (plus any amount directly from the credit customers) should be
recognised in statement of profit or loss.
Sale of freehold property
Sale of the property should be recorded at its fair value (N20million).
Profit on disposal would be N5million (N20million - N15million).
The excess of N4million (N24million - N20million) should be treated as a loan
(Non-current liability).
iii. Classes of charges to be incurred
Factoring
Administrative expenses in the form of factoring charges for Asejere collecting
receivables.

Finance cost reflecting the time taken to collect the receivables. Impairment of

trade receivables (bad debts).

Sale of freehold property


The total rental payment of W2.6 million should be split into three elements as:

Property rental cost N1.6m

Finance cost (10% of &4m) N0.4m

Capital repayment of the loan N0.6m

N2.6m

EXAMINER'S REPORT
The question tests the principle of substance over legal form and the accounting issues in the
principle of substance over form; the relevant accounting treatments; and classes of charges
involved in the transactions

About 29% of the candidates attempted the question and performance was below average.

Majority of the candidates were not familiar with the provisions of IFRS as it relates to "factoring",
sales and lease back etc.

Candidates are advised to pay more attention to the provisions of IFRSs as well as their practical
applications.

MARKING GUIDE Marks Total

(a) Explanation of substance over form 11/2


How major user groups can be misled with examples 41/2

6
(b) i. i. Major accounting issues 5
ii ii. Accounting treatments 5
iii. Classes of charges 3
14
Total Marks 20

SOLUTION 5
(a) The two perspective of identifying deferred tax balances are:
i. A statement of comprehensive income (income and expenses) perspective:

• The differences arising in the period are identified by comparing income and
expenses recognized under IFRS to the equivalent figures that are taxable or
allowable under tax legislation; and

• The approach identifies the deferred tax expense or credit recognized in the
statement of comprehensive income for the period (with the other side of the
entry recognised as a liability or asset).

ii. A
statement of financial position (assets and liabilities) perspective:
• The differences are identified on a cumulative basis by comparing the carrying
amount of assets and liabilities under IFRS to the carrying amount of the same
assets and liabilities according to the tax rules; and

• The approach identifies the deferred tax liability (or asset) that should be
recognised (with the movement on this amount recognized as a credit or
expense in the statement of comprehensive income).
(b)
Carrying Tax base Temporary
amount difference
Non-current assets N N N
Plant and equipment 250,000 218,750 31,250
Receivables
Trade receivables 62,500 68,750 (6,250)
Interest receivable 1,250 - 1,250
Payables
Fine 12,500 12,500 -

Interest payable 2,500 - (2,500)


Temporary Deferred tax @ 30%
Differences
Deferred tax liabilities 32,500 9,750
Deferred tax assets (8,750) (2,625)
7.125
Deferred tax @ 30%

Deferred tax as at January 1, 2017 1,500

Profit or loss balancing 5,625


figure
Deferred tax as at December 31, 2017 7.125

EXAMINER'S REPORT
The question tests candidates' knowledge of computation of deferred tax provision in line with IAS
12.
About 52% of the candidates attempted the question and the performance was below average.
Candidates’ lack of adequate knowledge of the concept of deferred taxes lead to the inability to use
the data given to accurately compute and present deferred tax provisions.
Candidates are advised to ensure a good coverage of the syllabus at this level of the Institute
examination.
SOLUTION 6

(a) Treatment of foreign transactions and gain or loss arising therefrom

A company can have transactions that are denominated in foreign currency. These must be
translated into the company's functional currency for the purpose of recording the
transactions in its books of accounts and preparing its financial statements in accordance
with guidelines provided in IAS 21.

S/N Items or period of Applicable Treatments of gain or loss


translation or transaction rates
1. On initial recognition The spot rate on No exchange difference will arise,
the date of the therefore, no gain or profit.
transaction
2. Balances at the beginning of Opening rate Any exchange difference should be
the year in statement of recognized in profit or loss.
financial position
3. All items for statement of profit Average rate Any exchange difference should be
or loss and other recognized in profit or loss.
comprehensive income and
where the exchange rate does
not fluctuate significantly over
the period
4. Exchange difference before Rate ruling on the Exchange rate difference to be
year end day of exchange recognized in profit or loss.

5. For year end balances the


following rules apply:

i. Monetary items i.e current Closing rate Any exchange gain or loss that arises
assets or current liabilities should be recognized in profit or loss.

ii. non-monetary items such as Historical rate Therefore, no exchange


non-current assets, or rate at difference will arise.
depreciation non-current acquisition
liabilities
iii. non-monetary items Date of Gain or loss should be recognised in
measured at fair value determining the profit or loss and Other
fair value Comprehensive Income (OCI)

(b) How to account for investment property


i. Omotola Nigeria Plc. will have to translate the investment property into its
functional currency (Naira) on purchase at the end of the year.

ii. The recording in the books will be as follows:

• On the initial recognition, the $1 million should be translated at the spot rate on
May 1, 2017.

• At April 30, 2018, the fair value of $1.2m should be translated using closing rate
at the date of determining the fair value.

• Any exchange difference arising from this property should be recognized in


profit or loss.

(c) Accounting treatment for wholly owned subsidiary and other equity investment

The financial statements of the subsidiary together with the goodwill arising on the
acquisition of the subsidiary will be treated as follows:

i .Treatment of acquisition of wholly owned subsidiary

• On acquisition use spot rate or the rate on the date of acquisition.

• At the year-end, on April 30 2018, use closing rate.

• Any exchange difference arising on this exercise will be recognised in other


comprehensive income (OCI) and accumulated in equity through other
component of equity (OCE).
ii. Treatment of equity investment
• The equity investment at fair value through other
Comprehensive income (FVOCI) will be translated at initial recognition using
spot rate at the date of initial recognition - May 1, 2017.

• At April 30, 2018, the fair value of this investment will be translated using
the exchange rate at the date of determining the fair value.

• Any exchange gain or loss arising from the translation will be recognised in
OCI.

EXAMINER'S REPORT

The question tests practical applications of IAS 21 - Effect of Changes in Foreign Exchange Rate; IAS
40 - Investment Property and IFRS 9 - Financial instrument

About 69% of the candidates attempted and the performance was below average.
Commonest pitfalls were the inability of the candidates to discuss the treatment of foreign currency
transactions and their inclusion in the financial statements.

Candidates are advised to study all accounting standards and understand their applications.

MARKING GUIDE
(a) Marks Total
Functional currency 1
- Foreign currency 1
- Applicable exchange rates 3%
- Treatment of exchange difference 11/2 7

(b) Translation to functional currency (Naira) 1

Recognition and presentation 3 4

(c) i. Accounting treatments of wholly owned subsidiary 2

ii. Accounting treatments of equity investment 2 4


Total Marks 15

SOLUTION 7

(a) Environmental reports are reports that give details of the entity's activities and its impacts in
the environment where they operate and the efforts of the entity to reduce and clean up
the impact.

The following are the principal arguments against voluntary disclosures:


i. Voluntary disclosures are of limited usefulness as they are not readily comparable
with other reporting entities;

ii. Voluntary information may not be audited and therefore the reliability of the
information is questionable and makes it less useful to decision makers;

iii. The lack of comparability may mean that the cost of producing the information
outweighs the potential benefits to shareholders;

iv. Any cost incurred will reduce profit and subsequent potential returns to
shareholders;

v. Voluntary disclosures are part of the annual report which implies that there is a risk
of information overload and where this occurs, the relevance and usefulness of the
information is reduced;

vi. The disclosure of wrong environmental report may be used against the entity if
contended in a law court;

vii. Since voluntary disclosures are not regulated by laws, disclosures tend to lack
standardization;

viii. Information overload in voluntary disclosures may lead to window dressing (creative
accounting); and

ix. Voluntary disclosures will state the type of materials the business uses in the course
of their production, but will not mention the adverse effect of the chemical
components of these materials on the community/environment., for example:

■ In an oil and gas company like in this question, there will be water pollution that
will adversely affect the community/environment vis a vis the farming/fishing of the
community.

■ Gas emission from power generation constitute health hazard but voluntary
disclosure will not mention this.

■ Movement of heavy tractors/vehicles for the purposes of the business will make
the road to wear out on time and this will affect the well being of the community
which voluntary disclosure will never mention.
(b) Information that could be disclosed by entities in their external reports in order to comply
with the Global Reporting Initiative (GRI) guidelines are as follows:
i. The economic aspects are likely to contain information about how the
entity impacts on the economic conditions of its stakeholders and the
area in which it operates globally;

ii. An entity may also include its policies regarding local and global economies and disclose
targets and its performance to date;

iii. The environmental aspect provides information about how an entity impacts on the
environment and management's policies on waste, emissions and pollution;

iv. Targets on wastage, emission and pollution are likely to be set and
strategies for achieving these and performance to date could also be
included;

v. The social aspect relates to the impact the entity has on the social systems in which it
operates in relation to human rights, labour practices including employer/employee
relations, occupational health and safety precautions; and

vi. There could also be information on research and development in respect of technologies
for environmental conservation and environmentconscious products and services.

Global Reporting Initiative (GRI) is a reporting guideline issued by the Sustainability


Accounting Standards Board to entities on how to measure and report on management's
approach to the economic, environmental and social aspects that impact on their
businesses.
EXAMINER'S REPORT
The question tests the candidates' understanding of the principal arguments against voluntary
disclosures and information that could be disclosed in external reports in respect of the economic,
environment and social aspects.

About 64% of the candidates attempted the question and the performance was below average.

Commonest pitfall was that most of the candidates failed to highlight the principal arguments
against voluntary disclosures.
Candidates are advised to cover all sections of the syllabus for better performance in future
examinations.

MARKING GUIDE
Marks Total

(a) Identification of principal arguments 4


Explanation of principal arguments 4 8

(b) Information disclosures in external reports:


- economic 2
- environmental 2
- social 2
Description of GRI 1 7
15
THE INSTITUTE OF CHARTERED ACCOUNTANTS OF NIGERIA

PROFESSIONAL LEVEL EXAMINATION - NOVEMBER 2018

CORPORATE REPORTING

Time Allowed: 3 1/4 hours (including 15 minutes reading time)


INSTRUCTION: YOU ARE REQUIRED TO ANSWER FIVE OUT OF SEVEN
QUESTIONS IN THIS PAPER

SECTION A: COMPULSORY QUESTION (30 MARKS)

QUESTION 1

Adegaga Laboratories Plc ("AdeLabs") is one of the largest companies in Nigeria engaged in cosmetic
development and manufacturing. Its largest customer base is in the healthcare sector for post-
surgery patients and the Nigeria movie industry (aka Nollywood). In the prior financial period,
AdeLabs' expansion strategy has been largely focused on growth by acquisition and joint ventures.

Additional Information:

(i) As part of this, AdeLabs acquired 80% of the equity share capital of Bodegas limited
("Bodegas") on January 1, 2015 when the retained earnings of Bodegas was N93.75 million.
Following the share acquisition, AdeLabs had control over Bodegas - no shares have been
issued by Bodegas following the acquisition on January 1, 2015. The non-controlling interest
in Bodegas was measured at its fair value of N20 million at the date of acquisition.

(ii) On January 1, 2016, AdeLabs acquired 50% of the equity share capital of ChidePlastics
limited ("ChidePlast") when the retained earnings of ChidePlast was N41.25 million. This
acquisition was classified as a joint venture in accordance with IFRS 11 Joint Arrangements.
ChidePlast has not issued any shares since the acquisition date.

(iii) The balance on "other reserves" relates to movements in the values of investments in
Bodegas and ChidePlast in the books of AdeLabs. N18.75 million relates to Bodegas and the
remainder to ChidePlast.

(iv) AdeLabs' non-current liabilities relate to a borrowing (long-term) taken out on January 1,
2017. This borrowing has an agreed coupon rate of 4% p.a and the interest expense due in
respect of 2017 has been paid and accounted for in profit for the year. The effective interest
rate estimated with this financial liability is 8% p.a.
(v) As part of its annual impairment review, AdeLabs concluded that the goodwill on the
acquisition of Bodegas was impaired by 20% at December 31, 2017. No other impairments
of goodwill have arisen.

(vi) AdeLabs sold goods to ChidePlast with a value of N75 million and a selling margin of 40% in
November 2017. As at year end December 31, 2017, 75% of these items are unsold.

Accounts for all companies are made up to December 31 annually.

The draft statements of financial position for AdeLabs, Bodegas and ChidePlast
as at December 31, 2017 are as follows:

ASSETS ADELABS BODEGAS CHIDEPLAST


N’000 N’000 N'000
Non-current assets
Property, plant and equipment 300,000 262,500 225,000
Investment in Bodegas 262,500 -
Investment in ChidePlast 150,000 -

712.500 262.500 225.000


Current assets 112,500 150,000 75.000
Total assets 825.000 412.500 300.000
EQUITY AND LIABILITIES
Equity
Share capital (N1 ordinary
shares) 225,000 150,000 112,500
Share premium 75,000 18,750 18,750
Other reserves 26,250 - -
Retained earnings 243.750 161.250 93.750
Total equity 570,000 330,000 225.000
Non-current liabilities 187,500
Current liabilities 67.500 82,500 75.000
Total liabilities 255.000 82.500 75.000
Total equity and liabilities 825.000 412.500 300.000

Required:

Prepare for Adegaga Laboratories Plc:

a. A consolidated statement of financial position as at December 31, 2017.


(20 Marks)

b. On January 1, 2018, AdeLabs acquired an additional 10% of the equity


shares of Bodegas. The purchase consideration for this additional acquisition
was N52, 500,000.
Required:

i.
Briefly explain how this additional acquisition will impact on the
preparation of AdeLabs consolidated financial statements for the year ended December 31,
2017. (4 Marks)

ii. Calculate the adjustment that willbe required to be made to AdeLabs'


statement of financial position as aresult of this acquisition. (6 Marks)
(Total 30 Marks)

SECTION B: YOU ARE REQUIRED TO ANSWER TWO OUT OF THREE QUESTIONS


IN THIS SECTION (40 MARKS)

QUESTION 2

(1) KutuKutu Plc. has a policy in place to pay its employees a performance bonus. This bonus is to be paid in
cash and is unrelated to the movement in its share price. KutuKutu Plc. has the choice of settling the
bonus in cash or in equity shares to the value of the cash bonus. Based on bonuses paid in prior years,
KutuKutu Plc. has always settled the bonuses of qualified employees in shares.

(2) KutuKutu Plc. grants one share option to each of its 50 employees on January , 2016. The share options
1

will vest at the end of 2 years provided that:

The employees remain in KutuKutu Plc.'s employment at that date; and the Earnings before Interest,
Tax, Depreciation and Amortisation (EBITDA) of KutuKutu Plc. for the second year achieves a specified
target.

At the grant date, each recipient is required to make a non-refundable cash payment of N2,000 to KutuKutu
Plc. This payment is based on the estimated fair value (FV) of the share option which reflects the probability
that the target EBITDA will be achieved in the second year. FV of each option would be N250 (excluding the
effect of the EBITDA condition).

If KutuKutu Plc. does not achieve the target EBITDA or if an employee leaves the employment of KutuKutu Plc.,
no shares will be issued and the employee will not be entitled to a repayment. Accordingly, both service and
non-market vesting conditions are deemed to be substantive.
Required:

a. Do the transactions entered into in (1) above meet the definition of equity- settled share
based payment transactions within the scope of IFRS 2?
(8 Marks)

b. How should the transaction in (2) above be accounted for? (12 Marks)
(Total 20 Marks)

QUESTION 3
Banny Plc. (Banny) is a diversified company that has achieved its present size through vertical and
horizontal acquisition. The directors have identified two potential target entities for acquisition. The
first is Abana Limited (Abana) which is into cement business near Offa, Kwara State. The second is
Doha Limited (Doha) which is also into cement business near Oturukpo, Benue State. Banny has
obtained copies of their audited financial statements. Extracts of these financial statements
together with notes providing additional information are given below.
Notes:

(1) Doha revalued its non-current assets for the first time following the adoption of IFRS on
January 1, 2017. The non-current assets of Abana are very similar in age and type to the non-
current assets of Doha. However, Abana has a policy of maintaining all its non-current assets at
depreciated historical cost. Both entities charge depreciation of non-current assets to cost of
sales. Abana has transferred the excess depreciation on the revalued assets from the
revaluation reserve to retained earnings as permitted by IAS 16 - Property, plant and
equipment.
Banny uses ratio analysis to appraise potential acquisition target and bases its judgement
using four key ratios:
• Return on capital employed;
• Gross profit margin;
• Turnover on capital employed; and
• Leverage.

For the purpose of the ratio analysis, Banny computes capital employed as capital and
reserves plus borrowings, long- term plus short- term.
Your assistant has computed the four key ratios for the two entities from the financial
statements provided and concluded that Abana is a better target for acquisition than Doha.
However, you are not sure, given the information on the revaluation of its non-current
assets as stated above.

Required:

a. Compute and explain the adjustments that would be appropriate in


respect of Note 1 on revaluation of property, plant and equipment so as to make
the financial statements of Abana and Doha comparable for analysis. (14 Marks)

b. Calculate the four ratios used by Banny for both Abana and Doha
after making the adjustments you haverecommended in your answer
to part (a). (4 Marks)

c. In the light of your analysis in (a) and (b) above, advise Banny which
of the two companies is a better target for acquisition based on the
Adjusted ratios. (2 Marks)
(Total 20 Marks)

QUESTION 4

a. Happy is a publicly listed company. Its financial statements for the year ended July 31, 2017
including comparatives are shown below:
Notes:

(i) On November 1, 2016, Happy acquired an additional plant under a finance lease that had a
fair value of N3 million. On this date, it also revalued its property upwards by N4 million
and transferred N1.3 million of the resulting revaluation reserve to deferred tax. There
were no disposals of non-current assets during the period.

(ii) Depreciation of property, plant and equipment was N1.8 million and amortisation of the
deferred development expenditure was N400,000 for the year ended July 31, 2017.
Required:

Prepare a statement of cash flows for Happy for the year ended July 31, 2017, in accordance
with IAS 7 Statement of Cash Flows, using the indirect method. (10 Marks)

b. Compliance with the fundamental principles as expressed in ICAN's Code of


Ethics, may potentially be constrained by a range of circumstances among which can be
threats.

Required:
As a Consultant to the management of Gatuso Plc., write a draft report to identify and
evaluate the various threats to the fundamental principles that might disrupt its objectivity
and give circumstances that can create such threats. (10 Marks)
(Total 20 Marks)

SECTION C: YOU ARE REQUIRED TO ANSWER TWO OUT OF THREE QUESTIONS IN


THIS SECTION (30 MARKS)

QUESTION 5

Atigen Manufacturing Limited bought a new machine for its factory in Otta, Ogun State for N140
million on January 1, 2015. At acquisition, the machine was estimated to have a life span of 7 years
with no scrap value. The carrying amount at December 31, 2017 is N80 million.

The machine generates largely independent cash flows and therefore is tested for impairment as a
stand-alone asset. Due to a downturn in the economy and reduction as well as cancellation of major
customer orders due to changes in the market place, the directors concluded that the machine may
be impaired. You are provided with the following information:

Fair value of the machine: N60 million;

Selling costs is 5% of the fair value; and

Value-in-use based on discounted future cash flows is N-63.5 million.


Required:

a. Determine if the machine is impaired based on the above information.


(6 Marks)

b. Calculate (if any) the impairment charge, the directors should take to profit or
loss. (9 Marks)
(Total 15 Marks)

QUESTION 6

You have been asked to make a presentation to your team on cryptocurrencies. A snapshot of your
draft presentation includes the following:

"Cryptocurrency is a new phenomenon in the financial market. A cryptocurrency is a digital or


virtual currency designed to serve as a medium of exchange.

Cryptocurrencies are created through cryptography, often with a maximum possible number of
'coins' that can exist through solutions to a complex algorithm with their value supported only by
the laws of supply and demand. Cryptocurrencies are currently not regulated by government or
other similar entity.

The following are some of the types of cryptocurrency in the market:

Bitcoin - The first ever cryptocurrency that started the market awareness and "boom";

Ethereum - A programmable currency that lets developers build different distributed apps and
technologies that wouldn't work with Bitcoin; and

Ripple - Unlike most cryptocurrencies, it doesn't use a Blockchain in order to reach a network-wide
consensus for transactions. An iterative consensus process is implemented, which makes it faster
than Bitcoin but also makes it vulnerable to hacker attacks.

There are a lot of merchants - both online and offline - that accept Bitcoin as the form of payment
while Ethereum and Ripple, are not widely accepted yet.

Required:

Following your presentation, you are asked how a holding of cryptocurrency should be classified in
the financial statements of your clients. (15 Marks)
(Total 15 Marks)
QUESTION 7

a. Agbero plc is a public company rendering services to Lagos State Government especially in
the area of public transport. Agbero is listed on the Lagos Stock Exchange. In its annual
financial statements for the year ended March 31, 2018, Agbero plc had identified the
following operating segments:

Segment 1 local bus operations;

Segment 2 inter-city bus operations, and

Segment 3 road constructions.

The company disclosed two reportable segments. Segments 1 and 2 were aggregated into a
single reportable operating segment. Operating segments 1 and 2 have been aggregated on
the basis of their similar business characteristics, and the nature of their products and
services. In the local bus operations market, it is the Lagos state local transport authority
which awards the contract and pays Agbero plc for its services. In the local bus operation
market, contracts are awarded following a competitive tender process, and the ticket prices
paid by passengers are set by and paid to the Lagos state transport authority. In the inter-
city bus operation market, ticket prices are set by Agbero and the passengers pay Agbero for
the service provided.

Required:

i. Advise Agbero plc on how the above accounting issues should be dealt
with in its financial statements. (6 Marks)

ii. Although the company is happy with IFRS 8-Operating segment, hence its desire to
comply with the above, it is not clear about who to designate as its chief operating
decision maker in line with the standard. Define Chief operating decision maker in
line with IFRS 8 - Operating Segment and identify who this should be in Agbero plc.
(2 Marks)
b. Agbero Plc entered into a contract with Lagos State Government on April 1,
2016 to undertake construction of a new light railway line. The total
Revenue from the contract is N5 billion over a three-year period.
The contract states that N1 billion will be paid at thecommencement of the
contract but although invoiceswill be subsequently sent at the end of each
year, the Lagos State Government will only settle the subsequent amounts owing when the
contract is completed. The invoices sent by Agbero Plc to date (including N1 billion above)
were as follows:

Year ended March 31, 2017 N28 billion


Year ended March 31, 2018 N12 billion
The balance will be invoiced on March 31, 2019. Agbero Plc has only accounted for the initial
payment in the financial statements to March 31, 2017 as no subsequent amounts are to be
paid until March 31, 2019. The amounts on the invoices reflect the work undertaken in the
period.

Required:
Agbero Plc. wishes to know how to account for the revenue on the contract in the financial
statements to date. Advise Agbero Plc.

Market interest rates are currently at 6%. (7 Marks)


(Total 15 Marks)

SOLUTION 1
a.
ADEGAGA LABORATORIES GROUP
CONSOLIDATED STATEMENT OF FINANCIAL POSITION
AS AT DECEMBER 31, 2017
N'000
Non-Current Assets

Property, plant and equipment (300,000 + 262,500) 562,500


Goodwill wk3 1,000
Investment in associate (Chideplast) wk6 157,500
Total non-current assets 721.000

Current Assets

Current assets (112,500+150,000) 262,500


Total assets 983.500

Equity & Liabilities

Equity
Share capital 225,000
Share premium 75,000
Retained earnings wk5 305,050
Non-controlling interest wk4 33,450
Total equity 638,500
Non-current liabilities (187,500+7500) wk9 195,000
Current liabilities (67,500+82,500) 150.000

Total equity & liabilities 983.500

Wk 1 Control structure

Bodegas

WK 2 Fair value of net asset of subsidiary Bodegas

At acquisition Date At reporting date

1/1/15 31/12/17
N'000 N'000
Share capital Share 150,000 150.0
premium Other 18.750 18,750
reserves Retained
earnings Fair value 93.750 161.250
262.500 330.000
Alternative calculations
Net assets of the subsidiary - Bodegas

Reporting Acquisition Post


Date Date Acquisition
N'000 N'000 N'000
Share capital 150,000 150,000 -
Share premium 18,750 18,750 -
Retained earnings 93,750 161,250 67,500
262.500 330.000 67.500

Wk 3 Goodwill
N'000
Fair value of investment in Bodegas as at 31/12/17 262,500
Less: increase in FV recognised in other reserves (18.750)
Purchase consideration as at 1/1/2015 243,750
Non-controlling interest 20% 20.000
Total consideration 100% 263,750
Fair value of net asset at acquisition wk2 (262.500)
Goodwill at acquisition 1/1/15 1,250
Impairment (250)
Goodwill at reporting date 1.000

Wk 4 Non-controlling interest N'000

Fair value of NCI at acquisition 20,000


Share of profits (20% x 67,500) (wk6) 13,500
Share of impairment in subsidiary (20% x 250) (50)
Fair value at reporting date 33.450

Wk 5 Group retained earnings

Parent 243,750
Interest on borrowings (7,500)
Share of impairment in subsidiary (80% x 250) (200)
Share of profits in (Bodegas) (80% x 67,500) 54,000
Share of profits in (Chideplast)(50% x 52,500) 26,250
Unrealized profit on sales to chideplast wk 6 (11,250)
Retained earnings at reporting date 305,050
Wk 6 Joint Venture

Cost of investment in Chideplast 150,000


Less Fair value gain recognized in other reserve (7,500)
Unrealised profit on sales (75f000x75%x40%x50%) (11,250)
Share of profit in Chideplast 26,250
Joint venture at reporting date 157.500

WK 7 Fair value gain recognized in other reserves

Adelabs cannot carry out fair valuation of investment in Bodegas and Childeplast after gaining
control in Bodegas on 1/1/2015; similarly, Chideplast is an equity accounted investment, therefore
fair value gain would have to be derecognized as follows:
N'000 N'000
Dr Other reserves 26,250
Cr Investment in Bodegas 18,750
Cr Investment in Chideplast 7,500

WK 8: Unrealized profit on goods sold to Chideplast needs to be accounted for


and deducted from investment in Chideplast to the tune of control, i.e N75,000x
at 75% unsold x 40% margin x 50% control = N11,250,000.
N'000 N'000
Dr Retained earnings of parent 11,250
Cr Investment in Chideplast 11,250

Wk 9: Financial Investment measured at amortized cost

Opening balance Nominal Effective Closing balance


Interest (4%) Interest 8%
N'000 N'000 N'000 N'000
187,500 (7,500) 15,000 195,000

Debit N7,500,000 to Adelab Group retained earnings and credit financial liability
N7,500,000.
ALTERNATIVE
N'000
Long term borrowings 187,500
(Difference of unpaid Interest (8% - 4%) x N187,500,000 7,500
195.000

b. (i) Step acquisition where control already exist

This is step acquisition where control already exists. IFRS 3 - Business combinations
(revised) states that where control already exists, parent needs to account for
additional acquisition as mere change in shareholding between shareholders. In
relation to Adelabs, Adelabs control Bodegas by 80% on 1/1/15 but stepped up its
acquisition by acquiring 10% more on 1/1/2018 after the year end. This will increase
holding to 90% while NCI will become 10%

This falls under IAS 10 - event after reporting date as it occurred after reporting date but
before the approval of financial statements.

Event after reporting date are event both favourable and unfavourable that occurred after
reporting date but before the approval date. It can be an adjusting or non-adjusting event.

In this case, this is non-adjusting event therefore it requires only disclosure to the financials
statement as at 31/12/2017.

ii. In 2018 the following adjustments will be required to be made to Adelabs' statement of
financial position:

N'000 N'000
Dr. Investment in Bodegas Cr. 52,500
Bank account 52,500
Being Adelabs additional 10% acquisition of Bodegas on January 1, 2018.

EXAMINER'S REPORT
The question tests candidates' knowledge on the preparation of consolidated statement of financial
position and the impact of additional acquisition on the group and parent separate statement of
financial position.
Candidates demonstrated poor understanding of the question, as only few of them understand
clearly the difference between subsidiary, associate and joint venture relationships.
Commonest pitfall was that candidates were consolidating the figures of chideplast (joint venture)
with those of the parent and subsidiary in Part (a) of the question.
Candidates could not demonstrate adequate knowledge of step acquisition and additional
acquisition after control has been achieved by the parent.
Candidates are advised to prepare adequately to internalize the principles involved generally in
group accounts as this is consistently tested in this syllabus.

SOLUTION 2

a. According to IFRS 2 - Share based payments. An entity should recognize the


financial effects of all share based payment transactions in its financial statement.

Share based payment transaction is a transaction in which an entity issues shares or share
options as consideration for goods or services rendered. It also includes a transaction
where an entity acquires goods or services and agrees to pay cash which will be based or
influenced by price of the entity's share.

It simply infers that an organization, i.e. Kutukutu Plc uses its shares as a basis of payment
for the services rendered by employees or goods purchased. It can be either equity settled
or cash settled.
Equity-settled is where organization used their own shares in exchange and will be
recognized like this:

Dr Profit or loss over the vesting period


Cr Equity options with total amount

Whereas cash-settled will be recognized thus:

Dr Profit or loss account


Cr Liability

As regards Kutukutu Plc, the bonus is not cash-settled due to the facts that

• Performance bonus is to be paidin cashbut the bonus payment is


unrelated to the movementin its share price.

At this juncture, it must be mentioned that the transactions entered into in(i) does not meet
the definition of equity - settled share based payment transactions within the scope of 1FRS
2.

b. 1FRS 2 explains that the share based payment transaction should be measured at the fair
value of the goods or services received. 1f fair value of goods and services cannot be
measured reliably then the entity should measure the transaction at fair value of the equity
instruments granted.

This transaction should be accounted for as equity-settled share based payment


arrangement and this falls within the scope of 1FRS 2.

The postings in the books of Kutukutu Plc will be as follows:

Year 1

Dr Expenses (P or L) N50,000
Cr Equity N50,000

Year 2

Dr Expenses (P or L) N100,000
Cr Equity N100,000

Kutukutu Plc

Statement of profit or loss for the period ended December 31,

2017 2016
Expenses N50.000 N50.000
Kutukutu Plc

Statement of financial position as at December 31,

2017 2016

Equity Components N100.000 N100.000

Workings
Year 1: 50 staff x N2,000 x y2
Year 2: 50 staff x N2,000 x 2/2 N50,000
N50,000

Assumptions:

No employees left the organization (Kutukutu Plc) during the two years in question.

Fair value (FV) of each option (N250) is considered constant at the grant date (i.e. date of
agreement between employees and Kutukutu Plc)

EXAMINER'S REPORT

The question tests candidates' knowledge of the conditions and the provisions of equity-settled
share based payment transactions within the scope of IFRS 2 - Share based payments and how to
account for the transactions in the financial statements.

About 36% of the candidates who wrote the paper attempted the question and performance was
poor.

The candidates' commonest pitfalls were poor understanding of IFRS 2 and their inability to
differentiate between equity-settled share-based payment and cash- settled share- based payment
transactions.

Candidates are advised to study in-depth all the applicable IFRS/IAS for better performance in future
examinations.

Marking Guide

Marks Marks
a. Features of equity-settled share-based under IFRS 2 2
Features of cash-settled share-based under IFRS 2 2
Journal entries for equity and cash-settled share-based
payments 2
Relating the features to Kutukutu Plc 2 8

b. Accounting treatment for equity-settled share based payment for:


Year 1 3
Year 2 3
Showing the expenses in the statement of profit or loss of
Kutukutu Plc 2
Showing the equity components in the statement of
2
financial position of Kutukutu Plc
2 12
Correct assumptions stated
20
Total

SOLUTION 3

a. For comparison to be thorough, the basis of accounting of the two companies


Doha and Abana should be the same, hence Abana using historical cost and Doha using IFRS
as first adopter should be harmonised.

(i) Reduce non-current assets by N10 billion


(ii) Reduce the revaluation reserves to NIL
(iii) Reduce the cost of sales by N2 billion for the excess depreciation as a result of the
revaluation.
(iv) Increase gross profit by N2 billion

Based on the foregoing, the new figures for Doha are as follows:
c. Abana is better off in terms of leverage, ROCE and gross profit margin
compared with Doha, consequently, Abana would be a better investment for acquisition
purposes.

The adjustment of revaluation which was carried out to make the financial statements of
the two companies comparable makes it a bit more difficult to decide which entity Banny
plc. should target. After this adjustment, the asset turnover ratio of both companies appears
to be the same at 1.6 times, meaning none is more efficient than the other in the use of the
assets to generate income.

Abana has a higher gross profit and return on capital employed when compared to Doha.
The main reason may be that Doha has a higher other operating expenses than Abana. This
is because the turnover figures of both companies are nearly identical.
Where Abana has an advantage over Doha is in the leverage ratio. Leverage of both entities
has increased but more so in the case of Doha. The question is whether the directors will
attach more importance to leverage in their investment decision.Consideration as to whether this influences
directors' decision depends on whether they intend to change the financial structure of the
company.

Overall, it would appear that Abana would be a better investment on account of


profitability. However, it will not make much difference a decision as to choose between the
two entities given the close nature of their financial figures. However, this exercise shows
the importance of adjusting financial statements to achieve uniform accounting policies
when making investment decisions. It is notable that Doha's revaluation is permissible but
can impact on ratios if not adjusted.

EXAMINER'S REPORT
The question tests candidates' knowledge on revaluation of non-current assets for the first time
following the adoption IFRS and the computation of ratios after making adjustment on the
revaluation.
About 71% of the candidates attempted the question. The candidates did not have clear
understanding of the question and therefore the performance was generally below average.
The commonest pitfall was the inability of the candidates to compute adjustments to the financial
statements before computing the ratios.
Candidates should ensure full coverage of the syllabus to have in-depth knowledge for better
performance in future examination.

Marking Guide Marks Marks

a. Computation of adjustment necessary to make financial


statement of Abana and Doha comparable:

State all the adjustments 4


Computation of new figures 10
14

B Ratio computation for Doha and Abana

ROCE 1
Gross profit 1
Turnover of capital employed 1
Leverage 1

4
C Advice and conclusion 2
Total 20
SOLUTION 4

Happy Plc Statement of Cash Flow for the year ended July 31, 2017
Depreciation (1.8)
26.6
Paid** 14
Closing balance 28.0

3 Dividend paid
Retained profit before 3.5
PAT 40
Dividend paid (1.1)
6.4
4 Lease rental paid
Opening: finance non-current 1.8
Opening: finance current 1.2
Property, plant and equipment 3.0
6.0

Closing: finance non-current 2.4


Closing: finance current 1.5

(3.9)

Lease paid 2.1

5 Tax paid
Opening balance 3.05
Tax provision 2.0
New PPE (Deferred tax) 1.3
Closing balance (5.5)
Tax paid 0.85

b. Date
To: The management of Gatuso Plc

From: The consultant

Subject: DRAFT REPORT TO IDENTIFY AND EVALUATE THREATS TO


FUNDAMENTAL PRINCIPLE

Below are the possible threats to the fundamental principles that might disrupt its
objectivity and the circumstances that can create such threats.

Evaluation of threats to the fundamental principle

1. Self interest threat


- Financial interest
- Incentives compensation
- Inappropriate use of corporate assets
- Concern over employment security
- Commercial pressure from outside
2. Self review threat
- Members not willing to fault themselves on records
prepared by them
- Business decision data being subject to review by same
accountant.
3. Advocacy threat
- The professional accountant needs to promote the
organisation position.
- Need to window dress accounts
4. Familiarity threat
- Long association with business contacts
- Immediate close family members who benefit from the
influence of the professional accountants.
5. Intimidation threat
- Professional accountants conduct is influenced by fear
- Threat of dismissal or replacement
- Presence of dormant personality influencing
decisions.
If you desire further explanation in any submission above, kindly contact us.

Thanks

The Consultant

EXAMINER'S REPORT

The question tests candidates' knowledge on the preparation of statement of cash flows using the
indirect method in accordance with IAS 7 - Statement of Cash Flows. Part (b) of the question tests
the identification and evaluation of the various threats to the fundamental principles that might
disrupt its objectivity.
About 96% of the candidates attempted the question. Those who attempted part (a) of the question
demonstrated a fairly good understanding of the requirement. In Part (b), they mixed up the threats
with the fundamental principles of accounting.
Candidates' commonest pitfall was their inability to correctly derive the cash flows. Also, some were
not able to identify and evaluate the various threats to the fundamental principles in ICAN Code of
Ethics.
Candidates are advised to prepare very well for future examinations using the Institute's study
texts and pathfinder.

Marking Guide Marks Marks

a. Derivation of cash flow items and preparation of cash flow


statement:
- Operating activities 3
- Investing activities 3
- Financing activities 3
- Cash and cash equivalent 1 10

b. Identification of threats 5
Circumstances creating threats 5 10
Total 20
a. Determination of impairment charge to take to profit or loss.

N'000
Carrying amount 80,000
Recoverable amount 63.500
16.500

- Thus an impairment write-down of N16.5m (80m -


63.5m) is required.

- The accounting entries are as follows:


Dr. Profit or Loss (Admin expenses) N16.5m
Cr. Property, plant and equipment N16.5m
Being impairment charge on the machine at
December 31, 2017

EXAMINER'S REPORT
The question tests the candidates' knowledge of impairment of a machine as a stand-alone asset
and the computation of impairment charge to be taken to statement of profit or loss.

About 97% of the candidates attempted the question. The candidates understand the requirements
of the question and their performance was above average.

However, major pitfalls were inability to apply the principles and theories in IAS 36 -impairment of
assets and making correct presentation of the figures.

Candidates are advised to be well acquainted with the requirements of the accounting standards
and its application to the real corporate world situation for better performance in future
examinations.

Marking Guide Marks Marks

a. Calculation to determine whether the machine is


impaired or not
-Determination of carrying amount 2
-Determination of fair value 2
-Determination of value -in-use 2 6

b. Determination of Impairment charges 6


Accounting entries 3 _9
Total 15
SOLUTION 6

A cryptocurrency is a digital or virtual currency that uses cryptography for security. A cryptocurrency
is difficult to counterfeit because of this security feature. Many cryptocurrencies are decentralized
systems based on blockchain technology, a distributed ledger enforced by a completely different
network of computers.

In classifying cryptocurrencies in the financial statements, holders of such currencies will need to
determine how to classify them based on whether they are:

Financial instruments - IAS 32


Intangible assets - IAS 38
Inventory - IAS 2

a. Financial Instruments - IAS 32 defines a financial asset as any asset that is


cash or equity instrument of another entity (e.g. shares of another entity). It can equally be
described as a contractual right to receive cash or another financial asset from another
entity; or to exchange financial instruments with another entity under conditions that are
potentially unfavorable to the entity.

Cryptocurrencies do not meet this definition because they do not give any contractual rights
to the holder.

In view of this, cryptocurrencies cannot be disclosed or classified as financial instrument in


the financial statements.

b. Intangible assets - IAS 38 defines an intangible asset as an "identifiable


non-monetary asset without physical substance". An asset is a resource that is controlled by
the entity as a result of past events and from which future economic benefits are expected.

The three critical attributes of an intangible asset are:

Identifiability

Control (power to obtain benefits from the asset)


Future economic benefits (such as revenues or reduced future costs).

Cryptocurrency meets the definition of an intangible asset because it has no physical


substance and is non-monetary.
In view of this, cryptocurrency can be classified as intangible asset in the financial
statements either at cost or, if there is an active market and the entity chooses to apply the
revaluation model, at fair value with changes in value recognized in other comprehensive
income (if above cost) or profit or loss (if below cost) as required by IAS 38.

c. Inventory - IAS 2 provides that inventories include assets held for sale in the ordinary course
of business (finished goods), assets in the production process for sale in the ordinary course
of business (work in progress), and materials and supplies that are consumed in production
(raw materials).

Cryptocurrency falls in the scope of IAS 2 and can be classified as inventory in the financial
statements.

EXAMINER'S REPORT
The question tests candidates' understanding of the treatment and presentation of cryptocurrencies
as a new phenomenon in financial market and its classification in the financial statements.

About 22% of the candidates attempted the question. The candidates displayed a poor
understanding of cryptocurrency as a contemporary current issue in financial engineering that is still
being subject to future debates.

Candidates' commonest pitfall borders on the inability to link the concept to relevant International
Financial Reporting Standards (IFRS) like IAS 38 - Intangible assets, IAS 2 - Inventory and IAS 32 -
financial instruments.

Candidates' should endeavour to familiarize themselves with contemporary issues in the financial
world by reading relevant journals and articles.

Marking Guide Marks Marks


Definition of cryptocurrency and general introduction 3
Different classification of cryptocurrency as:
- Financial Instruments - IFRS 9 4
- Intangible Assets - IAS 32 4
- Inventory - IAS 2 4

Total 15
SOLUTION 7

a. i. IFRS 8 - defines an operating segments as a component of an entity that


engages in revenue earning business activities, whose operating results are regularly
reviewed by the chief operating decision maker.
The standard permits or allow for one or more operating segments to be aggregated into a
single reportable segment provided certain conditions are met. These conditions are:

Principally, operating segments should have similar economic characteristics.

- They are expected to have the same future outcome with their similar characteristics.

Areas of similar characteristics to satisfy are as follows:

- Nature of the products and services;


- Nature of the production process;
- Type or class of customers for their products and services;
- Methods used to distribute their products or provide their services;
and
- The nature of regulatory environment.

Treatment of accounting issues in Agbero Plc


Information given reveals the following:
Segment 1 - Local bus operation - Serves passenger within the town Segment 2: Inter -
city bus operation - Serves passenger across the state. This shows the 2 Segments have
different classes or type of customers.

Regulatory environment:
Local bus operation:
- The Lagos State Local Transport Authority is vested with the power to award or
withdraw local bus operation contract to Agbero Plc, which is not a function of
passengers availability.

Intercity bus operation market:


- Decision to withdraw from a route in intercity operation rests with Agbero Plc,
based on availability of passengers and viability of the route.

Pricing: Based on their operating environments.


- Local Bus Operation: Ticket prices paid by passenger are set by the Lagos
Transport Authority, where Agbero Plc only collects commission. No economic
risk.
Inter-city bus operation: Agbero sets the ticket price paid by
passengers. revenue risk exposure is here.

The 2 segments do not have similar characteristics for aggregation; as such they
should be reported separately.

i. According to IFRS 8:
- Chief operating decision maker is often the Chief Executive Officer (CEO) or Chief
Operating Officer (COO). It is a function and not necessarily a person.

- The person identified as chief operating decision maker is the person(s)


responsible for making strategic decision about the entity's segments (title or no
title)

- In line with the above, the 2 operations managers at segment 1 and 2 and the
General Manager overseeing their operations are qualified to be chief operating
decision makers.

b. AGBERO PLC Accounting for the revenue on the contract in the


financial statement

- Revenue should be measured at the fair value of the consideration received or


receivable.
- Fair value of the consideration, of deferred inflow of cash or cash equivalent
received is less than the nominal value, hence need to discount.
- Though Agbero Plc is providing interest free credit to buyer, interest must still be
imputed at the market rate of 6% given, based on revenue recognition, as defined in
the IASB Framework.
- The criteria for incorporating an item that meet definition of revenue in the
statements of profit or loss are as follows:

i. It is probable that any future economic benefit associated with the item of
revenue will flow to the entity;
ii. The amount of revenue can be measured with reliability;
iii. Must recognize revenue as work is performed throughout the
contract life; and
iv. Revenue should be discounted to reflect delay. Agbero Plc
should correct the prior period errors retrospectively in the financial
statements for the year ended March 31, 2018 - by re-
EXAMINER'S REPORT

The question tests candidates' knowledge on the presentation and operations of provisions of iFRS-8
- Operating segment and the treatment of accounting issues in the financial statements. The
question further test the recognition and how to account for the revenue contract in the financial
statements.

About 72% of the candidates attempted the question. Generally, candidates displayed poor
understanding of the question and their performance was poor.

Commonest pitfall was the inability of majority of the candidates to properly apply the provisions of
the accounting standards iFRS 8 and iFRS 15 to the situation depicted in the scenario.

Candidates should have adequate knowledge of accounting standards and be able to apply it to
practical scenarios. They should also use iCAN study texts for better performance in future
examinations.
Marking Guide Marks Marks
THE INSTITUTE OF CHARTERED ACCOUNTANTS OF NIGERIA

PROFESSIONAL LEVEL EXAMINATION - MAY 2019


CORPORATE REPORTING
Time Allowed: 3 1/4 hours (including 15 minutes reading time)

INSTRUCTION: YOU ARE REQUIRED TO ANSWER FIVE OUT OF SEVEN


QUESTIONS IN THIS PAPER

SECTION A: COMPULSORY QUESTION (30 MARKS)

QUESTION 1
Oyin Plc. a Nigerian company acquired 960 million equity share capital of Kemy Plc., a foreign
subsidiary based in Brazil, on 1 October, 2015 for 1.08 billion Brazilian real (BRL). The functional and
presentation currency of Kemy Plc. is the BRL. Since acquisition, Kemy Plc., has operated
autonomously of Oyin group.

The statements of financial position of Oyin Plc. and Kemy Plc. as at 30 September, 2017 are as
follows:
(i) Exchange rates are as follows:

Date BRL to N1
1 October, 2015 6.0
30 September, 2015 5.5
30 September, 2017 5.0
Average for the year to 30 September, 2016 5.2

Required:
Prepare the consolidated statement of financial position of Oyin group at 30 September,
2017. (30 Marks)

SECTION B: YOU ARE REQUIRED TO ANSWER ANY TWO OUT OF THREE


QUESTIONS IN THIS SECTION (40 MARKS)

QUESTION 2
Below is the draft financial statement of Lanwani Plc. a manufacturer of fast moving consumer
goods.
Statement of financial position as at
2017 2016
Non-Current Assets N'000 N'000
Property, plant and equipment 195,230 191,181
Intangible assets 148,277 99,477
Other non-current assets 1,226 1,927
344,733 292,585
Current Assets
Inventories 42,728 31,244
Trade receivables 20,384 19,974
Cash and bank 15,866 12,156
78,978 63,374
Total assets 423,711 355,959

Equities and liabilities:


Equities
Share capital (N1 each) 3,998 3,964
Share premium 73,770 64,950
Revaluation reserve 45,320 -
Retained earnings 99,692 96,343
222.780 165.257
Non-Current liabilities:
Loans and borrowings 5,000 17,000
Employee benefits 13,209 10,101
18.209 27,101
Current liabilities:
Bank overdraft 8,028 12,676
Current tax liabilities 19,606 19,024
Trade payables 155,088 131,901
182,722 163,601
Total equity and liabilities 423.711 355.959

Statement of profit or loss


2017 2016
N'000 N'000
Revenue 344,562 313,743
Cost of sales (201,103) (178,218)
Gross profit 143,459 135,525
Distribution expenses (66,898) (61,357)
Administrative expenses (21,747) (21,924)
Finance cost (10,419) (13,228)
Profit before tax 44,395 39,016
Income tax expense (13,581) (11,257)
Profit for the year 30.814 27.759

The following additional information are relevant:

(i) The Company changed its accounting policy from the cost model to the revaluation
model for its property. The amount in revaluation reserve represents the revaluation
surplus recognised in 2017. No adjustment was made in respect of 2016.

(ii) Development cost of N45 billion was capitalised during 2017. The related asset is not
expected to generate any economic benefit until 2020.

Required:

a. Assess the accounting treatment of the change in accounting policy and


state the impact on the return on capital employed (ROCE). (3 Marks)
b. Analyse the profitability, liquidity and efficiency of Lanwani Plc. (15 Marks)

c. Briefly discuss TWO limitations of the analysis done in (b) above. (2 Marks)
QUESTION 3

Ariba Bank Pic. (the Bank) is a Tier 1 Bank in Nigeria with branch network across all the six geo-
political zones of the country. Its credit portfolio is spread among many industries with a special
focus on the oil and gas industry and real estate. One of its major customers with a very good credit
standing is Dunga Property Development Company (DPDC).
The management of DPDC recently approved a plan to build four shopping malls in major cities
across the country. A special purpose entity was registered as a limited liability company, Dunga
Malls Limited (DML), dedicated to the development and management of the malls. The project will
be solely financed by a loan to be obtained from Ariba Bank. There will be no equity contribution
from DPDC other than the minimum required by law to establish a company.

Ariba Bank has approved a loan of N80 billion at a fixed interest rate of 15% per annum payable
annually in arrears. The loan has a maturity of 10 years with a moratorium of 3 years. There was no
transaction cost and therefore the contractual rate is the same as the effective rate. The loan was
granted directly to DML on 1 January, 2018.

The Financial Controller of Ariba Bank Plc. is concerned about the accounting treatment of the loan
as IFRS 9 Financial Instrument was adopted by the bank during the year. He noted that majority of
the bank loans areclassified at amortised cost in the statement of financial position but the loans
must pass certain tests before such classification.

The Chief Risk Officer noted in his memo that the arrangement is substantially the same as the other
borrowing arrangements of the bank except that a borrowing entity would normally have equity or
other assets that could be called upon by the bank in a case of default other than the asset being
financed.

Required:

a. Discuss how financial assets are classified in accordance with the


requirements of IFRS 9. (8 Marks)
b. Advise the Bank on how the loan granted to DML should be classified in the
statement of financial position. (6 Marks)

c. Discuss, with supporting calculations, how the loan will be accounted for in the financial
statement of the bank for the year ended 31 December, 2018
(6 Marks)
(Total 20 Marks)
QUESTION 4

The Central Bank of Kangora (CBK) operates a post-employment benefit plan whereby employees
are entitled to an amount upon completion of employment. Each employee is paid an amount equal
to 150% of the annual pay at the time of retirement multiplied by the number of years in service.
The plan is not funded.

CBK uses a professional actuary to determine its liability under the plan at the end of every
reporting period. The report of the actuary shows that the plan obligation was N620 million and
N906 million as at 1 January, 2018 and 31 December, 2018 respectively. The current and past
service cost for the year was N108 million. The discount rates were 8% and 12% as at 1 January,
2018 and 31 December, 2018 respectively.

CBK paid a total benefit of N48 million during the year.


The financial controller is struggling to complete the reconciliation and accounting entries for the
plan. He is particularly confused about the concept of remeasurement and its accounting treatment.

Required:

a. Differentiate between a defined contribution plan and a defined benefit plan and advise CBK
on how its post-employment plan should be classified.
(5 Marks)

b. Complete the reconciliation and show the journal entries required to record
the transactions for the year ended 31 December, 2018. (10 Marks)

c. Discuss the components of re-measurement gain or loss and state the


accounting treatment of a re-measurement gain or loss arising on a defined benefit plan. (5
Marks)
(Total 20 Marks)

SECTION C: YOU ARE REQUIRED TO ANSWER ANY TWO OUT OF THREE


QUESTIONS IN THIS SECTION (30 MARKS)

QUESTION 5

a. LPG Plc. is a publicly traded entity on the Nigerian Stock Exchange involved
in the production of and trading in natural gas in Nigeria. LPG Plc. jointly owns a gas storage
facility with another entity, Tan Oil Nigeria Limited. Both parties extract gas from onshore
gas fields in the Niger Delta, which they own and operate independently from each other.
LPG owns 55% of the gas storage facility and Tan Oil Nigeria owns 45%. Services and costs
are shared between them according to their percentage holding, however, decisions
regarding the storage facility require unanimous agreement of the parties. The gas storage
facility is pressurised so that the gas is pushed out when extracted. When the gas pressure is
reduced to a certain level, the remaining gas is irrecoverable and remains in the gas storage
facility until it is decommissioned. The Nigeria law requires the decommissioning of the
storage facility at the end of its useful life. LPG Plc. wishes to know how to treat the
agreement with Tan Oil Nigeria Limited including any obligation or possible obligation
arising on the gas storage facility.

NB Ignore accounting for the irrecoverable gas.

b. LPG purchased a major gas plant on 1 January, 2018 and the Directors estimated that a
major overhaul is required every two years. The costs of the overhaul are approximately
N25 million which comprises N15 million for parts and equipment and N10 million for
labour. The Directors proposed to accrue the cost of the overhaul over the two years of
operations up to that date and create a provision for the expenditure.

Required:
Discuss, with reference to International Financial Reporting Standards (IFRS), how LPG Plc should
account for the agreement in (a) above (11 marks) and the transactions in (b) for its year ended 31
August, 2018.
(4 marks)
(Total 15 Marks)

QUESTION 6
Dangogo Plc. has adopted 1FRS in the preparation and presentation of its financial statements in
line with Financial Reporting Council of Nigeria requirements. During deliberations on their financial
statements for the year ended 31 March, 2019 the directors of Dangogo Plc. found the distinction
between profit or loss and other comprehensive income confusing. This is the case with many other
preparers or users of financial statements in Nigeria who seem to be unclear about the relationship
between profit or loss and other comprehensive income (OCI). They blame the conceptual
framework for Financial Reporting and IAS 1 regarding the confusing nature of re-classification. The
emergence of integrated reporting holds promises for better reporting, but preparers are equally
uncertain about whether the International Integrated Reporting Councils (IIRC) or Integrated
Reporting (IR) Framework constitutes suitable criteria for report preparation.
Required:
a. Discuss the nature of a re-classification adjustment and the arguments for and against
allowing re-classification of items to profit or loss. (6 Marks)
bi. Discuss the objectives of integrated reporting and key components
(content elements) of integrated report. (6 Marks)

ii. Comment on any concerns which could limit the Framework's suitability for
assessing the performance and prospects of an entity. (3 Marks)
(Total 15 Marks)

QUESTION 7

a. Agbinye Farms operates many plantations across Nigeria. The Company


recently acquired a freehold land in Benue for a total of N12 million. The trees were planted
with the company incurring an operating cost of N4 million up to 31 March, 2018 which is
the company's year end.

The fair value of the plantation (excluding the land) was determined to be N16.4 million as
at 31 March, 2018. Based on management assessment, the company is expected to get
produce from the plantation for a period of 20 years. The first harvest was done during the
year ended 31 March, 2018 and the fair value of the produce was estimated as N2.5 million.
The Company incurred a total cost of N600,000 to complete the harvest.

The company uses the cost model when possible.

Required:

a. Discuss the accounting treatment of the above transactions showing clearly


the amount to be recognised in the statement of profit or loss and statement of financial
position as at 31 March, 2018. (10 Marks)

b. Megida Plc. has ranches across the North where its cattle are reared. The company is quoted
on the Nigerian Stock Exchange under Agricultural Sector. The Financial Controller is not
clear as to how to measure its cattle in the financial statement according to IAS 41.

Required:
Advise the Financial Controller on how to measure the cattle in the financial
statement of Megida Plc. (5 Marks)
(Total 15 Marks)
SOLUTION 1

OYIN GROUP PLC

Consolidated Statement of Financial Position as at 30 September, 2017

N'000 N'000
Assets:
Non-current assets:
Property, plant & equipment (945 + 240) 1,185,000
Goodwill on consolidation (w4) 57,600
Total Non-current assets 1.242.600
Current Assets
Inventories (375 + 90 - 3.75) 461,250
Trade receivables (w8) 369.000
Cash (w9) 120,000 950.250
Total Assets 2.192.850
Equity and liabilities
Equity
Ordinary share capital of N0.50 each 450.000
Retained Earnings (w5) 577,200
Revaluation reserves 18.000
1,045,200
Non-controlling interests (w7)
43,650
Non-current liabilities
10% loan notes (300+75) 375.000
8% redeemable preference shares (90+30) 120,000 495,000
Current liabilities
Trade payables (375+60) 435.000
Taxation (105+24) 129.000
Bank overdraft (45+0) 45,000
609.000
2.192.850
W2 Adjustment for revaluation of PPE BRL'000
PPE 1 October, 2016 375.0
Depreciation 1/25 (15,000)
Carrying amount Revalued amount 360.000
Revaluation reserves Dr. PPE 450.000
Cr. Revaluation reserves 90.000
90,000
90.000

W3 Translation of statement of financial position BRL'000 Rate N'000


of Kemy Plc.
Property, plant & equipment 1,200,000 5 240,000
Inventories 450,000 5 90,000
Trade receivables 420,000 5 84,000
Cash 75,000 5 15,000
Total Assets 2.145.000 429.000
Ordinary share capital 600,000 6 100,000
Revaluation reserves Retained earnings: 90,000 5 18,000

Pre-acquisition 390,000 6 65,000


Post-acquisition 1.120.000 Bal 57.000
10% loan notes 375,000 5 75,000
8% redeemable preference shares 150,000 5 30,000
Trade payables 300,000 5 60,000
Taxation 120,000 5 24,000
Total equity and liabilities 2.145.000 429.000

W4 Goodwill on acquisition BRL'000 Rate N'000

Investment at cost N180m x 6 1,080,000 6 180,000


Non-controlling 20% interest 198.000 6 33.000
1.278.000 213.000
Ordinary share capital 600,000 6 100,000
Retained earnings 390.000 6 65,000
990.000
Goodwill at acquisition Translation of goodwill 88.000 165.000

30 September, 2017 288,000 5 57,600


At acquisition 288,000 6 48.000
Translation gain on goodwill 9.600
EXAMINER'S REPORT

The question tests candidates' understanding of the preparation of consolidated statement of


financial position of foreign entities.
All the candidates attempted the question and the performance was below average.
The commonest pitfall was the inability of candidates to properly translate the line items of
statement of financial position and goodwill using appropriate rates.
The examiners have been consistent in examining this aspect of the syllabus hence, candidates are
advised to concentrate on all aspects of the consolidated financial statements for better
performance in future examinations.
Solution 2

(a)
Accounting policies are rules, principles, convention that underpin the preparation of financial
statement.
Any accounting policy chosen by an entity must be consistently applied. A change can only be made
if it is required by IFRS or if the change will bring about a more relevant and reliable information.
• If a change in accounting policy is as a requirement of an IFRS, the policy should be applied
in the light of the IFRS.
• Retrospective application is not applied to change in cost model to revaluation model.
However, when there is a change in accounting policy, such a change must be applied
retrospectively, that is, going backward to re-instate the comparative figures except where it is
impracticable to do so. It is considered impracticable only when reasonable effort has been made.
Considering the financial statements of Lanwani Plc, if a retrospective application is not applied,
such inconsistency will impact on the financial statement analysis e,g on the Return on capital
employed (ROCE). The revaluation surplus and the development cost capitalized will inflate the
capital employed for the year 2017 and consequently lead to a deterioration of the ROCE in 2017
compared with the prior year.
b) LANWANI PLC

COMPUTATION OF FINANCIAL RATIOS FOR THE YEAR ENDED 2017


BRIEF COMMENTS ON LANWAN1 PLC

FINANCIAL STATEMENT ANALYSIS FOR THE YEAR ENDED 2017


This analysis is based on the financial statements of Lanwani Plc, a manufacturer of fast moving
consumer goods, on the profitability, liquidity and efficiency of the company.

PROFITABILITY
These ratios measure how well the entity is performing in the area of returns. The higher the ratios
the better. The primary ratio here is ROCE; that measures the overall profitability and efficiency of
an entity.

The ROCE has significantly declined from 28.7% in 2016 to 24.1% in 2017. This reduction may not be
unconnected with the revaluation surplus and development cost capitalised for the year that
inflated the capital employed. This slump could also be traceable to the falling profit margins.

Management is advised to look inward especially into the production area and the policy on
overhead with a view to cutting down unnecessary wastages.
EFFICIENCY

The ability of an entity to use it assets to generate revenue is measured by the efficiency ratios. The
higher the times the better. The principal ratio here is the asset turnover.

The asset turnover has slightly diminished from 1.7 times in 2016 to 1.5 times in 2017. In the same
vein, the inventory turnover nosedived from 5.7 times to 4.7 times. However, in 2017 Lanwani Plc
has come to be more efficient in the collection of receivables from the customers while the trade
payables are gradually on the increase, the payment period is unreasonably long.

Management is advised to dispose off or possibly replace the non-performing noncurrent assets.
The use of Just-in-time (JIT) system of inventory control could be contemplated to improve on the
inventory holding period.

LIQUIDITY
These ratios measure the ability of an entity to settle its short-term obligations as they fall due. The
higher the ratios the better. The key ratios here are the current, quick and cash ratios.

The liquidity position of Lanwani Plc is really worrisome as evidenced in the key ratios
aforementioned, although these ratios marginally increased from 2016 to 2017 but they are far
below the generally acceptable industry average. The two year ratios are very poor but the current
year is better when compared with the previous year.

Management is advised to consider disposing non-performing assets to cushion the liquidity


position.

In conclusion, the financial performance, efficiency and liquidity position of Lanwani Plc is poor.
However, if the recommendation aforesaid are implemented and a team of professionals are
injected to manage the affairs of the company, the fortune can be turn around.

(c)
The two limitations of ratios
(i) The use of different accounting policies:
Using ratio analysis to compare entities on year by year basis will be misleading where the
preparation of the financial statement are based on different accounting policies.
(ii) Entities in different locations:
Where entities are not in the same location, drawing conclusions from the ratios computed
could be misleading because they are not in the same terrain.
(iii) The use of different accounting dates
(iv) Effect of inflation
(v) Manipulation of accounts
EXAMINER'S REPORT

The question tests accounting treatment of the changes in accounting policy and its impact on the
profitability, liquidity and efficiency of an entity.
About 90% of the candidates attempted the question and the performance was above average.
The commonest pitfall was inability of the candidates to classify ratios appropriately into
profitability, efficiency and liquidity and give recommendations based on the ratios calculated.
Candidates are advised to pay particular attention to ratio analysis, interpretation and limitation of
ratios for better performance in their examination.

Marking Guide

Question 2
a. Definition of accounting policy 1
Reasons for change of accounting policy 1
Implications on Lanwani Plc ROCE 1 3

b. Computation of three profitability ratios and comments 5


Computation of Efficiency ratios and comments 5
Computation of liquidity ratios and comments 5 15
c. Any two limitations of ratios 2
20
Solution 3

Ariba Bank
(a) According to IFRS 9, on initial recognition, the classification of a financial asset depends on
whether it is a debt instrument or an equity instrument. A debt instrument will normally be
classified at amortised cost, fair value through other comprehensive income (FVOCI) and fair
value through profit or loss (FVTPL). An equity instrument cannot be classified at amortised
cost.

Classification of a debt instrument will depend on whether the debt instrument passes the
cash flow test (normally referred to as the solely payment of principal and interest (SPPI) test)
and the business model test.
A debt instrument is generally measured at amortised cost if both of the following conditions
are met:
(i) The asset is held within a business model whose objective is to hold assets in order to
collect contractual cash flows (business model test)
(ii) The contractual terms of the financial asset give rise on specified dates to cash flows
that are solely payments of principal and interest (SPP1) on the principal amount
outstanding (contractual cash flows characteristics test)
A debt instrument is normally measured at FVOC1 if both of the following conditions are met:

(i) The asset is held within a business model in which assets are managed to achieve a
particular objective by both collecting contractual cash flows and selling financial assets
(ii) The contractual terms of the financial asset give rise on specified dates to cash flows
that are solely payments of principal and interest (SPP1) on the principal amount
outstanding (contractual cash flows characteristics test)

The requirements above are applied to an entire financial asset, even if it contains an
embedded derivative.
A debt instrument that is not measured at amortised cost or at FVOC1 must be measured at
FVTPL.

(b) The objective of a bank would normally be to collect contractual cash flow, therefore it could
be assumed that the business model objective for amortised cost has been met. However, it is
important to consider whether the contractual terms give rise to cash flows that are SPP1.

SPP1 implies that the Bank will be compensated for time value of money and the credit risk it
is exposed to. Compensation for liquidity risk could also be considered for the purpose of
SPP1 but not the exposure to the risk of the underlying asset.
1n the lending to DML, it could be argued that Ariba Bank is exposed to the risk of the
underlying asset and facing a risk similar to that of an equity owner since the projects
undertaken by DML are solely financed by the loan and there is no equity.
However, Ariba Bank could gain comfort on the basis that the loan was guaranteed by DPDC
with a very good credit standing.
Based on the above, the loan will pass the business model test and SPP1 test for classification
as a debt instrument at amortised and should be classified as such.
(c) Debt instruments classified at amortised cost are initially measured at fair value including
transaction cost. In this case, the fair value of the loan granted will be the total face value of
N80 Billion less the transaction of N2 Billion deducted at source I.e. N78 Billion. It should be
noted that the N2 Billion is not recognised immediately in profit or loss, rather it is
recognised over the life of the loan as part of the effective interest.
Subsequently, it is measured at amortised cost. The amortised cost of a financial instrument
is defined as the amount at which it was measured at initial recognition minus principal
repayments, plus or minus the cumulative amortisation using the 'effective interest method'
of any difference between that initial amount and the maturity amount, and minus any write-
down.

The carrying amount of the loan at year end will be as follows:

N'million

Fair value at 1 January 2018 (80,000 - 2,000) 78,000

Finance income @ EIR of 16% (78,000 x 16%) 12,480

Interest received (80,000 x 15%) (12,000)

Amortised cost at 31 December 2018 78,480


Finance income of N12.48 Billion will be recognised as revenue while the carrying amount at
year end in the statement of financial position will be N78.48 Billion. It should be noted that
an impairment loss based on expected credit loss will be recognised on the asset.

EXAMINER'S REPORT

The question tests classification of financial assets in accordance with the requirements of IFRS 9
and the required tests before such classification.

About 70% of the candidates attempted the question and the performance was below average.
The commonest pitfall was the inability of the candidates to differentiate classification under IFRS
39 from the classification under IFRS 9.
Candidates are advised to review and learn new international financial reporting standards (IFRSs),
their implications and challenges for better performance in future examinations.
Solution 4

(a) Differences between Defined Contribution Plan and Defined Benefit Plan

Defined Contribution Plan


This is a benefit plan where the entity (employer) and employee both pay a
fixed contribution into a separate Entity and the employer has no legal or
constructive obligation to pay further contribution if the fund does not hold
sufficient assets to pay all employee benefits relating to their service. The
risk and rewards have been transferred to the employee
Defined Benefit Plan
This is a plan where an entity (employer) guarantees a particular level of
pension benefit to its employees upon retirement, employees will receive
annual payment from the fund based on their number of year of service and
their final salary.
The entity has obligation to pay extra funds into the pension plan to meet
this promised level of pension. The risk is with the employer.
Based on the foregoing, CBK post-employment plan might appear to be a
defined benefit plan and should be classified as such. This is evident
because CBK's employees are said to be entitled to "an amount equal to
150% of the annual pay at the time of their retirement multiplied by the
numbers of years in service".

(b) Reconciliation

Present Value of Pension Obligation


N'million
Opening Balance 620 @ 1/1/2018
Interest cost @ 8% (620 X0.08) 49.6
Current & Past108 Service Cost
Benefit (48) Paid
Re-measurement 176.4 Balancing figure loss
Closing balance @ 31/12/2018 906
Journal entries:
(i) Interest cost
Dr. Profit or loss N49.6
Cr. Plan obligation N49.6
To recognise the interest cost on defined benefit plan obligation

(ii) Current and past service cost


Dr. Profit or loss N108
Cr. Plan obligation N108
To recognise the service cost on defined benefit plan obligation

(iii) Benefits paid


Dr. Plan obligation N48
Cr. Bank N48
To recognise the amount of benefit paid to past employees
(iv) Re-measurement
Dr. Other comprehensive income N176.4
Cr. Plan obligation N176.4
To recognise the loss arising from the re-measurement of plan obligation
• Contributions into the plan
• Benefits paid

After the components above are computed, the net pension deficit could differ from the
amount calculated by the actuary as at the current year end.
This is for several reasons that include the following:
• The actuary's calculation of the value of the plan obligation and assets is based on
assumptions, such as life expectancy and final salaries, inflation, implicit rate of interest. All
these would have changed year-on-year basis.
• An adjustment, known as the "re-measurement component" must therefore be posted. This
is charged or credited to other comprehensive income for the year and identified as an item
that will not be reclassified to profit or loss in future periods.
EXAMINER'S REPORT
The question tests the classification of post - employment plan under defined contribution plan and
defined benefit plan. 1t also tests accounting treatment of re-measurement gain or loss and how the
components are classified.

More than 80% of the candidates attempted the question and the performance was
above average.
The commonest pitfall was that some of the candidates that attempted the question displayed lack
of clear understanding of the difference between defined contribution plan and defined benefit plan
as stated in the accounting standards.
Candidates are advised to have deep knowledge of the different aspect of the syllabus in order to
perform better.
a. Discussion of re-measurement gain or loss
Accounting treatment of a re-measurement gain or loss arising on
defined benefit plan 2 5
20

Solution 5
(a) The classification of a joint arrangement as a joint operation or a joint venture depends upon
the rights and obligations of the parties to the arrangement (IFRS 11 Joint Arrangements).

• A joint arrangement occurs where two or more parties have joint control.

• The parties to the joint arrangement contractually agree that sharing of


control exists only when decisions about the relevant activities require the unanimous
consent of the parties sharing the control.

• The structure and form of the arrangement determines the nature of the relationship.

• Irrespective of the purpose, structure or form of the arrangement, the


classification of joint arrangements under IFRS 11 - Joint Arrangements
depends upon the parties' rights and obligations arising from the arrangement.

• A joint arrangement which is not structured through a separate vehicle is a joint operation. In
such cases, the contractual arrangement establishes the parties' rights and obligations.

• A joint operator accounts for the assets, liabilities, revenues and expenses relating to its
involvement in a joint operation in accordance with the relevant IFRSs.

• Based on the above, the arrangement between LPG Plc and Tan Oil Nigeria Ltd qualifies as a
joint operation because:

o there is no separate vehicle involved;


o they have agreed to share services and costs;
o decisions regarding the platform required unanimous agreement of the
parties.

• The joint venture here relates to ownership of storage facility that would be used for more
than one accounting year.
• IAS 16 states that property, plant and equipment are tangible items which:
- are held for use in the production or supply of goods or services, for rental to others, or
for administrative purposes; and
- are expected to be used for more than one accounting period.

Consequently, LPG Plc should recognise its share of the asset as property, plant and
equipment.

Under 1AS 16 property, plant and equipment (PPE), the cost of an item of property, plant and
equipment includes the initial estimate of the costs of dismantling, removal of the item and
restoration of the site on which it is located.

1AS 37 Provisions, Contingent Liabilities and Contingent Assets contains requirements on how
to measure decommissioning, restoration and similar liabilities.

Where the effect of the time value of money is material, the amount of a provision should be
the present value of the expenditures expected to be required to settle the obligation.

The costs incurred by an entity in respect of obligations for dismantling, removing and
restoring the site on which an item of property, plant and equipment is located are recognised
and measured in accordance with 1AS 16 and 1AS 37. LPG Plc should recognise 55% of the
cost of decommissioning the underground storage facility.

However, because LPG Plc. is a joint operator, there is also a contingent liability for 45% of the
decommissioning costs.

The possible obligation for the remainder of the costs of decommissioning depends on the
occurrence of some uncertain future event, such as if Tan Oil Nig Ltd goes into liquidation and
cannot fund the decommissioning costs.

Therefore, LPG Plc should also disclose a contingent liability relating to the Tan Oil Nigeria
Ltd's share of the obligation to the extent that it is contingently liable for Tan Oil Nigeria Ltd's
share.

(b) 1t is not acceptable to accrue the costs of the overhaul.

1AS 37 requires a provision to be recognised if the following conditions are met


- Contractual or legal obligations exist as a result of past event
- 1f it is probable that there will be an outflow of economic benefits
- 1f it can be measured reliably
The entity does not have a constructive obligation to undertake the overhaul. Under IFRS, The
major overhaul component of N15m will then be depreciated on a straight-line basis over its
useful life (i.e. over the period to the next overhaul) and any remaining carrying amount will
be derecognised when the next overhaul is performed.

Costs of the day-to-day servicing of the asset (i.e. routine maintenance labour cost of N10m)
are expensed as incurred.

Therefore, the cost of the overhaul should have been identified as a separate component of
the gas storage at initial recognition and depreciated over a period of two years. This will
result in the same amount of expense being recognised in profit or loss over the same period
as the proposal to create a provision in the financials of LPG.

EXAMINER'S REPORT
The question tests application of international financial reporting standards (IFRSs) on joint
arrangements; property, plant and equipment and provisions, contingent liabilities and contingent
assets to given scenarios.

About 80% of the candidates attempted the question and performance was slightly above average.

The commonest pitfall was lack of deep knowledge and application of the applicable IFRSs to
practical situations.

Candidates are advised to understand the provisions of IFRSs and their applications to issues in real
life situations at this level of the Institute examination for better performance.

Marking Guide

Question 5
a. Accounting for the agreement with reference to IFRS on:
- Qualification of LPG Plc and Tan Oil Nig. Ltd. Under IFRS II
5
- Joint venture agreement with reference to ownership of store facility
under IAS 16 on PPE 3
- Measurement of decommissioning restoration and similar liabilities
under IAS 37 3 11
b. Conditions for recognizing provisions under IAS 37 2
Accounting for the cost of overhaul
2 4
15
Solution 6

(a) Re-classifications adjustments are items that have been recognised in other comprehensive
income (OCI) in the current or previous periods now converted or recycled to profit or loss in
the current period
An example of items recognised in OCI which may be reclassified to profit or loss include
realised gains or losses on cash flow hedges and foreign currency gains on the disposal of a
foreign operation.
Certain items recognised in OCI may not be reclassified to profit or loss.
Examples of items which may not be reclassified are changes in a revaluation surplus under
IAS 16 Property, plant and equipment, and actuarial gains and losses on a defined benefit plan
under IAS 19 Employee Benefits.
The directors of Dangogo Plc may find distinction between profit or loss and other
comprehensive income challenging because, there is a general lack of agreement about which
items should be presented in profit or loss and in OCI.
The interaction between profit or loss and OCI is unclear, especially the notion of
reclassification and when or which OCI items should be reclassified.
A common misunderstanding is that the distinction is based upon realised versus unrealised
gains.
Arguments in favour of reclassification include
• If reclassification ceases, there would be no need to define profit orn loss, or
any other total or subtotal in profit or loss, and any presentation decisions can be left to
specific IFRS.
• It is argued that re-classification protects the integrity of profit or loss and provides users with
relevant information about a transaction which occurred in the period.
• Additionally, it can improve comparability where IFRS permits similar items to be recognised in
either profit or loss or OCI.
Those against re-classification argue that:
• The recycled amounts add to the complexity of financial reporting;
• Re-classification may lead to earnings management;
• The re-classification adjustments may not meet the definitions of income or
expense in the period as the change in the asset or liability may have occurred in a previous
period.
• The lack of a consistent basis for determining how items should be presented
has led to an inconsistent use of OCI in IFRS.
Opinions vary but there is a feeling that OCI has become a home for anything controversial
because of a lack of clear definition of what should be included
in the statement. Many users are taught to ignore OCI, as the changes

reported are not caused by the operating flows used for predictive purposes.

(b) Objectives of integrated reporting

The objectives of integrated reporting include:


- To improve the quality of information available to providers of financial capital to enable a
more efficient and productive allocation of capital

- Provide a more cohesive and efficient approach to corporate reporting that draws on
different reporting strands and communicates the full range of factors that materially affect
the ability of an organisation to create value overtime

- Enhance accountability and stewardship for the broad base of capitals (financial,
manufactured, intellectual, human, social relationship) and promote understanding of their
interdependencies

- Support integrated thinking, decision-making and actions that focus on the creation of value
over the short, medium and long term.

- To integrate both qualitative and quantitative report for effective decision making
Content elements of Integrated Reporting Framework
Q Organisational overview and external environment - What does the
organisation do and what are the circumstances under which it operates?
Q Governance - How does an organisation's governance structure support
its ability to create value in the short, medium and long term?
Q Business model - What is the organisation's business model?
Q Risks and opportunities - What are the specific risk and opportunities
that affect the organization's ability to create value over the short, medium and long
term, and how is the organisation dealing with them?
Q Strategy and resource allocation - Where does the organisation want to
go and how does it intend to get there?
Q Performance Measurement - To what extent has the organisation
achieved its strategic objectives for the period and what are its outcomes in terms of
effects on the capitals?
Q Future Outlook - What challenges and uncertainties is the organisation
likely to encounter in pursuing its strategy, and what are the potential implications for
its business model and future performance?
Q Basis of preparation and presentation - How does the organisation determine what
matters to include in the integrated report and how such matters are quantified or
evaluated.
(b) ii.

Comment on any concerns which limit the framework's suitability for assessing the performance and
prospects of an entity
- With the development of the 11RC Framework, defining the "providers of financial capital" as
the primary target audience of the integrated report was an informed decision. A key point
here is that disclosing other information on the performance and direction of an enterprise
is still relevant in the annual report.
- The firm sustainability reporting still targets various types of stakeholder groups who are not
the primary audience of the integrated report. The integrated report is of special value to
those user groups who want to see a connection made between sustainability performance
and financial performance. This means it's of special value to the various providers of
financial capital. 1nstitutional investors and those committed to responsible investment
have a special interest in integrated reports.
- To the providers of capital it's critical that reporting is more strategic and forward looking as
opposed to annual reporting which has been backward looking and only focused on narrow
corporate financial performance of the previous year. Describing the logic of how the
enterprise generates income and wealth, where its growth is heading in the longer term,
how it uses different types of capital and manages the risks involved is especially relevant to
responsible investors in their decision-making.
EXAMINER'S REPORT
The question tests the nature of re-classification adjustment of items into profit or loss; the
objectives and key components of integrated reporting and the frameworks suitable for assessing
the performance and prospects of an entity.
More than 90% of the candidates attempted the question and the performance was above average.
Most of the candidates that attempted the question displayed poor understanding of re-
classification adjustments of items into profit or loss.
Candidates are advised to have good knowledge of the area of the syllabus and beyond financial
reporting for better performance.
Marking Guide

Solution 7

(a) Accounting Treatment of Agbinye Farms.

The relevant standards for consideration here are:

• IAS 41- Agriculture


• IAS 2 - Inventory
• IAS 16 Property, plant and equipment
IAS 41 Agriculture outlined assets that are outside of its scope. This includes
- Bearer plants
Bearer plants are used to produce agricultural produce for more than one period. Examples
include grape vines or tea bushes. Bearer plants are accounted for in accordance with IAS 16
Property, Plant and Equipment. However, any harvested produce growing on a bearer plant,
such as grapes on a grape vine, is a biological asset and so it is accounted for in accordance
with IAS 41.

- Land related to agricultural activity.


Land is not a biological asset. It is treated as a tangible non-current asset and accounted for
under IAS 16-Property, Plant and Equipment. When valuing a forest, for example, the trees
must be accounted for separately from the land that they grow on.

As explained above Agbinye farms operates a plantation, this should be accounted for in
accordance with property plant and equipments.

Accounting Treatment
Initial Measurement of Land The

initial cost of land is N12m

- Subsequent Measurement of Land


For land it remains the same at N12m in the statement of financial position as at 31 March,
2018 since land cannot be depreciated.
- Initial Measurement of Trees (Plantation)
The N4m operating cost of planting the trees should be taken as the initial cost of PPE i.e the
Plantation.

- Subsequent Measurement of Trees (Plantation) using cost model

Cost (Initial) 4,000,000

Depreciation (4.000.000) (200.000)


20
3.800.000
Carrying amount at 31/3/2018

DR: Profit or loss depreciation 200,000


CR: Property Plant & Equipment 200,000
Being depreciation for the year

Subsequent measurement of trees using revaluation model.


N

Cost (Initial) 4,000,000


Depreciation (4,000,000) ( 200,000)
20
3,800,000
Revaluation Surplus 12,600.000
Revaluation 16,400.000

DR: Profit or loss 200,000


CR: PPE 200,000
Being depreciation for the year

DR: PPE 12,600,000


CR: Revaluation Reserve (OC1) 12,600,000
Being Revaluation gain of plantation at 31/3/2018
Extract of Statement of profit or loss N'm

Fair value gain 2.5


Operating cost (0.6)
Depreciation (0.2)
Extract of Statement of financial position
Non-current asset N'm
Land 12
Bearer plants (4,000,000 - 200,000) 3.8

Current asset
Agricultural produce 25

Agricultural Produce
The produce growing from the trees in the plantation are biological assets. They should initially be
recognised at fair value less costs to sell. Any gain or loss on initial recognition is reported in
Statement of profit or loss and be revalued at the year end to fair value less costs to sell with any
gain or loss reported in profit or loss
Therefore, in Agbinye farms, the harvested produce are agricultural produce and should be
accounted for in line with provisions of 1AS 41.
Initially recognised at N1.9m (2.5m - 0.6m), with a gain of N1.9m reported in profit or loss, the
harvested grapes are now accounted for under 1AS 2 Inventories and will have a deemed cost of
N1.9m.
b. Megida Plc.
Living animals are biological assets that are within the scope of 1AS-41 Agriculture. An entity
applies this standard so far it is engaged in agricultural activities which involve the
management of biological transformation of biological assets.
If it's assumed that Megida cattle are majorly diary cattle.
Megida should apply the following standards in line with 1ASB recommendations
Diary cattle - Recognised as property plant and equipment and depreciated over the useful life
of the cattle, if the cattle were used to produced milk
Milk - This should be recognised as agricultural produce in line with 1AS 41 as explained above
Cheese -They should apply 1AS 2 because these are products that are the result of processing
after harvest.
Alternatively,

At initial recognition, they will be accounted for at fair value less cost to sell, except where the
fair value cannot be determined reliably hence, it is initially measure at cost. This is unlikely to
be the case for Agbinye especially because of its target market.
Subsequently, it will be re-measured at the end of every reporting period at fair value less cost
to sell. Change in the fair value less cost to sell will be recognised directly in profit or loss.
It will be presented as biological asset in the statement of financial position which will be a
separate class of asset.

EXAMINER'S REPORT

The question tests accounting treatment of

- plantation and land related to agricultural activities; and


- ranches for agricultural company quoted on the Nigerian Stock Exchange.
About 70% of the candidates attempted the question and the performance was poor.
The commonest pitfall was lack of understanding of the treatment of plantation and ranches, linking
them to the relevant international accounting standards like IAS 41, Agriculture; IAS 2 - inventory
and IAS 16 - property, plant and equipment. The candidates' displayed poor understanding of the
differences between cost model and revaluation model.
Candidates are advised to understand all aspect of the syllabus for better performance.
Marking Guide
a. Identification of the relevant International Accounting Standards

3
- Accounting treatment for cost model 2
- Accounting treatment for revaluation model 2 4
- Amount to be recognised in the statement of P or L 1½
- Amount to be recognised in the statement of financial
position 1½
b. Introduction of cattle as biological assets 2 3
1
- Treatment as
1
Diary cattle
- Treatment as 1 5
Milk 15
- Treatment as
Cheese

You might also like